MCAT Chemistry

Ace your homework & exams now with Quizwiz!

Using a given mass of KClO3, how would one calculate the mass of oxygen produced in the following reaction, assuming it goes to completion? 2 KClO3 → 2 KCl + 3 O2

(Grams of KClO3 consumed) (3 moles of O2)(molar mass of O2)/ (molar mass KClO3)(2 moles KClO3) Keeping in mind that molar mass is measured in grams of a substance per moles of that substance, you need something that comes out with the units of grams of oxygen.

Ethanol

C2H5OH Molar mass of about 46

How many electrons are involved in the following half-reaction after it is balanced? Cr2O72- + H+ + e- → Cr2+ + H2O A. 2 B. 8 C. 12 D. 16

Correct Answer: B Explanation: First, balance the atoms in the equation:Cr2O72- + 14 H+ → 2 Cr2+ + 7 H2O Now, adjust the number of electrons to balance the charge. Currently, the left side has a charge of +12 (-2 from dichromate and +14 from protons). The right side has a charge of +4 (+2 from each chromium cation). To decrease the charge on the left side from +12 to +4, we should add 8 electrons:Cr2O72- + 14 H+ + 8 e- → 2 Cr2+ + 7 H2O

Acetone

CH3COCH3 Molar mass = 58 g/mol

Which group has highest priority when naming? Alcohol, alkane, aldehyde, Or carboxylic acid

Carboxylic acid: C is most oxidized (connected to two Oxygens) Carboxylic acid C will always be #1

Metathesis Reaction

(Double Displacement)

3/8

0.375

5/8

0.625

Which of the following statements best describes the effect of lowering the temperature of the following reaction? A+B = C+D Delta H = -1.12 kJ/mol A. [C] and [D] would increase. B. [A] and [B] would increase. C. ΔH would increase. D. ΔH would decrease.

Correct Answer: A Explanation: A negative ΔH value indicates an exothermic reaction, meaning that the forward reaction produces heat. Visualize this as follows: A + B <-> C + D + heat This means that removing heat by decreasing the temperature is similar to removing any other product of the reaction. To compensate for this loss, the reaction will shift to the right, causing an increase in the concentrations of C and D as well as a decrease in the concentrations of A and B.

Determine the vapor pressure of a solution containing 190 g MgCl2 in 540 g water at room temp VP of pure water = 3.2 kPa

190 MgCl2 / 95.3 g/mol = 2 mol MgCl2 540 g H2O = 30 mol H2O X of water = 30 moles of water / 32 moles total P of water = X water * P0 of water = 30/32 * 3.2 kPa = 3.0 kPa

Balance the reaction: C4H10 (l) + O2 (g) -> CO2 (g) + H2O (l)

2 C4H10 (l) + 13 O2 (g) -> 8 CO2 (g) + 10 H2O (l) *First, balance least common atoms *Then, balance more common (usually H and O) *Balacne change if necessary

Maillard Reaction

A nucleophlilic reaction between amino terminus of peptide chain of protein and carbonyl functionality of sugar to form N-substituted glycoslamine -155 degrees celcuis -Browning meat -Gives cooked food pleasing odor and taste

A positive emf means the cell is ___________Spontaneous or non spontaneous (____________) and a negative emf means the cell is ___________Spontaneous or non spontaneous (____________) Ecell = -0.59 V: E cell = 0.25 V

A positive emf means the cell is Spontaneous (galvanic) and a negative emf means the cell is non spontaneous (electrolytic) Ecell = -0.59 V: -> electrolytic E cell = 0.25 V -> galvanic

Entropy (S)

A property related to dispersion of energy through a system or degree of disorder in that system; the change in entropy (delta S) in the course of a reaction is the difference between the entropies of the products and the reactants.

What is the product of reacting a ketone with LiAlH4?

A secondary alcohol

As electrons go from a lower energy level to a higher energy level:

AHED (absorb light, higher potential, excited, distant from nucleus)

Arrhenius Acids and bases

Acid will dissociate to form excess H+ in solution (contain H at the beginning of formula: HNO3, HCl...) Base will dissociate to form excess OH- (Contain OH at end of formula: NaOH, Fe (OH)3....) Most restrictive Limited to aqueous acids and bases Every Arrhenius base can be clarified as Bronsted-Lowry and Lewis base (same for acid), but it doesn't always work the other way around

Order of reactivity for carboxylic acid derivatives

Acyl halides (most reactive, least stable, example is Acetyl chloride)> acid anhydrides> esters> amides

At sea level and 25°C, the solubility of oxygen gas in water is 1.25 × 10-3 M. In Denver, a city in the United States that lies high above sea level, the atmospheric pressure is 0.800 atm. What is the solubility of oxygen in water in Denver? A. 1.00 × 10-3 M B. 1.05 × 10-3 M C. 1.50 × 10-3 M D. 2.56 × 10-3 M

Correct Answer: A Explanation: The solubility of gases in liquids is directly proportional to the atmospheric pressure. This is called Henry's law. Even without the formula, we know there is a direct proportionality between partial pressure and solubility. Because 0.800 atm is 80% of the pressure at sea level (1 atm), oxygen's solubility will be 80% of 1.25 × 10-3, which is 1.00 × 10-3 M

As NAD+ is converted into NADH during fermentation, it is acting as:

An oxidizing agent Gains electrons (gets reduced) = oxidizing

SN2 reaction

Back side attack. Inverted stereochemistry (R)--> (S) Primary alkyl halide substrate most likely One step(concerted rxn) Bimolecular rxn: Rate will increase when the concentration of either substrate or nucleophile is increased Best in polar aprotic (DMSO)

What is the molar solubility of Zn(OH)2 (Ksp = 4.1 X 10^-17) in a 0.1 M solution of NaOH?

Balanced equation: Zn(OH)2 -> Zn^2+ + 2 OH- Ksp = (x)(0.1 + 2x)^2 (x is negligible for OH) Ksp = (x) (0.1)^2 4.1 X 10^-17 = 0.01 x x = 4.1 X 10^-15 = molar solubility of Zn(OH)2

What will result in greatest amount of copper plated from copper (II) sulfate solution?

Current with highest amount of amperes in longest amount of time Current: Charge over unit of time (q/t), so current X t = greatest amount of charge I * t = F * n where F = faraday constant and n = moles of metal plated

The following reaction has a Keq of 2.1X10^-7. Given an initial concentration for A equal to 0.1 M and an initial concentration of B equal to 0.2 M, what is the eq concentration of C? Is X negligible? A (aq) + B (aq) = C (g) + D (s)

Keq = [C]/[A][B] = 2.1X10^-7 = [x]/[0.1-x][0.2-x] x is negligible because keq is tiny 2.1X10^-7 = x/[0.1][0.2] x = 4.2X10^-9 (eq concentration of C and amount of A and B that have reacted

______________ molecules absorb __________ wavelengths

Larger molecules absorb larger wavelengths, and conjugation allows even larger wavelengths to be absorbed

Titration of Strong acid and weak base

Looks like inversion of titration of weak acid and strong base Gradual drop in pH with addition of strong acid Equivalence point (steepest part of the curve) in acidic range *Starts at high pH

SP2

Mixture of 2 P orbitals and one S orbital Three orbitals that are 33% s and 66% p and are all situated 120 deg from each other Examples: Alkenes (carbon carbon double bonds)

Molecules

Molecule: Combination of 2 or more atoms held together by COVALENT bonds -Ionic compounds form from combinations of elements with large electronegativity difference (dont form true molecules, hard to define, use formula units and formula weights instead), molecular compounds form from elements of similar electronegativity Molecules: Smallest units of compounds -Molecular weight: Sum of atomic weights of all atoms in molecule (units = atomic mass units (amu)/ mole) *Atomic weight is weighted average of masses of isotopes *The difference between mass and weight is that mass is the amount of matter in a material, while weight is a measure of how the force of gravity acts upon that mass. *Molecular weight of SOCl2 = 32.1 amu + 16 amu + 71 amu = 119.1 amu/molecule

Common polyatomic ions to know:

NH4+ = Ammonium (Charge: +1) C2H3O2- = Acetate (Charge: -1) CN- = Cyanide (Charge: -1) MnO4- = Permanganate (Charge: -1) SCN- = Thiocyanate (Charge: -1) CrO4^2- = Chromate (Charge: -2) Cr2O7^2- = Dichromate (Charge: -2) Bo3^3- = Borate (Charge: -3) Note: Oxyanions of transition metals, like MnO4- and CrO4^2- ions have really high oxidation number on the metal, tend to gain electrons in order to reduce oxidation number (make good oxidizing agents)

IR Peaks

OH group corresponds to broad peak around 3000-3500 cm^-1 1680-1750 cm^-1 is representative of a carbonyl C-C single bond: 1200- 1400 cm^-1 C-H: 2800-3000

Write the net ionic equation for: Mg (s) + AlCl3 (aq) -> Al (s) + MgCl2 (aq) *dont forget to balance equation

On the left: Mg = 0, Cl = -1, Al = +3 Right: Al = 0, Mg = +2, Cl = -1 Cl stays the same so we can ignore it Mg + Al^3+ -> Al + Mg^2+ Now balance charges Mg + 2Al^3+ -> Al + 2Mg^2+

What volume would 12 g of helium occupy at 27 deg C and 380 mmHg?

P = 380 mmHg * 1 atm/ 760 mmHg = 0.5 atm T = 300 K n = 12 g He / 4 g/mol = 3 mol He V= 3X0.0821X300 = 3 X 8 X 3 = about 144 L

Gay-Lussac's Law (Isovolumetric Heating)

P1/T1=P2/T2 or P/T = k Complementary to Charles law Derivative of ideal gas law Increase in temperature is directly proportional to increase in pressure

Combined Gas Law

P1V1/T1=P2V2/T2, Compares 2 states: STP and another state Assumes moles stays constant

Reduction always occurs at the ________ and oxidation always occurs at the __________

RED CAT: Reduction always occurs at the cathode AN OX: Oxidation occurs at anode

Neutralization Reactions

Specific type of double displacement reaction, in which an acid reacts with a base to produce a salt and usually water. Acid and base reactions not always visible (use pH indicator strips) Al(OH)3 + H3PO4 -> 3H2O + AlPO4

Base-Catalyzed Keto-Enol Tautomerization

Step 1: Formation of carbonation (step driven by the acidity of the proton alpha to the carbon)

Systems and Surroundings (thermodynamics)

System: Matter that is being observed (total amount of reactants and products in a chemical reaction)(Solute and solvent) Surroundings/environment: Everything outside of the system Boundary between system and surroundings not permanently fixed and changes depending on what is being studied Ice pack would be system (Using energy), person with ice on leg would be surroundings (providing heat) Coffee is system, cup is surroundings, or, coffee and cup is system and environment is surroundings Types of systems: Isolated: system cant exchange energy (heat and work) or matter with surroundings (insulated bomb calorimeter) Closed: System can exchange energy but not matter (steam radiator) Open: Can exchange energy and matter (pot of boiling water) When a system experiences a change in one or more property (reactants or products, temp, pressure) it undergoes process *Processes associated with change of state, but some are uniquely identified by constant property (simplify 1st law of thermodynamics)

Determine the new boiling point of a solution containing 190 g MgCl2 in 1800 g water at room temperature: (Kb=.512 k*kg/mol)

Tb = i Kb m = 3 * .512 * (2/1.8) = 2 K 190/ 95 = 2 So, new BP = 373 + 2 = 375 K

The only physical factor that can change the value of Keq is:

Temperature Ka, Kb, Ksp all = Keq

Electron Affinity

The amount of energy released when a gaseous atom or ion gains an electron Energy dissipated by a gaseous species when it gains electron Energy released to form an anion Exothermic -> Expels energy/heat Opposite of ionization energy (energy to remove electron from gaseous species) Increases from left to right and decreases from top to bottom (stronger pull between VE and nucleus = higher Zeff = greater energy release)(Valence shell further away as n increases, so it decreases going down) Halogens (group 17/ VIIA) are the most greedy (only needs one more = highest EA) -Noble gases have electron affinity of 0 Electron affinity is related to several factors, including atomic size and filling of the valence shell. As atomic radius increases, the distance between the nucleus and the outermost electrons increases, thereby decreasing the attractive forces between protons and electrons. As a result, increased atomic radius will lead to lower electron affinity. Because atoms are in a low-energy state when their outermost valence electron shell is filled, atoms needing only one or two electrons to complete this shell will have high electron affinities. If two atoms both need only one more electron to have a noble gas-like electron configuration, the smaller will have the highest electron affinity.

Root-Mean-Square Speed (u rms) What is average speed of xenon difluoride molecules at 20 deg C? What is average speed of helium atoms at -173 deg C?

The average speed of a gas molecule at a given temperature; As a scalar, does not take direction into account One way to define average speed by determining average kinetic energy per particle and then calculating speed to which it corresponds u rms = Sqr rt (3RT/M) where R is ideal gas constant, T is temp, M is molar mass R = 8.314 J/ K*mol (J is derived from kilograms) molar mass = 169.3 g/mol = about 0.17 kg/mol u rms = sqr rt (3 X 8.314 X 293)/(0.17) = sqr rt (8 X 300 / 0.06) = sqr rt (40000) = about 200 m/s u rms = sqr rt (3X8X100 / 0.004 kg/mol) = about 800 m/s

An aqueous solution was prepared by mixing 70 g of an unknown nondissociating solute into 100 g of water. The solution has a boiling point of 101.11°C. What is the molar mass of the solute? Kb = 0.512 K*kg/mol

The equation ΔTb = iKbm can be used to solve this problem. The change in boiling point is 101.11 - 100 = 1.11°C. The van 't Hoff factor for this solute is 1 because the molecule does not dissociate into smaller components. The mass used in this equation is 0.1 kg because 100 mL of water has a mass of 0.1 kg. Molality (m): moles of solute/kilograms of solvent So, 1 = 1 * 0.5 K*kg/mol * m m = 2 2= x/0.1 , so x = 0.2 moles 70 g / 0.2 moles = 350 g/mol

Water dissociation Constant (Kw)

The equilibrium constant of the water dissociation reaction at a given temperature, equal to 10^-14 at 25 deg C (298 K)

Heat of formation (ΔHf) and Heat of fusion (ΔHfus) and Heat of sublimation (ΔHsub) and Heat of vaporization (ΔHvap)

The heat absorbed or released during the formation of a pure substance from its elements at a constant pressure the enthalpy change for the conversion of 1 gram or 1 mole of a solid to liquid at constant temperature and pressure Heat of sublimation: The enthalpy change for the conversion of 1 gram or 1 mole of a solid to a gas at constant temperature and pressure Heat of vaporization: The enthalpy change for the conversion of 1 gram or 1 mole of a liquid to a gas at constant temperature and pressure

Kinetic and Thermodynamic Control of a Reaction

The kinetic pathway requires less free energy to reach the transition state, but results in a higher energy/less stable product Kinetic Product: Formed with lower temperatures/ smaller heat transfer (doesn't need as much heat)/lower free energy so its often called "fast" product/ low heat transfer Thermodynamic Product: Formed favorably at higher temp/larger heat transfer/lower free energy (but larger difference in free energy between the products and reactants than the kinetic product)/more stable/ more negative delta G/More spontaneous *Kinetic = smaller gain in free energy to reach transition state, higher free energy of products (with smaller difference in free energy between transition state and products)

Absolute Zero

The temperature at which all substances have no thermal energy 0 K or -273.15 deg C

Yield (Theoretical VS Actual) What is the percent yield for a reaction in which 27 g of Cu is produced by reacting 32.7 g of Zn in excess CuSO₄ solution?

Theoretical Yield: is the amount of product generated if all of the limiting reactant is consumed with no side reactions, assumes entire product has been collected (amount of product predicted) Actual Yield: Amount of product one actually obtains % yield = (actual yield / theoretical yield) x 100 balanced: Zn (s) + CuSO₄ (aq) --> Cu (s) + ZnSO₄ (aq)theoretical yield for Cu: 32.7 g Zn (1 mol Zn/65.4 g Zn) (1 mol Cu/1 mol Zn) (63.5g Cu/1 mol Cu) = 31.8 g Cu. (28/31.8) x 100 = 87.5%

Charles' Law (Isobaric Expansion)

V1/T1=V2/T2 Derivation of ideal gas law Volume and temp are directly proportional (when one increases, the other does too) When T = absolute 0, volume = 0 too

400 g AlCl3 is dissolved in 1.5 L of water at room temp (Kf = 1.86 K*kg/mol). How much does BP increase after adding albumin chloride

Water at room temp density = 1 g/mL 1.5 L = 1.5 kg Vant Hoff factor fo AlCl3 = 4 Molality: 400 g AlCl3 / 133.5 g per mole = 3 mol AlCl3/ 1.5 Kg = 2 m Kb = 0.512 K*kg/mol Delta Tb = I Kb m = 4* 0.512 K*kg/mol * 2m = 4K

Heating Curves

When compound heated, temp rises until melting/boiling point reached, then stays constant until next phase (once entire sample converted, temp rises again) Phase change reactions dont undergo changes in temp (delta T = 0) heat absorbed/ all used to to overcome intermolecular forces Solid-Liquid boundary: Use values based on enthalpy/heat of fusion ( Delta H of fus) to determine heat transfer during phase change -Solid to liquid will be positive because heat is added, change in enthalpy will be negative from liquid to solid (heat removed) Liquid gas boundary: Enthalpy/heat of vaporization q = mL where m = mass and L = latent heat (enthalpy of isothermal process in units of cal/g)

Jones Oxidation

an oxidation reaction in which primary alcohols are oxidized to carboxylic acids and secondary alcohols are oxidized to ketones; requires CrO3 dissolved with dilute sulfuric acid in acetone

Law of Constant Composition

any pure sample of a given compound will contain the same elements in an identical mass ratio (For every 1 gram of H there will be 8 grams of O in H2O)

Molecular Orbital: Sigma and Pi Bonds

describes the probability of finding the bonding electrons in a given space Obtained by combining wave functions of atomic orbitals (overlap describes molecular orbital) -Same sign = bonding -Different atomic orbital signs = anti bonding orbital forms (molecular orbital formed by overlap of two or more atomic orbitals; energy is greater than the energy of the combining atomic orbitals) Overlap head-to-ehad = sigma bond -Allows free rotation about axes Overlap to form two parallel electron densities = Pi bond -No free rotation/can't be twisted -Formed between 2 p orbitals -Second bond in a double bond and second and third bond in triple bond

Benzaldehyde

hydrogen cyanide inhibits cytochrome c oxidase -> blocks aerobic respiration and benzaldehyde -Vaporizes at room temp/ reaches olfactory system as gas low concentrations: pleasant aroma of almonds High concentrations: Noxious irritant

Adiabatic Process

no heat exchange between system and environment (Q=0) ΔU = -W (change in internal energy = work done ON system (opposite of work done BY system)) Hyperbolic graph

Mixed order reactions:

non-interger orders (fractions) or broken-order reactions. Reactions that change order over time Rate = (k1*[C][A]^2)/ (k2 + k3 [A]) Where A represents a single reactant and C represents catalyst Large A at beginning = k3[A]>>>>> k2 and the reaction will appear to be first order with respect to A At the end, when A is low, K2>>>>> K3[A], making reaction appear second order

Boiling Point Elevation

ΔTb=iKbm where Tb is increase in boiling point, I is Van't Hoff factor, Kb is proportionality constant, and m is molarity of solution Van't Hoff factor = number of particles into which compound dissociates in solution (I = 2 for NaCl, covalent molecules like glucose dont dissociate so I= 1) Calculates the amount that the normal BP is raised: value calculated is not boiling point itself *When nonvolatile solute dissolvent in solvent, BP of solution will be greater than pure solvent *BP - temp at which VP of liquid = ambient/incident pressure *Adding solute decreases vapor pressure (if VP of solution is lower than that of pure solvent, more energy/higher temp required before boiling)

What is the gram equivalent weight of phosphoric acid? A. 24.5 g B. 32.7 g C. 49.0 g D. 98.0 g

Correct Answer: B Explanation: Gram equivalent weight is the weight (in grams) that releases 1 acid or base equivalent from a compound. Because H3PO4 contains 3 protons, we find the gram equivalent weight by dividing the mass of one mole of the species by 3. The molar mass of phosphoric acid is 98 g/mol so the gram equivalent weight is 32.7 g.

Which of the following electronic transitions would result in the greatest gain in energy for a single hydrogen electron? A. An electron moves from n = 6 to n = 2. B. An electron moves from n = 2 to n = 6. C. An electron moves from n = 3 to n = 4. D. An electron moves from n = 4 to n = 3.

Correct Answer: B Explanation: For the electron to gain energy, it must absorb energy from photons to jump up to a higher energy level. There is a bigger jump between n = 2 and n = 6 than there is between n = 3 and n = 4.

Which of the following is true of equilibrium reactions? I. An increase in k1 results in a decrease in k-1. II. As the concentration of products increases, the concentrations of reactants decreases. III. The equilibrium constant is altered by changes in temperature. A. I only B. II and III only C. I and III only D. I, II, and III

Correct Answer: B Explanation: Statement I is false because the addition of a catalyst could increase the rate constants of both the forward and reverse reactions. Statement II is true because—for products to come into existence—reactants must be used up. Statement III is also true: all K values are temperature-dependent.

If Keq> 1 If Keq = 1 If Keq < 1 If Keq <<<<<< 1

Products are present in greater concentration at equilibrium Products and reactants are both present at equilibrium at reasonably similar levels Reactants present in greater concentration at Eq Amount of reactants that have been converted to products can be considered negligible in comparison to initial concentration of reactants *The concentration of a reactant that converts to product cane considered negligible if it is two or more orders of magnitude less than the initial concentration of the reactant

Protons, Neutrons, and Electrons

Protons: Found in nucleus Charge = to fundamental unit of charge = 1.6X10^-19 C 1 Proton = 1 Atomic mass unit (AMU) Atomic number (Z) = # of protons Neutrons: Mass about equal to proton but slightly larger Electrons 1/2000 size of proton Electrons closer to nucleus are at lower energy levels Farthest from nucleus (higher electrons shells) = highest energy = stronger interaction with surrounding environment -Positive charged = fewer electrons = cation

1/8

0.125

Van der Waals Equation of State

(P+n^2a/V^2)(V-nb)=nRT Used to correct the ideal gas law for intermolecular attractions (a) and molecular volume (b) a and b are physical constants. a corrects for attractive forces beween molecules (smaller for smaller gases and less polarizable (like helium, a n d larger for larger more polarizable gases like Xe or N2 and largest for polar molecules like HCl and NH3) The b corrects for volume of molecules (larger molecules = larger b) a is the van der Waals term for attractive forces, b is term for big particles *If a and b are 0, it becomes ideal gas law Attempts to correct for the deviations from ideality that occur when gas does not closely follow ideal gas law Under same, non-ideal conditions, if a is increased while b remains negligible, n^2*a/V^2 gets larger and pressure drops to compensate, so methane will behave more ideally than chloromethane because a is smaller for methane Real pressure of methane will thus be higher (closer to ideal) Chloromethane = larger a = less pressure Smaller a = closer to ideal If methane and isobutane are placed in same size container under same conditions, (both with negligible attractive forces), isobutane is larger - larger correction term for size of molecule, b. This makes V-nb smaller Pressure or volume must rise to compensate Isobutane exerts higher pressure

Concentration Cells

(Special type of galvanic cells. When E°cell is 0 and ΔG° =0 Contains 2 half-cells connected by conductive material (Allows for spontaneous reaction) Distinguishing characteristic: Electrodes are chemically identical (same reduction potential), so current generated as function of concentration gradient established between two solutions surrounding electrodes (drives movement of electrons in direction of equilibrium ) *Current stops when concentrations of ions in half-cells are equal (implies that voltage (V) or electromotive force = 0 when concentrations are equal) *Concentration cells can be spontaneous; however, if the concentration cell had reached equilibrium, it would cease to be a spontaneous reaction Voltage as function of concentrations: Can use the Nernst equation: Ecell=E°cell-.0592/n*logQ (at 298K) or Ecell=E°cell-RT/nF*LnQ *Cell membrane of neuron (different ions exchange to produce electrical potential) -Resting membrane potential (Vm) can be maintained -> large disturbance stimulate firing action potential

Electrode Charge Designations

*Galvanic cell* anode: (-), oxidation (anode is negative because it is the source of electrons) cathode: (+), reduction *Electrons move from negative to positive, current goes from positive to negative *Current is the flow of POSITIVE charge *Electrolytic cell* anode: (+), oxidation (positive because anode is attached to positive pole of external voltage source) cathode: (-), reduction (attached to negative pole) Differences between two cells because an external source is used to reverse charge of electrolytic cell. *However, both types have reduction occur at cathode and oxidation at anode, and both have Anode attracts anions, cathode attracts cations (always). *Electrons always flow from anode to cathode and current flows from cathode to anode *Isoelectric focusing = technique to separate amino acids or polypeptides based on isoelectric points (pI) *positively charged amino acid goes towards cathode, negative charged migrate towards anode

Cell Potentials

*Note: for galvanic cells, direction of spontaneous movement of charge is from the anode (site of oxidation) to cathode (site of reduction) -> the tendency of different species to be reduced is determined using tendency of H+ ion to be reduced to arbitrary zero reference point Reduction potentials: Tendency of species to gain electrons *More positive the potential = greater tendency to be reduced = more likely reduction occurs *Measured in volts (V) and defined relative to standard hydrogen electrode (SHE)(given potential of 0V) *Standard reduction potential (E red) measured by setting up a cell relative to a standard hydrogen electrode, which is given reductio potential of 0 V, measured under standard conditions (25 deg C, 298K, 1 atm, 1M concentration) *For galvanic cells, more positive reduction potential is the cathode, less positive is anode -Species with stronger tendency to gain electrons (cathode wants to gain electrons more) is actually doing so, so the reaction is spontaneous (Delta G is negative) *For electrolytic cells, more positive reduction potential is forced by external voltage to be oxidized, and is therefore anode. The electrode with the less positive reduction potential is forced to be reduced and is the cathode. Movement of electrons against desires, so the reaction is non-spontaneous and delta G is positive

Combustion Reaction

-involves a fuel (usually hydrocarbon) and an oxidant (usually oxygen) -Oxidizing (usually with O2) a fuel -typically a hydrocarbon combines with oxygen to yield carbon dioxide and water -Example: glycolytic pathway -The larger the alkane reactant, the more numerous the combustion product CH4 (g) + 2O2 (g) -> CO2 (g) + 2H2O (l) Left: C = -4, H+1, O = 0 Right: C = +4, O=-2, H=+1, O=-2 Half reactions: CH4 + 2H2O -> CO2 + 8H+ + 8e- 2O2 +8H+ +8e- -> 4H2O (there were 4 O on left so you had to make 4 O on right by multiplying by 2, so you had +1*8 charge on H+ and -8 on O, so add 8e- and 8H+ to left side) Net ionic equation: CH4 + 2O2 -> CO2 + 2H2O Carbon is reducing agent Oxygen in oxidizing agent (reduced from O to -2, gaining electrons)

pH

-log[H^+] Example: -log 10^-6 = 6

Reaction Quotient (Qc)

-measures concentrations of reactants and products at any point in time during a reaction Qc = [C]^c * [D]^d / [A]^a * [B]^b -looks identical to the equation for Keq, but for Qc the concentrations are not constant -Q<Keq, then forward reaction has not established equilibrium -Great concentration of reactants/smaller concentration of products then at equilibrium -Reaction proceeds in forward direction/ forward rate of reaction is increased -Delta G< 0 If Q=Keq, reaction is in dynamic equilibrium -Forward and reverse reaction rates are equal -Delta G = 0 -Once reaction is at equilibrium, any movement (forward or reverse) will be non-spontenaoeus If Q>Keq, then forward reactions exceeded equilibrium -Greater concentration of products, fewer reactants -Reverse rate is increased to restore eq -Delta G>0, reaction proceeds in reverse direction

A sample is assayed for lead by a redox reaction with I3- (aq). A 10g sample is crushed, dissolved in sulfuric acid, and passed over a reducing agent so that all the lead is in the form Pb2+. The Pb2+ (aq) is completely oxidized to Pb4+ by 32.60 mL of a .7M solution of NaI3. The balanced equation for the reaction is: I3- (aq) + Pb2+(aq) --> Pb4+(aq) + 3I- (aq) Calculate the mass of lead in the sample.

0.7 M I3^- (32.6X10^-3L)(1 mol Pb2+/1 mol I3-)(207.2 g Pb2+/ 1 mol Pb2+) = 0.7 X 3X 2 = 4.2 grams You don't actually need the 10.0 gram value

7/8

0.875

Nomenclature of Ionic Compounds

1) elements that form 1+ positive ion, charge is indicated by Roman numerals following element 2) -ous or -ic to ion with lesser or greater charge e.g. Fe2+=ferrous Fe3+=ferric 3) monatomic anions end in -ide e.g. H=hydride F=fluoride Phosphide: P^3- 4) Oxyanions (polyatomic anions that contain oxygen); When an element form two oxyanions, less O=ite and more O=ate e.g. NO2^-=nitrite NO3^-=nitrate *lITE = less, ATE more O 5) In extended series of oxyanions: Hypo- and Per- are used to indicate less and more O2 e.g. ClO=hypochlorite ClO2=chlorite ClO3=chlorate ClO4=perchlorate 6) Polyatomic anions often gain 1 or more H+ to form anions of lower charge : Add hydrogen/dihydrogen or bi- e.g. HCO3^-= hydrogen carbonate or bicarbonate H2PO4^- = Dihydrogen Phosphate Phosphate = PO4^3- Chrommium (II): Cation with formula Cr^2+

Factors affecting Reaction Rates

1. Reaction Concentration: Greater concentration of reactants = greater number of effective collisions per unit time (increase in frequency factor (A)) (Reaction rate will increase for all but 0 order reactions) 2. Temperature: Usually reaction rate increases as temp increases (increases average kinetic energy of molecules) -All reactions have optimal temperature (if temp gets too high, catalyst may denature and rate drops) 3. Medium: Aqueous vs non aqueous environment, physical state of medium (solid, liquid, gas) -Polar solvents preferred becuase molecular dipole tends to polarize bonds of reactants, lengthening and weakening them, letting reaction go faster 4. Catalysts: Increase reaction rate without being consumed in reaction -Interact with reactants through absorption/ intermediate formation, and stabilize them to reduce Ea -Incldues all enzymes (chemically interact with reactants, but return to original state) -May increase collision frequency, change orientation to make collisions more effective, donate electron density, or reduce intramolecular bonding *By definition, a catalyst increases the rate of a reaction by lowering the activation energy, making it easier for both the forward and reverse reactions to overcome this energy barrier. Catalysts are neither used up in the reaction, nor do they alter the equilibrium of a reaction -Homogeneous catalysis: Catalyst in same phase as reactants (Solid, liquid, gas) -Heterogeneous Catalysis: Catalsyst in distinct phase -Decrease Ea for raactants and products, no impact on free energy -Only changes rate of reactions (forward and reverse changed by same factor), no impact on equilibrium position or measurement of Keq -Can't transform non-spontaneous to spontaneous, can only make spontaneous faster *rirum is dynamic, so they undergo change but net change will be 0 Note: The faster a reaction can reach its activation energy, the faster it will proceed to completion. If all conditions are equal, the reaction with the lowest activation energy will have the fastest rate, regardless of endergonic or exergonic

If [H3O+] is8.89 X 10-4 M , what is pH, pOH, and OH-?

10^-14 / (8.89 X 10-4 M) = 1.12 X 10^-11 M (this is OH- concentration) To find pH: 8.89 X 10-4 M = 4 - log (8.89) = about 4 - 0.889 = about 3.1 = pH pOH = 14 - 3.1 = about 10.9 *Double check: 1.12 X 10^-11 11 - 0.112 = about 10.9

Be(OH)2 is produced when water reacts with BeO. If one starts with 2.5 Kg BeO in excess water, and produces 1.1 kg Be(OH)2, what is the percent yield of this reaction?

2500 g BeO X (1 mol BeO/25 g BeO)(1 mol Be(OH)2/ 1 mol BeO)(43 g Be (OH)2/ 1 mol Be(OH)2) = 4300 g Be (OH)2 Percent yield = 1100/4300 g X 100 = 11/44 = about 25%

In acid base chemistry, the gram equivalent weight represents the mass of acid that yields one mole of protons or the mass of base that yields one mole of hydroxide ions. What is the gram equivalent weight (GEW) of sulfuric acid?

2X1 g/molH + 1X32.1 g/mol S + 4X16 g/mol O = 98.1 g/mol 98.1/ (2 mol H+/ mol H2SO4) = 49.05 g/mol H+

Normality (N) What is the normality of a 2M Mg(OH)2 solution?

4 Measure of concentration, in equivalents/ I Usually for hydrogen ions, where 1N solution = 1 mole per liter Molarity = Normality / n (number of particles per molecule) So, 1N solution of HCL = 1M and 1 N solution of H2CO3 = 0.5 M Benefit of equivalents and normality: Allows direct comparison

In the process of photosynthesis, carbon dioxide and water combine with energy to form glucose and oxygen, according to the following equation: CO2 + H2O -> C6H12O6 + O2 What is the theoretical yield of glucose if 30 grams of water are reacted with excess carbon dioxide and energy, according to the equation above? (Note: equation not balanced)

50 g Balanced equation: 6CO2 + 6H2O -> C6H12O2 + 6O2 The theoretical yield is the amount of product synthesized if the limiting reagent is completely used up. This question therefore asks how much glucose is produced if the limiting reagent is 30 grams of water. We can use the three-fraction method discussed in this chapter to solve for the mass of glucose produced: 30g H2O X (1mol H2O/ 18 g H2O)(1 mol C6H12O6/6 mol H2O)(180 g C6H12O6/1 mol C6H12O6) = 50 Thus, 50 grams of glucose are produced.

Titration

A procedure used to determine the concentration of a known reactant in a solution Different Types; Acid base, oxidation reduction, and complexometric (metal ion) Performed by adding small volumes of solution of known concentration (titrant) to solution of unknown concentration (titrand) until completion of reaction is achieved at equivalence point Acid-Base equivalence points *Reached when number of acid equivalents present in original solution = number of base equivalents added (and vice Versa) (steepest part of the curve) *Lets us calculate unknown concentration of titrand: NaVa = NbVb where Na and Nb are acid and base normalities, and Va and Vb are volumes of acids and base solutions *at any given pH, only 2 forms of acid exist in solution, so each conjugate is titrated separately *strong acid/base will have equivalence point at pH 7 *Titrating polyphonic = multiple equivalence points (each species titrated separately) *Equivalence point can be determined by graphical method (PH meter) or by watching color change of indicator Indicators are weak organic acids or bases with different colors in protonated and deprotonated states: (indicator must be weaker then what is being titrated or the indicator would be titrated first) Indicators change color as they shift between conjugate acid and base (H indicator (color 1) <-> H+ + indicator (color 2)) : this is equilibrium process, apply Le Chatelier's principle: Adding H+ shifts eq to left, adding OH- removes H+ and shifts eq to right *Point at which indicator changes to final color is endpoint *volume difference between endpoint and equivalence point is negligible *Endpoint is pH where indicator turns its final color *Selection of ideal indicator: know the pH and determine equivalence point, then pick indicator that has closest pKa Middle of flat part tells you pKa (because pKa = pH) Middle of steep part tells you equivalence

Titration of Weak acid and strong base

A strong base, like NaOH, titrated into solution of weak acid, like CH3COOH yields equivalence point > 7 *Weak acids do not dissociate to the same degree, so H3O+ concentration will be lower and pH will be higher *Less steep/sudden than strong acid and strong base *Starts at low pH

How many grams of calcium chloride are needed to prepare 71.7 grams of silver chloride according to: CaCl2 (aq) + 2 AgNO3 (aq) -> Ca(NO3)2 (aq) + 2AgCl (s)

About 27.8 1 mole of CaCl2 reacted with 2 moles of AgNO3 -> 2 moles of AgCl Molar mass of CaCl2 is 111.1g Molar mass of AgCl = 143.4g 71.7 g AgCl X (1 mol AgCl/143.4 AgCl)(1 mol CaCl2/2 mol AgCl)(111.1 g CaCl2/ 1 mol CaCl2)

Raoult's Law

Accounts for vapor pressure depression caused by solutes in solution. As solute is added to solvent, the vapor pressure of the solvent decreases proportionately. *if compound (solvent) A has higher vapor pressure than compound (solute) B, as the concentration of B increases, the vapor pressure of A decreases. As more solute is dissolved into solvent (more B dissolved into A), vapor pressure of solvent decreases *As VP decreases, temp/energy required to boil increases Presence of solute molecules can block evaporation of solvent molecules, nut not their condensation (Condensation rate unaffected)(net reduction in vapor pressure)(reduces vapor pressure of solution compared to pure solvent) *Equation shown in image, where PA is vapor pressure of solvent A when solutes are present, XA is mole fraction of solvent A in solution, and P0A is vapor pressure of solvent A in pure state Only holds true when attraction between molecules of different components = attraction between any 1 components in pure state (obeying Raoult law = ideal solution) *Benzene and toluene are both organic liquids and have very similar properties. They are both nonpolar and are almost exactly the same size. Raoult's law states that ideal solution behavior is observed when solute-solute, solvent-solvent, and solute-solvent interactions are all very similar. Therefore, benzene and toluene in solution will be predicted to behave as a nearly ideal solution.

Salt Formation

Acid and bases maybe act with each other, forming a salt and often but not always water in a neutralization reaction HA (aq) + BOH (aq) -> BA (s) + H2O (l) Usually go to completion reverse reaction in which salt ions react with water to give back acid or base is known as hydrolysis Four combinations of strong and weak acids and bases: Strong acid+ strong base: HCl + NaOH -> NaCl + H2O (pH of 7, equimolar amounts of water and salt) Strong acid + weak base: HCl + NH3 -> NH4Cl (product is salt, but often no water formed, can re-from weak base with hydrolysis: NH4+ + H2O -> NH3 + H3O+) *pH below 7 Weak acid + strong base: HClO +NaOH -> NaClO + H2O *pH in basic range (above 7) *Increase in hydroxide ion concentration (in reaction 2/reverse reaction) shifts system away from autoionziation, reducing concentration of hydronium ion Weak Acid + weak base: HClO + NH3 -> NH4ClO *pH depends on relative strength of reactants (If Ka is less than Kb, the solution is basic, and at equilibrium the concentration of hydroxide ions will be greater than concentration of hydronium ions

Polyvalence and Normality

Acid equivalent = 1 mole of H3O+ ions Base equivalent = 1 mole of OH- ions Polyvalent: each mole of acid or base liberates more than 1 acid or base equivalent (can also be called polyprotic) Looking at the equations in image: 1 mole of H2SO4 produces 2 acid equivalents ( 2 moles of H3O+) First dissociation goes to completion, but second reaches equilibrium *acidity of solution depends on concentration of acidic equivalents that can be liberated Normality: Directly indicates quantity of acidic or basic capacity (for example, 1 mole of H3PO4 yields 3 moles of H3O+, so a 2M H3PO4 solution would be 6N) Gram equivalent weight: mass of compound that produces 1 equivalent weight (1 mole of charge) *Example: H2SO4 (molar mass 98 g/mol) is divalent acid, so each mole yields 2 acid equivalents, so the gram equivalent weight = 49 grams *Common polyvalent acids: H2SO4, H3PO4, H2CO3 Common polyvalent bases: Al(OH)3, Ca(OH)2, Mg(OH)2 Normality of 2M Al (OH)3 = 2X3 = 6N

Lewis acids and Bases

Acids are electron pair acceptors Bases are electron pair donors (gives electrons to acid) Electron pair is lone pair and not involved in any other bonds Most inclusive: Every Arrhenius acid is also BL acid, and every BL acid is Lewis acid (same for bases), but the reverse isn't always true (like BF3 and AlCl3 can accept electrons so they are acid, but they can't donate H so they aren't BL or Arrhenius) *Often used as catalyst BL definition revolves around protons, Lewis revolves around electrons (this difference can be seen looking at arrows, in BL the arrow goes from the acid and in lewis, the arrow goes from the base) *Also called coordinate covalent bond formation or nucleophile-electrohpile interactions

Octet Rule and Exceptions

Always abide to the octet rule: Carbon, Nitrogen, Oxygen, Fluorine, sodium, and magnesium Incomplete octet exceptions: Stable with fewer than 8 electrons H (2 e-), He(2 e-), Li(2 e-), Be(4 e-), B(6 e-). Expanded octet: Any element in period three and greater can hold more than 8 electrons (phosphorus =10, sulfur = 12, chlorine = 14) Odd numbers of electrons: Any molecule with an odd number of valence electrons cant distribute those electrons to give 8 to each atom (Nitric oxide (NO) has 11 VE)

Acid-Base Behavior of Water

Aphoteric: when it reacts with base it behaves like acid, but when it reacts with acid it behaves like a base *Can react with itself in a process called autoionization *H2O (l) + H2O (l) <-> H3O+ (aq) + OH- (aq) One molecule donates hydrogen ion to produce hydronium ion (H3O+ or H+) and the other hydroxide ion *H+ is misleading, because proton is never isolated insulation, it is always attached to water or something *Reversible reaction, at equilibrium *At 298 K, the water dissociation constant (Kw) = [H3O+][OH-] = 10^-14 *Hydrogen ions and hydroxide ions always equal at equilibrium (concentration of each is 10^-7 when solution is neutral) *But even when not neutral, product of concentrations always equals 10^-14 at 298K *If something donates H ions to pure water, concentration will increase, shifting system toward reactants, decreasing hydroxide ion concentration to return to equilibrium *Kw is dependent only on TEMPERATURE (Like other eq constants, not effected by changes in concentration, pressure or volume) *At temperates above 298K, Kw will increase (direct result of endothermic nature of autoionziation reaction

Summary of Arrhenius vs Bronsted Lowry vs Lewis

Arrhenius acid dissociates to form excess H+ in solution, base dissociates to form excess OH- BL acid is H+ donor and base is H+ acceptor (arrow drawn from acid to base): F- can be base because it can accept a proton, but BL acid have to have Hydrogen / proton to donate Lewis is election pair acceptor and base is electron pair donor (Arrow drawn from base to acid)

Assuming standard conditions; If Keq<1 (Keq = 1.2 X 10^-2), then ΔG° is _, the reaction is ____________ (spontaneous/nonspontaneous), and the E°cell is _ If Keq>1, then ΔG° is _, the reaction is ____________ (spontaneous/nonspontaneous), and the E°cell is _ If Keq=1, then ΔG° is _, the reaction is ____________ (spontaneous/nonspontaneous), and the E°cell is _ NOT assuming standard conditions (so Q must be calculated, and then you end up doing Q/Keq) If Q<Keq (Q=10^-3 and Keq = 10^-2), the reaction goes _________ and E°cell is _ If Q>Keq, the reaction goes _________ and E°cell is _ Q=Keq, the reaction goes _________ and E°cell is _

Assuming standard conditions; If Keq<1 (Keq = 1.2 X 10^-2)(ratio of products greater than reactants), then ΔG° is +, the reaction is non-spontaneous, and the E°cell is - If Keq>1, then ΔG° is -, the reaction is spontaneous, and the E°cell is + If Keq=1, then ΔG° is 0, the reaction is at equilibrium and the E°cell is 0 NOT assuming standard conditions (so Q must be calculated, and then you end up doing Q/Keq) If Q/Keq greater than 1, (Q> Keq) then free energy will be positive (nonspontaneous) SO: If Q<Keq (Q=10^-3 and Keq = 10^-2), the reaction goes forwards and E°cell is + If Q>Keq, the reaction goes backwards and E°cell is - Q=Keq, the reaction is at equilibrium and E°cell is 0

Kinetic Molecular Theory of Gases (in reference to ideal gases)

Assumptions: 1) gases are made of particles with negligible volumes 2) Gas atoms or molecules have no intermolecular forces (no attractions or repulsions) 3) continous, random motion, collisions 4) collisions are elastic (conservation of momentum and kinetic energy) 5) average KE is proportional to absolute temperature, and is the same for all gases at given temp The average kinetic energy of a gas particle is proportional to the absolute temperature of the gas KE = (3/2) kB X T where kB is Boltzmann constant (1.38 X 10^-23 J/K) KE = (1/2) mv^2 where m = mass and v = speed Speeds of gases are defined in terms of average molecular speed *higher temp = faster molting molecules *Larger molecules = slower Maxwell-Boltzmann distribution curve shows distribution of speed of gas particles at given temp. (as temperature increases, average speed increases and the distribution becomes wider and flatter

What is density of CO2 gas at 2 atm and 273 deg C?

At STP, mole of gas occupies 22.4 L -> Double in pressure (from 1 atm at STP to 2 atm here) = decreases volume proportionally = 1 atm/ 2 atm = 0.5 Temp increase proportional to volume increase, so X2 (546 K/ 273 K ) = 2 P1V1/T1=P2V2/T2 V2 = 22.4 L/mol X 0.5 X 2 = 22.4 L/mol density (p) = mass/volume = 44 g/mol / 22.4 L/mol = about 2 g/L

Bond Length/Strength

Average distance between 2 nuclei of atoms in a bond Inverse relationship between bond length and strength: C-C is longest bond (single bond), and the weakest Double bond between carbons is medium Triple bond between carbons has the shortest length and is the strongest Bond energy is energy required to break bond by separating Greater bond energy = stronger bond More shared electron pairs comprising single bond = higher energy of bond

Avogadro's Number (NA)

Avogadros number: number of representative particles (atoms, ions, molecules) in a mole, 6.02 X 10^23 Atomic weight of C is 12.0 amu, so average carbon has mass of 12 amu (carbon -12 far more abundant than carbon-13) and 6.02 X 10^23 carbon atoms have a combined mass of 12 grams

The Ksp of Ba(OH)2 is 5.0x10^-3. Assuming that barium hydroxide is the only salt added to form a solution, calculate the ion product of the following solutions based on the concentration of Ba2+. Then, predict the behavior of the given solutions (dissolution, equilibrium, or precipitation) [Ba2+] a) 0.5M b) 0.1M c) 0.05M

Ba(OH)2 -> Ba2+ + 2OH- For every x of Ba(OH)2 that dissolves, x of Ba2+ and 2x of OH- will be produced Ksp = [Ba2+][OH-]^2 a) 0.5M: Ion product = (0.5 M)(1 M)^2 = 0.5 (0.5 > 5.0x10^-3 = precipitation) b) 0.1M: Ion product = (0.1 M)(0.2 M)^2 = 4 X 10^-3 (4 X 10^-3 < 5.0x10^-3 = dissolution) c) 0.05M: Ion product = (0.05 M)(0.1 M)^2 = 5 X 10^-4 ( < 5.0x10^-3 = dissolution)

Calculate the Ksp of Ni(OH)2 in water, given that it's molar solubility is 5.2x10^-6 M

Balanced equation: Ni(OH)2 -> [Ni 2+] + [2 OH-] Ksp = x(2x)^2 Ksp = 5.2x10^-6 M * 2(5.2x10^-6 M)^2 Ksp = 5.2x10^-6 M * (10*10^-6)^2 Ksp = (5X10^-6)(10^-5)^2 Ksp = 5X10^-6 X 10^-10 = about 5X10^-16

Rate Law

Because reactants are being used up, we designate them with a negative sign and products with positive 2A + B -> C where rate of reaction with respect to A is -deltaA/ a (stoichiometric coefficient) delta T *Rate of consumption of A is twice the rate of consumption of B and twice the rate of production of C Rate is expressed in mol/ L*s or molarity/seconds For general equations, aA + bB -> cC + dD, the rate is proportional to rate = k [A]^x[B]^y where k is reaction rate coefficient/rate constant and exponents are orders of the reaction NOTE: ON THE MCAT, THE VALUES OF X AND Y ARE ALMOST NEVER THE SAME AS THE STOICHIOMETRIC COEFFICIENTS *Might need to use Law of Mass action to derive intermediate concentration *Exponents not equal to stoichiometric coefficients, unless reaction occurs via single step. *Product concentrations NEVER appear in rate law, don't confuse rate law with equilibrium constant Rate Order: Add up exponents: If rate = 2* [A]^3 [C], rate order is 4 (3+1)

Intermolecular Forces

Bonding forces that keep a substance together in solid or liquid state and determine whether two substances are miscible or immiscible in solution Can impact melting and BP Weakest = London Dispersion forces Next weakest = Dipole dipole interactions Strongest: Hydrogen bonds (no actual sharing/transfer of electrons) H bonds are 10% of strength of the STRONGEST: Covalent bonds (INTRAmolecular force, along with ionic)

Oxidation number vs formal charge

Both account for perceived charge, but: Oxidation number assumes unequal division of electrons in bonds (more electrons on more electronegative) and formal charge assumes equal division Reality is somewhere in between

How does the transition state theory compare with the collision theory of chemical kinetics

Both theories require activation energy to be overcome in order for reaction to occur Transition state theory focuses on forming a high energy activated complex that can then go forward or backward, forming products or reverting back to reactants The collision theory focuses on energy and orientation of reactants, and considers each potential reaction "all or nothing" (either enough energy to form products or not) -Rate of reaction is proportional to number of collisions per second

Boyle's Law (derivation of ideal gas law) 100 mL of an ideal gas is placed in a sealed container at a pressure of 300 mmHg. If the pressure is increased to 450 mmHg, what is the final volume of the gas?

Boyle's Law: Pressure and volume are inversely related (when one increases, other decreases) P1 * V1 = P2 * V2 or PV = k (k is a constant) X = 67 mL

Buffers

Buffer solution: Mixture of weak acid and salt (conjugate acid and cation) or a mixture of weak base and salt (conjugate acid and anion) *useful for resisting changes in pH when small amounts of acid/base are added Common buffer solutions: Acetic acid (CH3COOH) and its salt (sodium acetate: CH3COO- Na+) *acid buffer: serves to neutralize strong base Ammonia (NH3) and its salt (ammonium chloride: NH4+ Cl-) *base buffer *What would happen if concentrations of both acid and conjugate base were doubled? pH would not change, but buffering capacity (ability to which the system can resist changes in pH) will double. *Addition of small amount of acid or base will cause even less deviation in pH *Buffering capacity usually maintained within 1 pH unit of pKa value

Which of the following structure(s) contribute most to NO2's resonance hybrid? Draw three resonance structures of NO2 A) Negative charge on first O and +1 on N (double bond to O on right) B) Double bond to O on left, +1 on N, -1 on O on right C) -1 on each O, +2 on central N A. I only B. III only C. I and II only D. I, II, and III

C The two greatest contributors are structures I and II. Resonance structures are representations of how charges are shared across a molecule. In reality, the charge distribution is a weighted average of contributing resonance structures. The most stable resonance structures are those that minimize charge on the atoms in the molecule; the more stable the structure, the more it will contribute to the overall charge distribution in the molecule. Structures I and II minimize formal charges, so will be the largest contributors to the resonance hybrid.

Which of the following carboxylic acids will be the most acidic CCL3CH2COOH CHF2CH2COOH CH3CFClCOOH CH3CF2COOH

CH3CF2COOH Flourine has stronger inductive effects than chlorine (I<Br<Cl<F) Halogen closer to COOH = more acidic Relating to acids and bases: Acids that have electronegative elements nearer to acidic hydrogens are stronger

Calculate the concentration of H30+ in 2.0 M aqueous solution of acetic acid, CH3COOH (Ka = 1.8 X 10^5)

CH3COOH (aq) + H2O (l) <-> H3O+ + CH3COO- (aq) Ka = [H3O+][CH3COO-] / [CH3COOH] 1.8 X 10^5 = [H3O+][CH3COO-] / [CH3COOH] Weak acid means concentration of CH3COOH at eq = initial concentration (2.0M) - amount dissociated (x) [H3O+] = [CH3COO-] = x because each molecule of CH3COOH dissociates into one ion of each Ka = 1.8 X 10^5 = [x][x] / [2-x] x is usually really small so 2-x is about = 2 M So, Ka = x^2 / 2.0 M = 1.8 X 10^-5 Sqr rt of 3.6 X 10^-5 = 36X10^-6 = x = 6 * 10^-3 M x = concentration of H3O+ Note: X is a lot less than 2M Note: Easiest way to square root is to make exponent even number, then just divide by 2 x is negligible if it is < 5% initial concentration (Ka should be at least 100X smaller than initial concentration)

Bicarbonate Buffer System

CO2 (g)+ H2O(l) ↔ H2CO3 (aq) ↔ H+ (aq)+HCO3- (aq) *Carbonic acid (H2CO3) and conjugate base (HCO3^-) form a weak acid buffer for mainitinaing the pH of blood *CO2 reacts with water in blood to form carbonic acid, which can break down into bicarbonate and H+ *CO2 is waste product of cellular respiration (low solubility) and is exhaled from lungs *excess H+ (acidosis) causes breathing rate to increase to compensate so you can blow off more CO2 -Blowing off CO2 (hyperventilation) causes reaction to shift left consuming H+ and reducing H+ in the blood making pH less acidic -has pKa = 6.37, maintains pH around 7.4, which is slightly outside optimal buffering capacity (this is why academia is more common than alkalemia) (pH needs to be between 7.35-7.45) *Lots of conditions affect pH balance, like COPD, renal tubular acidosis, ketoacidosis, metabolic diseases, lactic acidosis, hyperventilation, poisonings

A vessel contains 8 mol of O2, 3 mol of CH4, and 1 mol of CO2 at total pressure of 240 atm. What is partial pressure of each gas

Calculate mole fraction: 8 mol/12.00 mol = 0.67 3 mol / 12.00 mol = 0.25 4 mol/12.00 mol = 0.083 Then calculate partial pressure by multiplying each by 240 atm (total pressure) PA = XA * PT (Dalton's law) P of O2 = 160 atm P of CH4 = 60 atm P of CO2 = 20 atm

If enough water is added to 11g of CaCl2 to make 100 mL of solution, what is molarity of solution?

Calculate number of moles of CaCl2: 11g CaCl2 * (1 mol/111.1 grams) = 0.1 mol Molarity = moles of solute / liters of solution 0.1 mol / 0.1 L = 1 M

Given that [product] = 0.075 M and [reactant] = 1.5 M, determine the direction of reaction and the sign of the free energy change for reactions with the following Keq values; 5.0 x 10^-2 5.0 x 10^-3 5.0 x 10^-1

Calculate value of Q: 0.075/1.5 = 7.5X10^-2/1.5 = 5X10^-2 Compare Q to Keq 5.0 x 10^-2 = 5.0 x 10^-2: At equilibrium, no net reaction (delta G = 0) 5.0 x 10^-3 (Keq) < 5.0 x 10^-2 (Q): Proceeds toward reactants (left)(Delta G is positive) 5.0 x 10^-1 > Q: (proceeds toward products (right)(delta G is negative)

Molar Mass What is the molar mass of 22.4 L sample of gas with mass = 225 g at 273 deg C and 10 atm? (Note, 273 C not 273K)

Calculated as the product of the gas's density at STP and STP volume of one mole of gas (22.4 L/mol) M = density at STP X 22.4 L/mol P1V1/T1 = P2V2/T2 V2 = V1 (P1/P2)(T2/T1) So we have V1, T1, P1, and V2, T2 and P2 are at STP so, V at STP = 22.4 L (10 atm/ 1 atm)(273 K/ 546 K) = 112L Density = mass over volume, so 225/ 112 L = 2 g/L at STP Then, molar mass = 2 g/L X (22.4 L/mol) = 44.8 g/mol The molar mass is the mass of a given chemical element or chemical compound (g) divided by the amount of substance (mol). The molar mass of a compound can be calculated by adding the standard atomic masses (in g/mol) of the constituent atoms. Molar mass serves as a bridge between the mass of a material and the number of moles since it is not possible to measure the number of moles directly.

Rechargeable Cells (Rechargeable Batteries)

Can function as both a galvanic and electrolytic cell Lead-Acid Batteries (lead storage battery): Voltaic cell, when fully charged consists of 2 half-cells (Pb anode and porous PbO2 cathode connected by conductive concentrated 4 M H2SO4) -When fully discharged, it has 2 PbSO4 electroplated lead electrodes with dilute concentration of H2SO4 (both half reactions cause electrodes to be coated with lead sulfate and dilute the acid electrolyte when discharging) Net equation for discharging lead-acid battery Pb (s) + PbO2 (s) + 2H2SO4 (aq) -> 2 PbSO4 (s) + 2H2O Cell = 1.685 - (-0.356) =2.041 V Cathode - anode When charging, the cell is electrolytic (needs energy input) (opposite equation) Lead acid batteries have lowest energy to weight ratio (energy density) *Energy density is measure of battery ability to produce power as function of its weight *lead acid batteries require heavier amount of material to produce certain output *A battery with a large energy density can produce a large amount of energy with a small amount of material Nickel-Cadmium Batteries: *Rechargeable *2 half cells: solid cadmium (anode) and nickel (III) oxide-hydroxide (cathode) connected by KOH (salt bridge?) *AA and AAA cells *Oxidation half-reaction at cadmium (negative) anode: Cd (s) + 2 OH- (aq) -> Cd(OH)2 (s) + 2e- Ered = -0.86V *Reduction half-reaction at nickel(III) oxide-hydroxide (positive) cathode: 2NiO(OH) (s) + 2H2O + 2e- -> 2NI(OH)2 (s) + 2OH- Ered = 0.49 V (positive value for cathode reduction, negative value for anode reduction) Net equation: 2NiO(OH) (s) + Cd+ 2H2O -> 2NI(OH)2 (s) + Cd(OH)2 (s) Ecell = 0.49 - - 0.86 = 1.35 V *Charging reverses electrolytic cell potentials *higher energy density than lead-acid batteries *Higher surge currents (periods of large current) -Surge current: An above average current transiently released at the beginning of the discharge phase of a battery *useful in remote controls *Largely replaced by NiMH batteries (nickel-metal hydride)

Find the emf of a galvanic cell at 25 degC based on standard reduction potentials: Fe2+ + 2e- -> Fe E°red = -.44V Cl2 + 2e- -> 2Cl- E°red = 1.36 V Fe2+ = 0.01 M and Cl- = 0.1 M (products)

Chlorine has higher reduction potential, so it is the cathode Iron is the anode, which means it is actually oxidized, so the equation is shown switched around. The actual net ionic equation is: Fe + Cl2 -> Fe2+ + 2Cl- Standard cell potential = 1.36 - -0.44 V = +1.80 V Q= [Fe2+][Cl-]^2 = 0.01 * (.1^2)= 10^-4 Plug into Nernst Equation Ecell=E°cell-RT/nF*LnQ n = 2 because 2 electrons are transferred at 298K, the equation can be simplified to Ecell=E°cell-.0592/n*logQ Ecell = 1.8 - .0592/2 * log(10^-4) =1.8 - (0.03*-4) = about 1.92 V

Complex Ion Formation

Complex Ion/ coordination compound = molecule in which cation is bonded to at least one electron pair donor (could be water) Electron pair donor molecules called ligands Example of such a complexation reaction is tetraaquadioxouranyl cation, which has water and oxygen ligands Complexes are held together with coordinate covalent bond, in which electron pair donor (Lewis base) and electron pair acceptor (Lewis acid) form stable Lewis acid-base adducts Complex ions important in macromolecules (like proteins) -Iron in hemoglobin can bind various gases, carries oxygen, carbon dioxide, carbon monoxide as ligands Many coenzymes (vitamins) and cofactors also contain complexes of transition metals (presence of transition metal allows coenzymes and cofactors to bind other ligands or assist with electron transfer Chelation: central cation can be bonded to the same ligand in multiple places -Generally requires large organic ligands that can double back to form a second or third bond with central cation -Chelation therapy often used to sequester took metals *Complexation reaction: A reaction in which a central cation is bound to one or more ligands

Strong Acid and Strong Base Titration

Consider titration of 10 mL of 0.1 N solution of HCl with 0.1 N solution of NaOH (for the example, an indicator that changes color at pH 8 would be better approximation) *you can see on the graph that just a little bit of base doesn't do much *The addition of base will alter concentrations of H+ and OH- near equivalence point, and will elicit most substantial changes in pH in that region (always 7 for strong acid base monovalent) (Equivalence point is roughly equal to midpoint of steep part) Plot pH VS quantity of NaOH Strong acid and strong base = equivalence point at 7 Strong acid and weak base = equivalence point pH < 7 Weak acid and strong base = equivalence point pH > 7 *Weak acid and weak base: very shallow drop, equivalence point near neutral

For a cell with the following half-reactions: Anode: SO2 + 2 H2O → SO42- + 4 H+ + 2 e- Cathode: Pd2+ + 2 e- → Pd How would decreasing the pH of the solution inside the cell affect the electromotive force (emf)? A. The emf would decrease. B. The emf would remain the same. C. The emf would increase. D. The emf would become zero.

Correct Answer: A Explanation: A change in pH has a direct correlation to the hydrogen ion (H+) concentration. Decreasing the pH increases the H+ concentration, which means the concentration of products has increased in the oxidation of sulfur dioxide. This means it would be harder to liberate electrons, thus decreasing the emf. One could also view this decrease in oxidation potential as an increase in reduction potential. If E°red,anode increases, then E°cell must decrease according to E°cell = E°red,cathode - E°red,anode.

Carbonated beverages are produced by dissolving carbon dioxide in water to produce carbonic acid: CO2 (g) + H2O (l) -> H2CO3 (aq) When a bottle containing carbonated water is opened, the taste of the beverage gradually changes as the carbonation is lost. Which of the following statements best explains this phenomenon? A. The change in pressure and volume causes the reaction to shift to the left, thereby decreasing the amount of aqueous carbonic acid. B. The change in pressure and volume causes the reaction to shift to the right, thereby decreasing the amount of gaseous carbon dioxide. C. Carbonic acid reacts with environmental oxygen and nitrogen. D. Carbon dioxide reacts with environmental oxygen and nitrogen.

Correct Answer: A Explanation: Carbon dioxide gas evolves and leaves the bottle, which decreases the total pressure of the reactants. Le Chatelier's principle explains that a decrease in pressure shifts the equilibrium to increase the number of moles of gas present. This particular reaction will shift to the left, which in turn will decrease the amount of carbonic acid and increase the amount of carbon dioxide and water. Oxygen and nitrogen are not highly reactive and are unlikely to combine spontaneously with carbon dioxide or carbonic acid, as in choices (C) and (D).

Which of the following is the gram equivalent weight of H2SO4 with respect to protons? A. 49.1 g B. 98.1 g C. 147.1 g D. 196.2 g

Correct Answer: A Explanation: First, it is helpful to know the molar mass of one mole of H2SO4, which is found by adding the atomic weights of the atoms that constitute the molecule: 98.1 g/mol Gram equivalent weight is the weight (in grams) that would release one mole of protons.Because sulfuric acid has two hydrogens per molecule, the gram equivalent weight is 98.1 g divided by 2, or 49.1 g.

Which of the following sets of conditions would be LEAST likely to result in ideal gas behavior? A. High pressure and low temperature B. Low temperature and large volume C. High pressure and large volume D. Low pressure and high temperature

Correct Answer: A Explanation: Gases deviate from ideal behavior at higher pressures and lower volumes and temperatures, all of which force molecules closer together. The closer they are, the more they can participate in intermolecular forces, which violates the definition of an ideal gas. At extremely high pressures, low volumes, and low temperatures, the size of the gas particles becomes significant, which also violates the definition of an ideal gas.

Lithium aluminum hydride (LiAlH4) is often used in laboratories because of its tendency to donate a hydride ion. Which of the following roles would lithium aluminum hydride likely play in a reaction? A. Strong reducing agent only B. Strong oxidizing agent only C. Both a strong reducing agent and strong oxidizing agent D. Neither a strong reducing agent nor a strong oxidizing agent

Correct Answer: A Explanation: Hydride ions are composed of a hydrogen nucleus with two electrons, thereby giving it a negative charge and a considerable tendency to donate electrons. LiAlH4 is therefore a strong reducing agent. Strong reducing agents tend to have metals or hydrides; strong oxidizing agents tend to have oxygen or a similarly electronegative element.

A certain chemical reaction has the following rate law: rate = k[NO2][Br2] Which of the following statements necessarily describe(s) the kinetics of this reaction? I. The reaction is second-order. II. The amount of NO2 consumed is equal to the amount of Br2consumed. III. The rate will not be affected by the addition of a compound other than NO2 and Br2. A. I only B. I and II only C. II and III only D. I, II, and III

Correct Answer: A Explanation: If the sum of the exponents (orders) of the concentrations of each species in the rate law is equal to 2, then the reaction is second-order. The exponents in the rate law are unrelated to stoichiometric coefficients, so NO2 and Br2 could have any stoichiometric coefficients in the original reaction and still be a second-order reaction, invalidating statement II. Statement III is incorrect because the rate can be affected by a wide variety of compounds. A catalyst, for example, could increase the rate.

Methanol reacts with acetic acid to form methyl acetate and water. Type of Bond Bond Disassociation Energy C - C 348 C - H 415 C = O 805 O - H 463 C - O 360 Based on the values in the table above, what is the heat of formation of methyl acetate in kJ/mol? A. 0 B. 464 C. 824 D. 1288

Correct Answer: A Explanation: Image = Acetic Acid At first glance, this might seem like a math-heavy problem, but it really doesn't require any calculations at all. We just have to keep track of which bonds are broken and which bonds are formed. Remember, breaking bonds requires energy, while forming bonds releases energy. Two bonds are broken: a C-O bond between the carbon and oxygen of methanol, and an O-H bond between the hydroxyl oxygen and hydrogen of the acetic acid. Two bonds are also formed: a C-O bond between the esterifying group and the oxygen of the methyl acetate, and an O-H bond between the hydroxyl group and a hydrogen to form water. Given that the same two bonds are broken and formed in this reaction, the energy change must be 0 kJ/mol

Which of the following combinations of liquids would be expected to have a vapor pressure higher than the vapor pressure that would be predicted by Raoult's law? A. Ethanol and hexane B. Acetone and water C. Isopropanol and methanol D. Nitric acid and water

Correct Answer: A Explanation: Mixtures that have a higher vapor pressure than predicted by Raoult's law have stronger solvent-solvent and solute-solute interactions than solvent-solute interactions. Therefore, particles do not want to stay in solution and more readily evaporate, creating a higher vapor pressure than an ideal solution. Two liquids that have different properties, like hexane (hydrophobic) and ethanol (hydrophilic, small) in Choice (A), would not have many interactions with each other and would cause positive deviation. Choices (B) and (C) are composed of liquids that are similar to one another and would not show significant deviation from Raoult's law. Choice (D) contains two liquids that would interact very well with each other, which would actually cause a negative deviation from Raoult's law—when attracted to one other, liquids prefer to stay in liquid form and have a lower vapor pressure than predicted by Raoult's law. Because they cannot escape the container, the vapor molecules above the surface of the liquid exert a pressure on the walls of the container. The vapor pressure is a measure of the presure (force per unit area) exerted by a gas above a liquid in a sealed container. Vapor pressure is a property of a liquid based on the strength of its intermolecular forces. A liquid with weak intermolecular forces evaporates more easily and has a high vapor pressure. A liquid with stronger intermolecular forces does not evaporate easily and thus has a lower vapor pressure. For example, diethyl ether is a nonpolar liquid with weak dispersion forces. Its vapor pressure at 20°C is 58.96 kPa. Water is a polar liquid whose molecules are attracted to one another by relatively strong hydrogen bonding. The vapor pressure of water at 20°C is only 2.33 kPa, far less than that of diethyl ether.

In a sealed 1 L container, 1 mole of nitrogen gas reacts with 3 moles of hydrogen gas to form 0.05 moles of NH3 at equilibrium. Which of the following is closest to the Kc of the reaction? A. 0.0001 B. 0.001 C. 0.01 D. 0.1

Correct Answer: A Explanation: The first step to answering this question is to write out the balanced equation for the reaction of H2 and N2 to produce NH3: N2 + 3 H2 <-> 2 NH3. This means that Kc is equal to Keq = [NH3]^2/[N2][H2]^3 . Because the volume is 1 L, the amount of each gas (in moles) is equal to the value of the concentration of each gas (in M). X is negligible because 0.05 is small Keq = [0.05]^2/[1-x][3-x]^3 [0.05]^2 = [1][3]^3 = 25X10^-4/ 27 = About 1X10^-4 =0.0001 *Exponents don't come from number of moles, they come from the balanced equation

As methanol is converted to methanal, and then methanoic acid, the oxidation number of the carbon: A. increases. B. decreases. C. increases, then decreases. D. decreases, then increases.

Correct Answer: A Explanation: The formula for methanol is H3COH, for methanal is H2CHO, and for methanoic acid is HCOOH. If we assign oxidation numbers to carbon in each molecule, it starts at -2, then becomes 0, then becomes +2: In general, it is often easier to think of oxidation as a gain of bonds to oxygen (or a similarly electronegative element) or loss of bonds to hydrogen for organic compounds. Therefore, because the carbon is oxidized as one converts from an alcohol to an aldehyde to a carboxylic acid, the oxidation number must increase.

In the reaction shown, if 39.05 g of Na2S are reacted with 85.5 g of AgNO3, how much of the excess reagent will be left over once the reaction has gone to completion? Na2S + 2 AgNO3 → Ag2S + 2 NaNO3 A. 19.5 g Na2S B. 26.0 g Na2S C. 41.4 g AgNO3 D. 74.3 g AgNO3

Correct Answer: A The formula weight of Na2S is 78.1 grams per formula unit; the formula weight of AgNO3 is 169.9 grams per formula unit. 39/78 = About 0.5 mol 86/170 = About 0.5 mol Because we need two moles of AgNO3 for every mole of Na2S, AgNO3 is the limiting reagent, and the correct answer choice will be in grams of Na2S. If 0.5 mol of AgNO3 are used up (0.5 = 2X, X=0.25), and Na2S will be consumed at half the rate of AgNO3 (based on their mole ratio), then 0.25 mol Na2S will be used up (0.5-x =0.25). We then have 0.25 mol excess Na2S, which has a mass of (80X0.25) = about 20 grams

If the surface area of electrode material in an electrochemical cell is tripled, what else is necessarily tripled? I. E°cell II. Current III. Keq A. I only B. II only C. I and II only D. II and III only

Correct Answer: B Explanation: Potential, as measured by E°cell, is dependent only on the identity of the electrodes and not the amount present. Similarly, the equilibrium constant depends only on the identity of the electrolyte solutions and the temperature. However, as the electrode material is increased, the surface area participating in oxidation-reduction reactions is increased and more electrons are released, making statement II correct.

A 0.040 g piece of magnesium is placed in a beaker of hydrochloric acid. Hydrogen gas is generated according to the following equation: Mg (s) + 2 HCl (aq) → MgCl2(aq) + H2(g) The gas is collected over water at 25°C, and the gauge pressure during the experiment reads 784 mmHg. The gas displaces a volume of 100 mL. The vapor pressure of water at 25°C is approximately 24.0 mmHg. Based on this data, how many moles of hydrogen are produced in this reaction? (Note: R = 0.0821 L*atm/ mol * K and R = 8.314 J/ K* mol) A. 4.04 × 10-5 moles hydrogen B. 4.09 × 10-3 moles hydrogen C. 3.07 × 10-2 moles hydrogen D. 3.11 moles hydrogen

Correct Answer: B The pressure of the gas is calculated by subtracting the vapor pressure of water from the measured pressure during the experiment: 784 mmHg - 24 mmHg = 760 mmHg, or 1 atm. This is because the reaction is carried out in an aqueous environment; the water present will contribute to the partial pressures of the gas over the liquid. The ideal gas law can be used to calculate the moles of hydrogen gas. The volume of the gas is 0.100 L, the temperature is 298 K, and R = 0.0821 L*atm/ mol * K Plugging in, we get: n= PV/ RT = 1atm * .1L / 0.08 X 300 = 1/ .8 X 300 = 1/ 240 = Choice (A) incorrectly substitutes 8.314 into the gas law, rather than 0.0821. Remember that the value of R depends on the other variables in the equation; using 1 atm in the numerator necessitates using 0.0821. Choice (C) incorrectly substitutes the wrong R and keeps the pressure in mmHg. Choice (D) also keeps the pressure in mmHg.

In the following reaction: Au2S3 (s) + H2 (g) → Au (s) + H2S (g) If 2 moles of Au2S3 (s) is reacted with 5 moles of hydrogen gas, what is the limiting reagent? A. Au2S3 (s) B. H2 (g) C. Au (s) D. H2S (g)

Correct Answer: B Explanation: A limiting reagent is by definition a reactant. Because Au and H2S are products, they cannot act as limiting reagents, eliminating choices (C)and (D). Next, realize that what you are shown is an unbalanced equation. To answer this question, we must balance the reaction: Au2S3 (s) + 3 H2 (g) → 2 Au (s) + 3 H2S (g) We are given 2 moles of gold(III) sulfide and 5 moles of hydrogen gas. To use up both moles of gold(III) sulfide, we would need 6 moles of hydrogen gas because there is a 1:3 ratio between these reactants. We have only 5 moles of hydrogen gas, so that will have to be the limiting reagent.

Which of the following is closest to the pH of a solution containing 5 mM H2SO4? A. 1 B. 2 C. 3 D. 4

Correct Answer: B Explanation: First, convert the concentration to 5 × 10-3 M. Next, because sulfuric acid is a strong acid, we can assume that, for the majority of sulfuric acid molecules (although not all), both protons will dissociate. The concentration of hydrogen ions is therefore 2 × 5 × 10-3, or 10-2. The equation for pH is pH = -log [H+]. If [H+] = 10-2 M, then pH = 2.

When ammonia, NH3, is used as a solvent, it can form complex ions. For example, dissolving AgCl in NH3 will result in the complex ion [Ag(NH3)]2+. What effect would the formation of complex ions to have on the solubility of a compound like AgCl in NH3? A. The solubility of AgCl will increase because complex ion formation will cause more ions to exist in solution, which interact with AgCl to cause it to dissociate. B. The solubility of AgCl will increase because complex ion formation will consume Ag+ ions and cause the equilibrium to shift away from solid AgCl. C. The solubility of AgCl will decrease because Ag+ ions are in complexes, and the Ag+ ions that are not complexed will want to associate with Cl- to form solid AgCl. D. The solubility of AgCl will decrease because complex ion formation will consume Ag+ ions and cause the equilibrium to shift toward the solid AgCl.

Correct Answer: B Explanation: Formation of complex ions between silver ions and ammonia will cause more molecules of solid AgCl to dissociate. The equilibrium is driven toward dissociation because the Ag+ ions are essentially being removed from solution when they complex with ammonia. This rationale is based upon Le Chatelier's principle, stating that when a chemical equilibrium experiences a change in concentration, the system will shift to counteract that change.

Which of the following statements could be true about a Na-Cd cell, based on the information below? Na+ + e- -> Na Ered = -2.71 V Cd2+ + 2e- -> Cd Ered = -.40 V A. It is a galvanic cell, and sodium is the cathode. B. It is an electrolytic cell, and cadmium is the anode. C. It is a galvanic cell, with E°cell = 3.11 V. D. It is an electrolytic cell, with E°cell = -3.11 V.

Correct Answer: B Explanation: If this were a galvanic cell, the species with the more positive reduction potential (cadmium) would be reduced. The cathode is always reduced in an electrochemical cell, so sodium could not be the cathode in such a galvanic cell, eliminating choice (A). Sodium would be the cathode in an electrolytic cell (smaller/less positive), however, which would make cadmium the anode. Thus, the answer is choice (B). Note that we do not have to determine E°cell because we already know the answer. However, the E°cell would be -2.71 - (-0.40) = -2.31 V for an electrolytic cell, and +2.31 V for a galvanic cell, eliminating choices (C) and (D).

For a certain chemical process, Delta G^o = 4.955 kJ/mol. What is the equilibrium constant Keq for this reaction? (Note: R = 8.314 J/mol*K) A. Keq = 1.0 B. Keq = 7.4 C. Keq = 8.9 D. Keq = 10

Correct Answer: B Explanation: Solve this question using the equation ΔG°rxn = -RT ln Keq. ΔGrxn° is 4955 J/mol and T = 298 K because the reaction is occurring under standard conditions. Because ΔG°rxn uses kilojoules in its units and R uses joules, one will have to be converted. Plugging into the equation, we get: -4955 J/mol = -8.314 J/mol*K * 298 K * ln Keq lnKeq = about 5000/ (8X300) = 2 ln Keq = 2 Keq = e^2 e is about 2.7, so Keq is between 2 and 3 If ln Keq = 2, then Keq = e2. e is approximately 2.7, so we're looking for a number between 4 (22) and 9 (32). Both choices (B) and (C) fit these criteria; however, 8.9 is very close to 9, so we can assume that its square root is very, very close to 3. The answer choice should be a bit smaller, so choice (B), 7.4, is correct.

Which of the following would make the strongest electrolytic solution? A. A nonpolar covalent compound with significant solubility. B. A ionic compound composed of one cation with +3 charge and three anions with -1 charge. C. A polar covalent compound with a small dissociation constant. D. An ionic compound composed of two cations with +1 charge and one anion with -2 charge.

Correct Answer: B Explanation: The best electrolytes dissociate readily (have a high dissociation constant) and are ionic compounds with large amounts of cations and anions. This rules out choices (A) and (C). Choice (D) has fewer total ions with a smaller total magnitude of charge and therefore is not as strong an electrolyte as choice (B).

One hundred grams of sucrose are dissolved in a cup of hot water at 80°C. The cup of water contains 300.00 mL of water. What is the percent composition by mass of sugar in the resulting solution? (Note: Sucrose = C12H22O11, density of water at 80 deg C = 0.975 g/mol) A. 25.0% B. 25.5% C. 33.3% D. 34.2%

Correct Answer: B Explanation: The mass percent of a solute equals the mass of the solute divided by the mass of the total solution times 100%. To find the mass of the solution, first find the mass of the solvent, water. Multiplying the volume of the water by the density gives a mass of 292.5 grams of water. Adding 100 grams of sugar yields a solution with a mass of 392.5 grams. Next, divide 100 grams of sugar by 392.5 grams and multiply by 100 to get 25.5%

Given the following standard reduction potentials: Zn2+ + 2e- -> Zn Ered = -0.763 V Ag+ + e- -> Ag Ered = +0.337 V What is the standard electromotive force of the following reaction? Zn2+ + 2 Ag → 2 Ag+ + Zn A. -2.2 V B. -1.1 V C. +1.1 V D. +2.2 V

Correct Answer: B Explanation: To determine the standard electromotive force of a cell, simply subtract the standard reduction potentials of the two electrodes. In this case, the cathode is zinc because it is being reduced; the anode is silver because it is being oxidized. Thus, E°cell = E°red,cathode - E°red,anode = - 0.763 - 0.337 = - 1.10 V While we must multiply the silver half-reaction by two to balance electrons, the actual value for the reduction potential does not change. Remember that the standard reduction potential is determined by the identity of the electrode, not the amount of it present.

A gas at a temperature of 27°C has a volume of 60.0 mL. What temperature change is needed to increase this gas to a volume of 90.0 mL? A. A reduction of 150°C B. An increase of 150°C C. A reduction of 40.5°C D. An increase of 40.5°C

Correct Answer: B Explanation: We will use Charles's law. First, we must convert the temperature to kelvins by adding 273 to get 300 K as the initial temperature. Think of this as a proportionality: If the volume is multiplied by 3/2 the temperature will also have to be multiplied by 3/2 Thus the final temperature is 450 K, which represents a 150 K increase.

A solution is prepared with an unknown concentration of a theoretical compound with a Ka of exactly 1.0. What is the pH of this solution? A. Higher than 7 B. Exactly 7 C. Less than 7 D. There is not enough information to answer the question.

Correct Answer: C Explanation: A higher Ka implies a stronger acid. Weak acids usually have a Ka that is several orders of magnitude below 1. The pKa of a compound is the pH at which there are equal concentrations of acid and conjugate base; the pKa of this compound would be -log 1 = 0. With such a low pKa, this compound must be an acid. Therefore, the pH of any concentration of this compound must be below 7.

If the reaction FeI (aq) + I2 (g) → FeI3 (aq) were exothermic, what effect would decreasing the temperature have on the equilibrium? A. The forward reaction rate and the reverse reaction rate both increase. B. The forward reaction rate decreases while the reverse reaction rate increases. C. The forward reaction rate increases while the reverse reaction rate decreases. D. The forward reaction rate and the reverse reaction rate both decrease.

Correct Answer: C Explanation: An exothermic reaction produces heat. Decreasing the temperature favors product formation, resulting in an increase in the forward reaction rate with a concomitant decrease in the reverse reaction rate.

Which of the following types of reactions generally have the same number of reactants and products? I. Double-displacement reactions II. Single-displacement reactions III. Combination reactions A. I only B. II only C. I and II only D. II and III only

Correct Answer: C Explanation: Typically, both single-displacement and double-displacement reactions have two reactants that swap either one or two components between the two species. Combination reactions, on the other hand, have more reactants than products because the reactants combine together to form the product.

In which of the following situations is it impossible to predict how the pressure will change for a gas sample? A. The gas is cooled at a constant volume. B. The gas is heated at a constant volume. C. The gas is heated, and the volume is simultaneously increased. D. The gas is cooled, and the volume is simultaneously increased.

Correct Answer: C Explanation: Both a change in temperature and a change in volume can affect a gas's pressure. So if one of those two variables is kept constant, as in choices (A) and (B), we'll definitely be able to predict which way the pressure will change. At a constant volume, heating the gas will increase its pressure, and cooling the gas will decrease it. What about when both temperature and volume are changing? If both changes have the same effect on pressure, then we can still predict which way it will change. This is the case in choice (D). Cooling the gas and increasing its volume both decrease pressure. Choice (C), on the other hand, presents too vague a scenario for us to predict definitively the change in pressure. Heating the gas would amplify the pressure, while increasing the volume would decrease it. Without knowing the magnitude of each influence, it's impossible to say whether the pressure would increase, decrease, or stay the same.

The entropy change when a solution forms can be expressed by the term ΔS°soln. When an ion dissolves and water molecules are ordered around it, the ordering would be expected to make a negative contribution to ΔS°soln. An ion that has more charge density will have a greater hydration effect, or ordering of water molecules. Based on this information, which of the following compounds will have the most negative ΔS°soln? A. KCl B. LiF C. CaS D. NaCl

Correct Answer: C Explanation: CaS will cause the most negative ΔS°soln because the Ca2+ and S2- ions have the highest charge density compared to the other ions. All of the other ions have charges of +1 or -1,whereas Ca2+ and S2- each have charges with a magnitude of 2.

Which of the following best describes why over-charging a Ni-Cd battery is not detrimental? A. The energy density of a Ni-Cd battery is high, so it can store more charge than other batteries per its mass. B. The electrodes of a Ni-Cd battery can discharge through the circuit when they are fully charged. C. The Ni-Cd battery will stop accepting electrons from an outside source when its electrodes are recharged. D. Ni-Cd batteries have a high surge current and can dissipate the overcharge before damage can occur to electrodes.

Correct Answer: C Explanation: During the recharge cycle, Ni-Cd cells will accept current from an outside source until the Cd and NiO(OH) electrodes are pure; at this point, the reaction will stop because Cd(OH)2 runs out and no more electrons can be accepted. Choices (A) and (B) are both true statements, but they fail to explain why overcharging the battery (continuing to try to run current into the battery even when the electrodes are reverted to their original state) is not a problem with Ni-Cd batteries. Finally, surge current refers to the initial burst of current seen in some batteries; once charged, the surge current will not increase even if the power source continues to be run because no additional charge will be stored on the electrodes, eliminating choice (D).

Which of the following best describes the number and character of the bonds in an ammonium cation? A. Three polar covalent bonds B. Four polar covalent bonds, of which none are coordinate covalent bonds C. Four polar covalent bonds, of which one is a coordinate covalent bond D. Four polar covalent bonds, of which two are coordinate covalent bonds

Correct Answer: C Explanation: First recall that ammonium is NH4+, while ammonia is NH3. Ammonium is formed by the association of NH3, an uncharged molecule with a lone pair on the nitrogen, with a positively charged hydrogen cation. In other words, NH3 is a Lewis base, while H+ is a Lewis acid. This type of bonding between a Lewis acid and base is a coordinate covalent bond.

An assay is performed to determine the gold content in a supply of crushed ore. One method for pulling gold out of ore is to react it in a strong cyanide (CN-) solution. The equation is provided below: Au + NaCN + O2 + H2O → Na[Au(CN)2] + NaOH An indicator is used during this reaction, and approximately 100 mL of a 2 M NaCN solution is used to reach the endpoint. How many moles of Au are present in the crushed ore? A. 0.01 mol B. 0.02 mol C. 0.10 mol D. 0.20 mol

Correct Answer: C Explanation: First, balance the chemical equation: 4 Au + 8 NaCN + O2 + 2 H2O → 4 Na[Au(CN)2] + 4 NaOH Now, determine the number of moles of NaCN used in the reaction: 0.1 L X 2 mol/L = 0.2 mol NaCN If 0.2 mol NaCN are used in the reaction, then 0.2 mol NaCN X 4 mol Au/8 mol NaCN (what you are looking for over what you have) = 0.1 mol Au is oxidized.

An electrolytic cell is filled with water. Which of the following will move toward the cathode of such a cell? I. H+ ions II. O2- ions III. Electrons A. I only B. II only C. I and III only D. II and III only

Correct Answer: C Explanation: In an electrolytic cell, ionic compounds are broken up into their constituents; the cations (positively charged ions) migrate toward the cathode, and the anions (negatively charged ions) migrate toward the anode. In this case, the cations are H+ ions (protons), so option I is correct. Electrons flow from anode to cathode in all types of cells, meaning that option III is also correct. Option II is incorrect for two reasons. First, it is unlikely that the anions in any cell would be O2- rather than OH-. Second, and more significantly, these anions would flow to the anode, not the cathode.

Rusting occurs due to the oxidation-reduction reaction of iron with environmental oxygen: 4 Fe (s) + 3 O2 (g) → 2 Fe2O3 (s) Some metals cannot react with oxygen in this fashion. Which of the following best explains why iron can? A. Iron has a more positive reduction potential than those metals, making it more likely to donate electrons to oxygen. B. Iron has a more positive reduction potential than those metals, making it more likely to accept electrons from oxygen. C. Iron has a less positive reduction potential than those metals, making it more likely to donate electrons to oxygen. D. Iron has a less positive reduction potential than those metals, making it more likely to accept electrons from oxygen.

Correct Answer: C Explanation: In the oxidation-reduction reaction of a metal with oxygen, the metal (iron) will be oxidized (donate electrons) and oxygen will be reduced (accept electrons). This fact allows us to immediately eliminate choices (B) and (D). A species with a higher reduction potential is more likely to be reduced, and a species with a lower reduction potential is more likely to be oxidized. Based on the information in the question, iron is oxidized more readily than those metals; this means that iron has a lower reduction potential.

Compound A has a Ka (equilibrium constant of acid dissociation) of approximately 10-4. Which of the following compounds is most likely to react with a solution of compound A? A. HNO3 B. NO2 C. NH3 D. N2O5

Correct Answer: C Explanation: Ka is equal to the ratio of products to reactants, with each species raised to its stoichiometric coefficient. A compound with a Ka greater than 10^-7 contains more H+ cations than HA- anions at equilibrium, which makes it an acid. This means that the compound in question is likely to react with a compound that is basic. Of the four answer choices, NH3 is the only base. IMPORTANT: Ka > 10^-7 is acidic Ka < 10^-7 is basic

What is the approximate pH of a 1.2 × 10-5 M aqueous solution of NaOH? A. 4.92 B. 7.50 C. 9.08 D. 12.45

Correct Answer: C Explanation: NaOH is a strong base; as such, there will be 1.2 × 10-5 M OH- in solution. Based on this information alone, the pOH must be between 4 and 5, and the pH must be between 9 and 10. Using the shortcut, pOH ≈ 5 - 0.12 = 4.88. pH = 14 - pOH = 9.12 (actual = 9.08).

Consider the following two reactions: 3 A + 2 B ? 3 C + 4 D (Reaction 1) 4 D + 3 C ? 3 A + 2 B (Reaction 2) If Keq for reaction 1 is equal to 0.1, what is Keq for reaction 2? A. 0.1 B. 1 C. 10 D. 100

Correct Answer: C Explanation: Reaction 2 is simply the reverse of reaction 1. This means that Keq for reaction 2 is the inverse of Keq of reaction 1, so the answer is (0.1)-1 = 10.

At standard temperature and pressure, a chemical process is at equilibrium. What is the free energy of reaction (ΔG) for this process? A. ΔG > 0 B. ΔG < 0 C. ΔG = 0 D. There is not enough information to determine the free energy of the reaction.

Correct Answer: C Explanation: Standard temperature and pressure indicates 0°C and 1 atm. Gibbs free energy is temperature dependent, but if a reaction is at equilibrium, ΔG = 0.

Given that the gases at the center of the sun have an average molar mass of 2 g/mol compressed to a density of 1.20 g/cm^3 under 1.30 × 10^9 atm of pressure, what is the temperature at the center of the sun? A. 2.6 × 104 K B. 2.6 × 106 K C. 2.6 × 107 K D. 2.6 × 1010 K

Correct Answer: C Explanation: The ideal gas law can be modified to include density (ρ) because the number of moles of gas, n, is equal to the mass divided by the molar mass (m/M). Thus, PV = nRT, plug in (m/M) for n, m/V replaced with density (p), so you get density = PM/RT Temp = PM/ density X R Note: 1.2 g/cm^3 = 1200 g/L T = 1.3X10^9 atm X 2 g/mol / (1200 X 0.0821) = 2.6 X 10^9 / 100 = 2.6 X 10^7 K

H2O + H+ ----> H3O+ Which of the following is the best name for the new bond formed in the reaction shown? A. Nonpolar covalent bond B. Ionic bond C. Coordinate covalent bond D. Hydrogen bond

Correct Answer: C Explanation: The reaction in this question shows a water molecule, which has two lone pairs of electrons on the central oxygen, combining with a free hydrogen cation. The resulting molecule, H3O+ has formed a new bond between H+ and H2O. This bond is created via the sharing of one of oxygen's lone pairs with the free H+ ion. This represents the donation of a shared pair of electrons from a Lewis base (H2O) to a Lewis acid (H+, electron acceptor). This type of bond is called a coordinate covalent bond.

A reaction is found to stop just before all reactants are converted to products. Which of the following could be true about this reaction? A. The reaction is irreversible, and the forward rate is greater than the reverse rate. B. The reaction is irreversible, and the reverse rate is too large for products to form. C. The reaction is reversible, and the forward rate is equal to the reverse rate. D. The reaction is reversible, and the reverse rate is greater than the forward rate.

Correct Answer: C Explanation: This scenario likely describes a situation in which a reaction has reached equilibrium very far to the right (with high product concentration and low reactant concentration). This reaction must be reversible because the reaction did not proceed all the way to the right. Any reaction in equilibrium has equal forward and reverse rates of reaction.

How many liters of 2 M Ba(OH)2 are needed to titrate a 4 L solution of 6 M H3PO4? A. 1.33 L B. 12 L C. 18 L D. 56 L

Correct Answer: C Explanation: Use the equivalence point equation: NaVa = NbVb. Ba(OH)2 can dissociate to give two hydroxide ions, so its normality is 2 M × 2 = 4 N. H3PO4 can dissociate to give three hydronium ions, so its normality is 6 M × 3 = 18 N. Plugging into the equation, we get (18 N)(4 L) = (4 N)(Vb). Therefore, Vb is 18 L.

One way to test for the presence of iron in solution is by adding potassium thiocyanate to the solution. The product when this reagent reacts with iron is FeSCN2+, which creates a dark red color in solution via the following net ionic equation: Fe3+ + SCN- → FeSCN2+ How many grams of iron sulfate would be needed to produce 2 moles of FeSCN2+? A. 110 g B. 220 g C. 400 g D. 500 g

Correct Answer: C Explanation: What you are shown is a net ionic equation. If two moles of FeSCN are created, two moles of Fe3+ must be used because the mole ratio is 1:1. Read the question stem carefully—we are asked for the mass of iron sulfate that is necessary, not the amount of iron, which would be 111.6 g, choice (A). Iron sulfate has the formula Fe2(SO4)3 because sulfate has a charge of -2 and iron has a charge of +3 (based on the net ionic equation). Therefore, one mole of iron sulfate is needed to make two moles of iron for the reaction. The molar mass of iron sulfate is 2 X 55.8 g/mol + (3X32.1 g/mol) + (12X16 g/mol) = 400 g/mol

What is the highest-energy orbital of elements with electrons in the n= 3 shell? A. s-orbital B. p-orbital C. d-orbital D. f-orbital

Correct Answer: C Explanation: When n = 3, l = 0, 1, or 2. The highest value for l in this case is 2, which corresponds to the d subshell. Although the 3d block appears to be part of the fourth period, it still has the principal quantum number n = 3. In general, the subshells within an energy shell increase in energy as follows: s < p < d < f (although there is no 3f subshell).

Experimenters notice that the molar concentration of dissolved oxygen in an enclosed water tank has decreased to one-half its original value. In an attempt to counter this decrease, they quadruple the partial pressure of oxygen in the container. What is the final concentration of the gas? A. Half the original concentration B. The same as the original concentration C. Double the original concentration D. Quadruple the original concentration

Correct Answer: C Explanation: Henry's law: [A] = kH X PA where [A] is concentration of A in solution, kH is Henry's constant (depends on identity of gas), and PA is partial pressure of A Initially the concentration of the gas is decreased to one-half its original value. Recall that concentration (solubility) and partial pressure are directly related—as one increases, the other increases. If the experimenters then quadruple the partial pressure of oxygen in the vessel, the solubility is also increased by a factor of four. One-half times four gives twice the original concentration value. Misreading the answer choices as being related to the concentration before the experimenters increased the partial pressure leads to choice (D).

The following electronic configurations represent elements in their neutral form. Which element is the strongest oxidizing agent? A. 1s22s22p63s23p64s2 B. 1s22s22p63s23p64s23d5 C. 1s22s22p63s23p64s23d104p1 D. 1s22s22p63s23p64s23d104p5

Correct Answer: D Explanation: A strong oxidizing agent will be easily reduced, meaning that it will have a tendency to gain electrons. Atoms usually gain electrons if they are one or two electrons away from filling up their valence shell. Choice (A) has a full 4s-orbital, meaning that it can only gain an electron if it gains an entire p subshell. Choice (B) has a stable, half-full 3d-orbital, so it is unlikely to pick up electrons unless it can gain five. Choice (C) has only a single electron in the outer shell, which is more likely lost upon ionization. Choice (D) would fill up its 4p-orbital by gaining one electron, so it is easily reduced.

Which phases of solvent and solute can form a solution? I. Solid solvent, gaseous solute II. Solid solvent, solid solute III. Gaseous solvent, gaseous solute A. I and II only B. I and III only C. II and III only D. I, II, and III

Correct Answer: D Explanation: All three choices can make a solution as long as the two components create a mixture that is of uniform appearance (homogenous). Hydrogen in platinum is an example of a gas in a solid. Brass and steel are examples of homogenous mixtures of solids. The air we breathe is an example of a homogenous mixture of gases; while these are more commonly simply referred to as mixtures, they still fit the criteria of a solution.

In a third-order reaction involving two reactants and two products, doubling the concentration of the first reactant causes the rate to increase by a factor of 2. What will happen to the rate of this reaction if the concentration of the second reactant is cut in half? A. It will increase by a factor of 2. B. It will increase by a factor of 4. C. It will decrease by a factor of 2. D. It will decrease by a factor of 4.

Correct Answer: D Explanation: Based on the information given in the question, the rate is first-order with respect to the concentration of the first reactant; when the concentration of that reactant doubles, the rate also doubles. Because the reaction is third-order, the sum of the exponents in the rate law must be equal to 3. Therefore, the reaction order with respect to the other reactant must be 3 - 1 = 2. If the concentration of this second reactant is multiplied by 1/2, the rate will be multiplied by (1/2)^2 = 1/4

Aluminum metal can be used to remove tarnish from silver when the two solid metals are placed in water, according to the following reaction: 3 AgO + 2 Al → 3 Ag + Al2O3 This reaction is a: I. double-displacement reaction. II. single-displacement reaction. III. oxidation-reduction reaction. IV. combination reaction. A. II only B. IV only C. I and III only D. II and III only

Correct Answer: D Explanation: In the reaction, there is a single displacement, with the silver in silver oxide being replaced by the aluminum to form aluminum oxide. This single-displacement reaction also necessitates a transfer of electrons in an oxidation-reduction reaction; silver, for example, changes from the +2 oxidation state to neutral. Aluminum changes from neutral to the +3 oxidation state.

Which of the following best describes ionic compounds? A. Ionic compounds are formed from molecules containing two or more atoms. B. Ionic compounds are formed of charged particles and are measured by molecular weight. C. Ionic compounds are formed of charged particles that share electrons equally. D. Ionic compounds are three-dimensional arrays of charged particles.

Correct Answer: D Explanation: Ionic compounds are composed of atoms held together by ionic bonds. Ionic bonds associate charged particles with large differences in electronegativity. Rather than forming molecules or being measured by molecular weight, as in choices (A) and (B), ionic compounds form large arrays of ions in crystalline solids and are measured with formula weights. In ionic bonds, electrons are not really shared but rather are donated from the less electronegative atom to the more electronegative atom, eliminating choice (C).

Pure sodium metal spontaneously combusts upon contact with room temperature water. What is true about the equilibrium constant of this combustion reaction at 25°C? A. Keq < 0 B. 0 < Keq < 1 C. Keq = 1 D. Keq > 1

Correct Answer: D Explanation: Solve this question using the equation ΔG°rxn = -RT ln Keq. ΔGrxn° is negative (as it must be for a spontaneous reaction), and R and T are always positive. Therefore, ln Keq must also be positive for the sign convention to work out correctly. If ln Keq > 0, then eln Keq > e0 → Keq > 1

The process of formation of a liquid solution can be better understood by breaking the process into three steps: 1. Breaking the solute into its individual components 2. Making room for the solute in the solvent by overcoming intermolecular forces in the solvent 3. Allowing solute-solvent interactions to occur to form the solution Which of the following correctly lists the enthalpy changes for these three steps, respectively? A. Endothermic, exothermic, endothermic B. Exothermic, endothermic, endothermic C. Exothermic, exothermic, endothermic D. Endothermic, endothermic, exothermic

Correct Answer: D Explanation: The first step will most likely be endothermic because energy is required to break molecules apart. The second step is also endothermic because the intermolecular forces in the solvent must be overcome to allow incorporation of solute particles. The third step will most likely be exothermic because polar water molecules will interact with the dissolved ions, creating a stable solution and releasing energy.

Which of the following experimental methods should NEVER affect the rate of a reaction? A. Placing an exothermic reaction in an ice bath. B. Increasing the pressure of a reactant in a closed container. C. Putting the reactants into an aqueous solution. D. Removing the product of an irreversible reaction.

Correct Answer: D Explanation: The question asks which of the following does NOT affect the rate of the reaction. Temperature directly affects the rate constant (k), making choice (A) incorrect. Changing the partial pressure of a gas will affect the number of effective collisions per time. This makes choice (B) incorrect—but note that concentration changes will not affect the rate of zero-order reactions. Solvents affect the rate of reactions depending on how the reactants interact with the solvent, making choice (C) incorrect. Removing the product of an irreversible reaction, choice (D), should not affect the rate of the reaction because the rate law does not depend on the concentrations of products.

What is the pH of a solution with an ammonium concentration of 70 mM and an ammonia concentration of 712 mM? (Note: The pKb of ammonia is 3.45.) A. 2.45 B. 4.45 C. 9.55 D. 11.55

Correct Answer: D Explanation: The question is asking for pH, but because of the information given, we must first find the pOH and then subtract it from 14 to get the pH. Use the Henderson-Hasselbalch equation: pOH = 3.45 + log (70/710) = 3.45 + log (1/10) pOH = 3.45 - 1 = 2.45 If the pOH = 2.45, the pH = 14 - 2.45 = 11.55.

If the value of E°cell is known, what other data is needed to calculate ΔG°? A. Equilibrium constant B. Reaction quotient C. Temperature of the system D. Half-reactions of the cells

Correct Answer: D Explanation: This answer comes directly from the equation relating Gibbs free energy and E°cell. ΔG° = -nFE°cell, where n is the number of moles of electrons transferred and F is the Faraday constant, 96,485 C/mol e-. To determine n, one must look at the balanced half-reactions occurring in the oxidation-reduction reaction.

What is the [H3O+] of a 2 M aqueous solution of a weak acid HXO2with Ka = 3.2 × 10-5? A. 6.4 × 10-5 M B. 1.3 × 10-4 M C. 4.0 × 10-3 M D. 8.0 × 10-3 M

Correct Answer: D Explanation: This question requires the application of the acid dissociation formula. Weak acids do not dissociate completely; therefore, all three species that appear in the balanced equation will be present in solution. Hydrogen ions and conjugate base anions dissociate in equal amounts, so [H+] = [XO2-]. If the initial concentration of HXO2 was 2 M and some amount xdissociates, we will have x amount of H3O+ and XO2- at equilibrium, with 2 M - x amount of HXO2 at equilibrium. Ka = [H3O+][XO2^-] / [HXO2] = x^2 / (2M - x) Note that x was considered negligible when added or subtracted, per usual. Solving for x, we get: x^2 / 2M = 3.2 × 10-5 6.4 X 10^-5 = x^2 64 X10^-6 = x ^2 x = 8 X10^-3 M

The following system obeys second-order kinetics. 2 NO2 → NO3 + NO(slow) NO3 + CO → NO2 + CO2(fast) What is the rate law for this reaction? A. rate = k[NO2][CO] B. rate = k[NO2]2[CO] C. rate = k[NO2][NO3] D. rate = k[NO2]2

Correct Answer: D Explanation: To answer this question, recall that the slow step of a reaction is the rate-determining step. The rate is always related to the concentrations of the reactants in the rate-determining step (not the overall reaction), so NO2 is the only compound that should be included in the correct answer.

After balancing the following oxidation-reduction reaction, what is the sum of the stoichiometric coefficients of all of the reactants and products? S8 (s) + NO3- (aq) → SO32- (aq) + NO (g) A. 4 B. 50 C. 91 D. 115

Correct Answer: D Explanation: Utilize the method described earlier to balance this redox reaction. The balanced half-reactions are: S8 + 24 H2O -> 8SO3^2- + 48 H+ + 32e- NO3^- + 4H+ + 3e- ->NO + 2H2O To get equal numbers of electrons in each half-reaction, the oxidation half-reaction will have to be multiplied by 3, and the reduction half-reaction will have to be multiplied by 32: 3S8 + 72 H2O -> 24 SO3^2- + 144 H+ + 96e- 32NO3^- + 128H+ + 96e- -> 32 NO+ 64 H2O This makes the overall reaction:3S8 + 32 NO3- + 8 H2O → 24 SO32- + 32 NO + 16 H+ The sum of the stoichiometric coefficients is therefore 3 +32 + 8 + 24 + 32 + 16 = 115.

Assuming identical conditions, which of the reactions displayed on the energy diagram proceeds the fastest?

D The faster a reaction can reach its activation energy, the faster it will proceed to completion. Because this question states that all conditions are equal, the reaction with the lowest activation energy will have the fastest rate. In the diagram, choice (D) has the lowest activation energy.

Which of the following correctly ranks the compounds below by ascending boiling point? I. Acetone II. KCl III. Kr IV. Isopropyl alcohol A. I < II < IV < III B. III < IV < I < II C. II < IV < I < III D. III < I < IV < II

D The key to answering this question is to understand the types of intermolecular forces that exist in each of these molecules because larger intermolecular forces correspond to higher boiling points. Kr is a noble gas with a full octet, so the only intermolecular forces present are London dispersion forces, the weakest type of intermolecular forces. Acetone and isopropyl alcohol are both polar, so both have dipole-dipole interactions, which are stronger than dispersion forces. However, isopropyl alcohol can also hydrogen bond, increasing its boiling point. Finally, the strongest interactions are ionic bonds, which exist in potassium chloride.

What is the density of argon gas at 4 atm and 127 deg C?

Density (p) = PM / RT = (4 atm X 20 g/mol)/ (0.082 L*atm/mol*K X 400K) = about 5 g/L

Disproportionation (Dismutation) Reactions

Disproportionation (dismutation) reactions are a type of redox reactions in which one element is both oxidized and reduced *many biological enzymes use this *Catalysis of peroxides by catalase 2H2O2 (aq) -> 2H2O (l) + O2 Left: H = +1, O = -1 Right: H=+1, O = -2 and 0 Oxygen disproportioned between water and molecular oxygen (peroxide ends up with -2 charge overall) *Oxygen is both reduced and oxidized *If you have a negative charge split over two atoms it can be -1/2, like in superoxide dismutase *usually accomplished by enzymes

Weak Acids and Bases

Do not completely dissociate in solution and have corresponding dissociation constants (Ka and Kb) Acid dissociation constant (Ka) = [H3O+][A-]/[HA] Base dissociation constant (Kb) = [B+][OH]/[BOH] The smaller the Ka, the weaker the acid and the less it will dissociate *Water is a pure liquid so its not included The smaller the Kb, the weaker the base and the less it will dissociate If Ka less than 1 its a weak acid If Kb less than 1 its a weak base HCO3- (aq) + H2O (l) <-> CO3 ^2- (aq) + H3O+ (aq) HCO3- is weak acid, so CO3^2- is conjugate base H3O+ is conjugate acid of H2O, a weak base Ka = [CO3^2-][H3O+] / [HCO3-] Reversible reaction: CO3 ^2- (aq) + H2O (l) <-> HCO3^- (aq) + OH- Kb = [OH-][HCO3^-] / [CO3^2-] If you add the reversible reactions together, the net reaction is 2H2O (l) <-> H3O+ + OH- *Net reaction = autoionization of water Kw = [H3O+][OH-] = 10^-14 = product of Ka X Kb Ka of acid * Kb of conjugate base = Kw = 10^-14 Kb of base * Ka of conjugate acid = 10^-14 = Kw If Ka is large, Kb is small and vice versa Strong acid (Ka towards infinity) will produce weak conjugate base The conjugate of a strong acid or base is sometimes termed inert because it is almost completely unreactive On the other hand, weak acids and bases tend to have conjugates that are also weak

Energy of a photon (Planck) Energy of electron transition (Bohr)

E = hf = hc/wavelength planck's constant*speed of light/wavelength (m) E=-RH [1/ni^2-1/nf^2] -> E = -RH/n^2 Combine them together: hc/wavelngth = -RH [1/ni^2-1/nf^2] This tells us the energy of emitted photon corresponds to difference in energy between higher energy initial state and the lower energy final state

Given the following half-reactions and E red values, determine which species would be oxidized and which would be reduced in a galvanic cell. Ag+ + e- -> Ag (s) E red = +0.80 V TI+ + e- -> TI (s) E red = -0.34 V

E red shows reduction potential Positive E red = spontaneous reduction, negative = non-sponteaneous In galvanic cell, Ag+ will be spontaneously reduced to Ag because Ag+ has more positive E red and is more favorable towards reduction, and TI will be spontaneously oxidized to TI+ Net ionic equation: Sum of 2 spontaneous reactions Ag+ + TI (s) -> TI+ + Ag (s) *to obtain oxidation pontential, both reactions and signs would be reversed (for example, TI (s) -> TI+ +e- and E ox = +0.34 V) *Reduction potentials are usually what is given

Lewis Structure/ Lewis dot Structure / formal Charge

Each dot represents S or P valence electron of atom Atoms in period three or greater can sometimes expand octet by using d-orbital Drawing molecules: Least electronegative in center, H always terminal, F, Cl, Br, and I usually terminal H-C (triple bond) N : Formal Charge: Difference between number of electrons assigned to atom in Lewis and number of elections normally found in that atom's valence shell * Formal charge = VE - N (nonbonding) - 1/2 N (bonding) or Formal charge = VE - dots - sticks -NOTE: Oxidation number vs formal charge: FC underestimates effect of electronegativity, ON overestimates (assumes more electronegative atom has 100% share of bonding E- pair) Resonance structures allows you to draw two or more Lewis structures that show same arrangements of atoms but differ in placement of electrons (allows for greater stability) -> bonds in compound actually a hybrid of all resonance structures together -> Formal charges = less stable structure = contribute less A Lewis Structure with: -small/no formal charge preferred -Less separation between opposite charges preferred -Negative charges on more electronegative atoms is more stable than negative charge on less electronegative -Exceptions: SO4 ^2- (S can have 12 VE)

Nernst Equation

Ecell= E°cell - (RT/nF) (lnQ) Concentration and emf of cell are related: emf varies with changing concentrations When conditions deviate from standard conditions, use Nernst equation Used for non-standard conditions E cell is emf of cell under nonstandard conditions E°cell = emf of cell under standard conditions R = ideal gas constant (8.314 J/K*mol) T = temperature in kelvins n= number of moles of electrons F = Faraday constant Q = reaction quotient for reaction at given point in time If T is 298 K, the equation can be simplified to: Ecell = E°cell - (0.0592/n)*logQ *Stick with base 10 logarithms to make calculations easier Q = [C]^c * [D]^d / [A]^a * [B]^b for generic reaction where aA + bB -> cC + dD Remember that only species in solution are included (ignore solids and liquids)

Covalent Bonding

Electrons pair shared between 2 or more atoms (typically non-metals) that have relatively similar values of electronegativity (can be single, double, or triple covalent bond based on how many pairs of electrons are shared between two atoms) -Degree to which pair of electrons is shared determines degree of polarity in covalent bond -Polarity determined by differences in electronegativity (makes it energetically unfavorable to form ions when electronegetivities are similar) -More electronegative atom gets larger share of electron density -Attraction that each electron in the shared pair has for the 2 positive nuclei of the bonded atoms -Shared equally, bond is non-polar (Atoms have identical/ nearly identical electronegatitivites (less than 0.5))(includes the 7 diatomic molecules, H2, N2, O2, F2, Cl2, Br2, I2) -Pair shared unequally, bond is polar (electronegativity difference of 0.5-1.7) -Polar bond creates a dipole (positive end at less electronegative atom, negative end at more electronegative atom)(partial negative and partial positive) -Dipole moment of polar covalent bond is vector given by equation: p=qd (p is dipole moment, q is magnitude of charge, d is the displacement vector separating the 2 positive charges (Measured in Debye units of coulomb-meters) NOTE: Presence of bond dipoles doesn't always result in molecular dipole/overall separation of charge -Molecule with polar bonds can be polar or non polar depending on spatial orientation of polar bonds (if vector sum is 0 = non polar, like in CCl4) -Coordinate Covalent: Both shared electrons are contributed by only one of the atoms (usually from lone pair attacking unhybridized p-orbital, indistinguishable once formed, usually found in Lewis acid-base, where acid accepts lone pair, and base donates pair: BF3 (acid) + NH3 (base))) -Tend to have lower melting and boiling points, poor conductors

Effective Nuclear Charge (Zeff) Which of the following atoms or ions has the largest effective nuclear charge? A. Cl B. Cl- C. K D. K+

Electrostatic attraction between VE and nucleus Measure of net positive charge experience by outermost electrons -Somewhat mitigated by electrons close to nucleus -Increases left to right within period/row Larger the Zeff = stronger the charge Stays pretty constant moving down group Effective nuclear charge increases from left to right across period since number of protons in nucleus increases. D) The effective nuclear charge refers to the strength with which the protons in the nucleus can pull on electrons. This phenomenon helps to explain electron affinity, electronegativity, and ionization energy. In choice (A), the nonionized chlorine atom, the nuclear charge is balanced by the surrounding electrons: 17 p+/17 e-. The chloride ion, choice (B), has a lower effective nuclear charge because there are more electrons than protons: 17 p+/18 e-. Next, elemental potassium, choice (C), has the lowest effective nuclear charge because it contains additional inner shells that shield its valence electron from the nucleus. Choice (D), ionic potassium, has a higher effective nuclear charge than any of the other options do because it has the same electron configuration as Cl- (and the same amount of shielding from inner shell electrons as neutral Cl) but contains two extra protons in its nucleus: 19 p+/18 e-. *Higher proton to electron ration (more protons) = larger effective nuclear charge

Atomic Emission Spectra and Atomic Absorption Spectra

Emission : room temperature - atoms are at ground state electrons can be excited to higher energy levels by heat or other forms of energy but then will fall back to the orgial state (excited state is brief) when it falls/returns to ground state it will result in *emission* of discrete amounts of energy in the form of photons (given off as energy) -> this gives rise to fluorescence -Not a continuum, has certain values, spectrum composed of light at specific frequencies (line spectrum) Electromagnetic energy of photons given by E = hc/Wavelength of radiation where h is Planck's constant c is speed of light in vacuum (3.00X10^8 m/s) * Combination of E=hf and c = f*wavelength -Each element has electrons excited at different energy levels = atomic emission spectrum (explained by Bohr model of H atom) Absorption: Electron excited to higher energy must absorb energy = energy absorption at specific wavelength = unique absorption spectrum *Delta E is same for absorption or emission between any two energy levels *Absorption is basis for color -> we see color of light that is NOT absorbed by compound Brain mixes subtraction frequencies and perceives complementary color (Carotene absorbs blue light, so our brain sees white light - blue light, which is yellow)

Chemical Mechanics

Enzyme active sites can be saturated (high substrate conditions), leading to maximal turnover, or less saturated (low substrate conditions) *While increasing the concentration of reactants can alter the reaction rate in first--or higher-order reactions, saturated solutions containing a catalyst have a maximum turnover rate that cannot adjust the rate constant or the reaction rate any higher. Series of steps in a reaction is known as the mechanism (the sum of which give the overall reaction) Intermediate: molecule that does not appear in the overall reaction (usually consumed almost immediately Rate determining step: Slowest step in any proposed mechanism (rate of reaction only as fast as this step) Collision theory of chemical kinetics: rate of reaction is proportional to the number of collisions per second between the reacting molecules (for a reaction to occur, molecules must collide, but not all collisions result in chemical reactions because molecules have to be in the right orientation and have enough energy) Minimum energy of collision needed for reaction to take place/ minimum amount of energy required for reaction to reach transition state = activation energy (Ea)/ energy barrier Rate of reaction = Z * f where Z is total number of collisions per second and f is fraction of collisions that are effective

What are the concentrations of each of the ions in saturated solution of CuBr, given that Ksp of CuBr = 6.27 X 10^-9 at 25 deg C. If 3 g CuBr are dissolved in water to make 1 L of solution at 25 deg C, what would solution be?

Equation: CuBr (s) -> Cu + (aq) + Br- (aq) Ksp = [Cu+][Br-] Ksp = x^2 6.27 X 10^-9 = x^2 64 X 10^-10 = x^2 X = 8.0 X 10^-5 So, Cu+ and Br- are both 8.0 X 10^-5 M (also represents the molar solubility of copper (I) bromide Now, convert 3g if CuBr into moles 3g / 143.5 g/mol = about 3/150 = 2X10^-2 mol 2X10^-2 mol in 1 L solution = molarity of 2X10^-2 M, which is WAY more than molar solubility of CuBr, so the solution is SUPERSATURATED

An 8.00 g sample of NH4NO3 (s) is placed into an evacuated 10 L flask and heated to 227°C. After the NH4NO3 completely decomposes, what is the approximate pressure in the flask? NH4NO3 (s) → N2O (g) + H2O (g) A. 0.410 atm B. 0.600 atm C. 0.821 atm D. 1.23 atm Correct Answer: D

Explanation: The first thing to do is balance the given chemical equation: NH4NO3 (s) → N2O (g) + 2 H2O (g). The mass given is 8.00 g, which represents 0.1 mol NH4NO3 (molar mass = 80 g/mol). When 0.1 mol of the solid decomposes, it will form 0.1 mol N2O and 0.2 mol water. This gives approximately 0.3 moles of gas product. The ideal gas equation can be used to obtain the pressure in the flask: P = nRT/v = 0.3 mol X 0.0821 X 500 / 10 L= 1.2 atm Choice (C) is the result if one assumes the equation is balanced, obtaining 0.2 mol gas as the product.

A group of students prepares to standardize a Na2S2O3 solution. 32 mL of the Na2S2O3 solution is titrated into 50 mL of 0.01 M KIO3 solution to reach the equivalence point. They first titrate the KIO3 solution until it loses color, then add starch indictor until the reaction is complete. The reaction proceeds in there two steps: IO3^- + I- + H+ -> I3- + H2O I3^- + S2O3^2- -> I- +S4O6^2- Determine the concentration of the sodium thiosulfate solution at the beginning of the experiment

First balance equations IO3^- + I- + H+ -> I3- + H2O I3^- + S2O3^2- -> I- +S4O6^2- IO3^- + 2I- + 6H+ -> I3- + 3H2O (charges are not balanced because there is +3 charge on left and -1 on right)(we want there to be same charge on both sides) I3^- + 2S2O3^2- -> 3I- +S4O6^2- IO3^- + 8I- + 6H+ -> 3(I3-) + 3H2O I3^- + 2S2O3^2- -> 3I- +S4O6^2- Each iodate (IO3-) interacts with 3 triiodide anions, and each triiodide anion interacts with 2 thiosulfate anions, so the mole ratio of thiosulfate to iodate is 6 to 1 IO3^- + 8I- + 6H+ -> 3(I3-) + 3H2O 3(I3^-) + 6S2O3^2- -> 9I- +3S4O6^2- Now find molarity from 50 mL 90.05L) KIo3 solution 0.01M IO3^- (0.05 L) (6 mol S2O3^2- / 1 mol IO3^-) = 3X10^-3 mol Molarity of S2O3^2- = 3X10^-2 mol . (32mL / 1000) = 3X10^-3 / 32X10^-3 = 0.1 M

What amount of energy is required to change a 90 gram ice cube at -10 deg C to vapor at 110 deg C? Specific heat of water (g) = 2.00 J/g*K Specific heat of water (s) = 2.18 J/g*k Specific heat of water (l) = 4.18 J/g*K = 1 cal/ g*K Delta H of fusion = 6.02 kJ/mol Delta H vapor = 40.67 kJ/mol

First convert mass to moles: n = 90 g H2O/ 18 g/mol = 10/2 = 5 moles H2O Start in ice phase (solid -liquid transient occurs at 0 deg C) First step involves change in temp q1 = m of ice * c of ice * Delta T -> 90g*(2.18 J/g*K) * 10K = 1800 J = 1.8 kJ Step 2: Convert ice into liquid form, no temp change: q2 = mL = n of H2O * delta H of fusion -> 5 mol * 6.02 kJ/mol = 30 kJ Step 3: Heat water up to liquid-gas transition temp at 100 deg C q3 = m of water * c of water * Delta T -> 90g * 4.18 J/g*K * 100 K = 36 kJ Step 4: Vaporize water, no temp change q4 = mL = n of H2O * delta H of vaporization -> 5 mol * 40.67 kJ/mol = 200 kJ Step 5: Heat to target temp of 110 deg C q5 = m of steam * c of steam * Delta T 90g * 2.00 K/g*k * 10 K = 1.8 kJ Total heat = q1+q2+q3+q4+q5 = 1.8 + 30+36+ 200 +1.8 = about 270 kJ

Isothermal Process

First law of thermodynamics = Delta U = Q - W where U is change in internal energy of system, Q is heat added, and W is work done Isothermal Process: Temp is constant, so U is constant (Delta U = 0) because temp and internal energy are directly proportional, so equation becomes Q=W (heat added = work done BY system) -Produces hyperbolic curve, with work under the curve (area under curve represents work performed by gas and heat that entered system)

If IP< Ksp: If IP = Ksp If IP>Ksp:

If IP< Ksp: unsaturated (solute will continue to dissolve)(solution not yet at equilibrium )(dissolution favored) If IP = Ksp: Saturated (solution is at equilibrium: dissolution and precipitation are equal) If IP>Ksp: Supersaturated (precipitation will occur) (solution beyond equilibrium) (unstable-> can lead to spontaneous precipitation of the excess dissolved solute)

Polyvalent Acids and Bases

First region: little base added dominant species is CO3^2- Flat part of curve = buffer regions, corresponding to the pKa of HClO3- (Ka = 5.6 X 10^-11, pKa = 10.25) *The center of the buffer region = half equivalence point because half of a given species has been protonated or deprotonated Second region: more acid added and predominant species are CO3^2- and HCO3- Third region: Begins with the equivalence point in which all of the CO3^2 is titrated to HCO3-. (Sharp drop in pH) -> HCO3- is the dominant species Fourth region: acid has neutralized half of the HCO3- and H2CO3 and HCO3- are in roughly equal amounts *flat region is second buffer region and second half equivalence point between regions 3 and 4 (pKa of H2CO3 = 6.37, Ka = 4.3 X 10^-7) Region 5 starts with second equivalence point, as all of the HCO3^- is converted to H2CO3 (rapid pH change) *Titration of amino acids will have three equivalence points, for titration of carboxyl group, amino group, and acidic or basic side chain

What is the pH of a solution made from 1 L of 0.05 M acetic acid (CH3COOH, Ka = 1.8 X 10^-5 ) mixed with 500 mL of 1 M acetate (CH3COO-)

First, find concentration of acetic acid and acetate in final solution 2 solutions mixed, so some dilution of both Ni * Vi = N2 * V2 For acetic acid: Ni = Ch3COOH normality (0.05, same as molarity) Vi = 1 L N2 = final normality of Ch3COOH V2 = 1.5 L N2 = final normality of Ch3COOH = 0.033 N For acetate: Ni = Normality of CH3COO= (1N) Vi = 0.5 L V2 = 1.5 L N2 = final normality of Ch3COO- = 0.33 N Now use Henderson -Hasselbalch pH = pKa + log [A- / HA] = -log (Ka = 1.8 X 10^-5) + log (0.33 N / 0.033 N) = about 4.8 + 1 = about 5.8

If you mix 180 g of the following compounds in 250 mL of water (density = 1 g/mL) what are their concentrations in molality, molarity, and normality (for acid-base chem)? Glucose 180 g/mol Carbonate 60 g/mol

For both: Molarity cant be calculated directly because final volume is unknown. In dilute solutions, M =m about, at higher concentrations, M< m because solute particles increase overall volume of solution Glucose: Molality : 1 mol / 0.25 kg = 4 m Normality: 1 N (glucose does not dissociate) Carbonate (H2CO3): 3 mol / 0.25 kg = 12 m Normality: Approximately 24N (twice molality and approximate molarity) *If you know the Molarity of an acid or base solution, you can easily convert it to Normality by multiplying Molarity by the number of hydrogen (or hydroxide) ions in the acid (or base). N = (M)(number of hydrogen or hydroxide ions)For example, a 2 M H2SO4 solution will have a Normality of 4N (2 M x 2 hydrogen ions). A 2 M H3PO4, solution will have a Normality of 6N.

Complex Ions (Coordination Complexes)

Formation of complex ions (shift in Eq to right) increases the solubility of salt in solution (opposite of common ion effect) *Composed of metallic ions bonded to various neutral compounds/anions, referred to as ligands *central cation bonded to electron pair donors called ligands *forming complex ion involves electron pair donors and acceptors = coordinate covalent bonding Complexes are more stable in solution than isolated ions (complex ions have multiple polar bonds, lots of dipole dipole interactions) = higher Ksp values -Complex ions often mixture of solution -Dissolution of original solution = Ksp, and the subsequent formation of complex ion in solution = Kf Kf is significantly larger than Ksp of compound providing metal ion *Kf = eq constant for complex ion -Initial dissolution is rate limiting step -Complex ion form becomes more soluble in solution If Ksp is really big negative exponent (like 10^-30) then the forward reaction is not favorable Large kf value is a sign that the formation of product of second reaction is highly favorable, and if the two reactions are simultaneous, complete consumption of original ion will occur -The effect of complex ion increasing solubility isn't normal, its the opposite of what is usually seen

Aldol Addition

Forms C-C bonds Nucleophilic attack on carbonyl carbon of aldehyde or ketone by enolate ion -Enolate attacks carbonyl carbon leading to new C-C bond

Applications of Stoichiometry

Generate mole ratio of one reactant/product to another: 2H2 + O2 -> 2H2O 1 mole of H consumed, 1 mole of H2O can be produced 1 mole of O consumed, 2 moles of H2O can be produced Common conversions: 1 mole of ideal gas at STP = 22.4 L 1 mole = Avogadro's number (6.022 X 10^23 particles) 1 mole of A = A's molar mass (grams, from periodic table)

Equilibrium Calculations

If Keq = 1 means that concentrations of products and reactants are same (becuase Keq = [Products]/[Reactants]) *If concentration of products is greater, Keq is "top-heavy" and fraction = greater than 1 *If reactants greater, Keq less than 1 *Keq often expressed as exponent, so favoring products = large and positive Keq (the larger the exponent, the less reactant that will be present at eq = reaction almost to completion) *Large negative exponent = reaction favors reactants at eq (only small amount of reactant converted to prodcut) Example: A (Reactant)= B+C with Keq = 10^-12 and A=1M, so Keq = [C][B]/[A] 10^-12 = (x)(x)/ (1-x) We know x is negligible becuase Keq is so small, so we can just say that the denominator is 1 10^-12 = x^2 / 1, so x = 10^-6

Law of Mass Action

Gives expression for eq constant, Keq. The reaction quotient, Q has same form but can be calculated at any concentrations of reactants and products If the system is at equilibrium at a constant temperature, then the following ratio is constant: Keq = [C]^c * [D]^d / [A]^a * [B]^b for generic reversible reaction where aA + bB = cC + dD *Keq IS CONSTANT AT CONSTANT TEMPERATURE Example: 2A = B+C Rate (f) = rate (reverse) = kf [A]^2 = kr [B][C] =kf/kr = [B][C]/[A]^2 *Kf/kr = Kc = Keq *if reaction occurs in more than one step, Keq found by multiplying equilibrium constants for each step (concentration of products/ concentration of reactants) * When dealing with gases, use Kp where p indicates pressure *Pure solids and liquids do not appear in the law of mass action (only aqueous and gasses) Expression for equilibrium constant for 3H2 (g) + N2 (g) = 2NH3 (g) is: Kc=Keq = [NH3]^2 / [H2]^3 * [N2] , so Kp = [PNH3]^2 / [PH2]^3 * [PN2] Note: At equilibrium, Entropy is at maximum, and Gibbs free energy is at minimum! *In equilibrium expressions, the exponents = coefficients in balanced equation. In rate laws, exponents must be determined experimentally and often dont = stoichiometric coefficients Properties of Law of Mass action: *Concentrations of liquids and gases not in eq constant expression because they are = 1 *Equilibrium constant (Keq) is temperature dependent *Larger Keq = farther right equilibrium position is *If eq constant in one direction is Keq, the eq constant fro reverse reaction is 1/keq

In chromatography, which set of characteristics to the mobile and stationary phase will allow a sample to travel the furthest?

High affinity to the mobile phase (moves with it), low affinity to the stationary phase (Silica gel - moves away from it)

Highest Ka = __________ pKa = __________ acid

Highest Ka = lower pKa = stronger acid

Transition States/ Activated Complex

Highest energy relative to reactants and products (denoted by symbol that looks like two + stacked on top of each other) Can revert back to reactants or go to products with out additional energy Not same as intermediates because transition states are theoretical constructs that exist at the point of max energy, rather than distinct identities with finite lifetimes Intermediates exist at "valleys" in reaction diagrams, transition states are mountains Only theoretical structures, can't be isolated Reaction coordinate traces reactions to products

Solutions

Homogenous (the same throughout) mixtures of two or more substances that combine to form a single phase (usually liquid) All solutions are considered mixtures, but not all mixtures are considered solutions (Mixtures are combinations of gases) Solution consists of solute (NaCl, NH3, C6H12O6, or CO2, polar covalent), in solvent (like H2O, benzene, or ethanol) Solvent is component of solution that remains in the same phase (if 2 substances are already in same phase then solvent is the one with greater quantity) If equal amount, solvent is one that is most commonly solvent Solute molecules move about freely in solvent and interact with intermolecular forces (ion-dipole, dipole-dipole, hydrogen bonding). Dissolved solute molecules react pretty freely, so reactions occur easily Solvation/dissolution: electrostatic interaction between solute and solvent (when water is solvent this can be called hydration) -Breaking intermolecular interactions between solvent molecules and forming new ones between solute and solvent When new interactions are stronger than original ones, salvation is exothermic (favored at low temp)(dissolution of gas into liquid, CO2 into water) *Most dissolutions are endothermic When new interactions are weaker/less stable, salvation is endothermic (favored at high temp)(most common)(powder into water)(energy/heat must be applied) Ideal solution: Enthalpy dissolution = 0 (strength of new interactions= strength of old interactions) *Proteins dissolve in solution with hydrophilic amino acids on outside and hydrophobic inside because this maximizes increase in entropy during dissolution Spontaneity of dissolution depends on enthalpy change (solutions may form endothermic or exothermic dissolutions) and on the entropy change that occurs (entropy = degree to which energy is dispersed/amount of energy distributed from system/ measure of molecular disorder/ number of energy microstates). At Constant temp and pressure, entropy always increases upon dissolution Spontaneity also depends on Gibbs free energy (spontaneous = decrease in free energy, nonspontaneous = increase in free energy) NaCl in water: Na+ and Cl- ionic bonds broken, and hydrogen bonds between water molecules broken. This requires energy (endothermic). Water is polar, so it can interact with each ion through ion-dipole interactions (partially positive H end of H2O surround Cl-, and partially negative O surround Na+ -> formation of these bonds is exothermic, but magnitude is less than energy required to break ionic bonds and hydrogen bonds) Overall dissolution of salt into water is +3.87 kJ/mol, favored at high temp -Entropy: Ions freed from lattice structure, greater number of energy microstates = entropy increase, but water becomes more restricted because of new interactions with ions (small decrease in entropy), but overall increase in entropy -Low endothermicity and large positive change in entropy = spontaneous (Delta G = Delta H - T delta S) Proportion of solute to solvent is small = dilute Dilute and concentrated considered unsaturated if max equilibrium concentration (saturation) not reached Solubility ultimately function of thermodynamics When change in Gibbs FE reaches negative at given temp = spontaneous process and solute said to be soluble Positive change in Gibbs FE = non spontaneous = solute is insoluble Solubility of solids can be increased by increasing temperature. Solubility of gases can be increased by decreasing temperature or increasing the partial pressure of the gas above the solvent (Henry's Law) *Group I metals, ammonium, nitrate, and acetate salts are always soluble In general, solutes considered soluble if they have molar solubility above 0.1 M in solution Sparingly soluble salts: Solutes that dissolve minimally in solvent (molar solubility under 0.1M)

Equivalents

How many moles of the thing we are interested in (protons, hydroxide ions, electrons, or ions) will one mole of a given compound produce *Sodium will donate 1 mole of electrons (1 equivalent) while Magnesium will donate 2 moles of e- (2 equivalents) *To gather one mole of H+ we could use 1 mole of HCl, half mole of H2SO4, or 1/3 mole of H3PO4 Gram equivalent Weight: The amount of compound, in grams, that produces 1 equivalent of the particle = molar mass / n where n is the number of particles of interest per molecule *We would need 31 grams of H2CO2 (molar mass = 62 g/mol) to produce one equivalent of H ions because each H2CO2 can donate 2 H+ (n=2) To determine how many equivalents are present, equivalents = mass of compound (g)/ gram equivalent weight

Ideal Gases

Hypothetical gas with molecules that have no intermolecular forces and occupy no volume An ideal gas follows the gas laws at all temps and pressures. Real gas deviates from these laws at high pressures (low volumes ) and low temps becuase of intermolecular forces or volume effects Ideal Gas Law = PV = nRT where P is pressure, V is volume, n= number of moles, R = ideal gas constant = 8.21 X10^-2 (L*atm/mol*K) = 8.314 J/K*mol Can be rearranged for density, where n = m/M (mass/ molar mass) Therefore, PV = (m/M) RT So, density (p) = m/V = PM/RT A mole of gas at STP occupies 22.4 liters

Osmotic Pressure

II=iMRT i: # of particles in solution M: molarity R: gas constant T: tempt in Kelvins Sucking pressure generated by solutions in which water is drawn into solution Amount of pressure that must be applied to counteract attraction of water molecules Water moves towards higher solute concentration Pure water (no solute) will traverse semipermeable membrane to solution containing solute particles, and increase water as result

Increasing pH of H2SO4 (aq) = H+ (aq) + HSO4^- (aq): Decreasing pressure of 2C (s) + O2 (g) = 2 CO (g): Warming CH4 (g) + 2O2 (g) = CO2 (g) + 2H2O (l) + heat: Removing water from H3PO4 (aq) + H2O (l) = H3O+ (aq) + H2PO4- (aq):

Increasing pH of H2SO4 (aq) = H+ (aq) + HSO4^- (aq): [H+] decreases, shifting reaction to right Decreasing pressure of 2C (s) + O2 (g) = 2 CO (g): reaction shifts right, favoring side with more moles of gas Warming CH4 (g) + 2O2 (g) = CO2 (g) + 2H2O (l) + heat: Reaction shifts left, using additional heat energy to produce more reactants Removing water from H3PO4 (aq) + H2O (l) = H3O+ (aq) + H2PO4- (aq): Reaction shifts left. All concentrations would increase proportionately because there are more products than reactants (and the. stoichiometric coefficient is q for each reactant and product), the value of Q will increase)

Solubility Product Constant

Ionic solid into polar solvent dissociates into its component ions: Dissociation of solute in solution represented by: Am*Bn (s) <-> m A^n+ (aq) + n B^m- (aq) Sparingly soluble salts (like AgCl): ionic compounds with low solubtility in aqueous solutions Degree of solubility determined by relative changes in enthalpy and entropy associated with dissolution of ionic solute at given temp and pressure AgCl (s) <-> Ag^+ (aq) + Cl - (aq) Eq constant for solubility in aqueous solution: Solubtility product constant (Ksp) = [A^n+]^m[B^m-]^n where concentrations of ionic constituents are eq (saturation) concentrations Ksp = [Ag+][Cl-] For law of mass action solutions, there is no denominator *pure solids and liquids dont appear in eq constant *Ksp reactions should never have denominator because solid salt is reactant *Like all other product constants, solubility product constants are temp dependent *when solution consists of gas dissolved in liquid, value of eq constant depends on pressure (increases with increasing temperature for non-gaseous solutes and decrease for gas solutes . (Higher pressures favor dissolution of gas solutes, so Ksp larger for gases at higher pressures than lower ones) *gases more soluble in solution as pressure increases (results in the bends) Molarity of a solute in saturated solution is called molar solubility

Isobaric and Isovolumetric (Isochoric) Processes

Isobaric: pressure of system is constant Isovolumetric (Isochoric): No change in volume, gas neither expands nor compresses, no work performed -Delta U = Q (change in internal energy = heat added to system) -Vertical line on P-V graph, no area under curve

Determine the concentration of hydrogen ions and pH of a solution of 0.2 M acetic acid (Ka = 1.8 X 10^-5)

Ka = [H3O+][CH3COO-]/ CH₃COOH 1.8 X 10^-5 = x^2 / 0.2M - x (negligible) 1.8 X 10^-5 = x^2 / 0.2 0.36 X 10^-5 = x^2 sqr rt 3.6 X 10 ^-6 = about sqr rt 4X10^-6 = 2 X 10^-3 M -log [ h3O+] = -log(10^-3) + -log (2) 3+ -0.2 = 2.8

3 moles of N2O4 is placed in a 0.5 L container and allowed to reach equilibrium according to: N2O4 (g) = 2 NO2 (g) What is eq concentration of NO2, given Keq = 6X10^-6

Keq = [NO2]^2 / [N2O4] Starting concentration of N2O4 = 3M/0.5 L = N2O4/ 1 L = 6M x= amount of N2O4 thats reacts, 2x represents amount of NO2 that is produced 6X10^-6 = 2x^2/[6-x], but the keq indicates that x is negligible, so we can simpligy 6X10^-6 = 4x^2/6 36X10^-6 = 4x^2 9X10^-6 = x^2 x= 3X10^-3 M BUT WAIT, this is the amount of N2O4 that reacts, we are looking for NO2 produced, which is twice the amount of N2O4 that reacts So, the final answer = 6X10^-3 M

What serves as the backbone for DNA?

Sugars linked by phosphodiester bonds to phosphate groups DNA consists of a sugar phosphate backbone that has nitrogenous bases attached to the sugar rings

LDF and Dipole-Dipole Interactions

LDF: -Type of Van Der Waals force -millions together have a lot of force -In a given moment, electron density may be unevenly distributed = rapid polarization/counterpolarization = formation of short lived dipole moments -LDF = attraction/repulsion of interactions of short-lived dipoles -Only in short proximity -Strength depends on degree and ease by which molecule can be polarized (how easy electrons shift) -Large molecules more easily polarizable = greater LDF -Only force in Nobel gases, causes them to liquify Dipole-Dipole Interactions: -Only in solid and liquid phase because gas too far apart -Dipole consists of segment of molecule with partial positive and negative regions, positive end of one molecule attracted to negative of another -Cause polar species to have higher melting and BP LDF and DD are both electrostatic forces between opposite partial charges, but DD is stronger/lasts longer *Carbonyl groups have dipoles that cause nucleophilic attack

Limiting Reagent/Reactant If 27.9 g of Fe react with 24.1 g of S to produce FeS, what would be the limiting reagent? How many grams of excess reagent would be present in the vessel at the end of the reaction? Fe + S -> FeS

Limits amount of product that can be formed Reactant that remain after all limiting reagent is used = excess regeant/reactant *All comparison must be done in units of moles Mass of Iron (Fe)= 27.9 g Mass of Sulphur (S) = 24.1 g Molar mass of Iron (Fe) = 55.85 g/mol Molar mass of sulphur (S) = 32 g/mol 27.9 g FeX 1 mol Fe/55.8 g Fe = 0.5 mol Fe 24.1 g S/ 32.1 g S = 0.75 mol S The limiting reagent is Fe (because 1 mole of Fe is needed to react with 1 mole of S) 0.75 - 0.5 = 0.25 mol S in excess 0.25 mol S x (32.1 g S/mol S) = 8 g S in excess

pH and pOH Scales

Logarithmic scales showing concentration of hydrogen and hydroxide ions P-Scales: Negative logarithm of items pH = -log [H+] = log (1/[H+]) pOH = -log [OH-] = log (1/[OH-]) At 298 K, both ions have concentrations of 10^-7 M, so pH and pOH = 7 at 298K -log (10^-7) = 7 So, pH + pOH = 14 pH less than 7 (or pOH greater than 7) = excess of hydrogen ions = acidic pH of greater than 7 (pOH less than 7) = excess of hydroxide ions = basic Remember: log (xy) = log(x) + log(y) If [H+] = 0.001 or 10^-3 , then pH = 3 and pOH = 11 If Kb = 1 X 10^-12, then pKb = 12 IF THE NONLOGARITHMIC VALUE IS WRITTEN IN PROPER SCIENTIFIC NOTATION, IT WILL BE IN THE FORM: n X 10^-m where n is a number between 1 and 10. Taking the negative log and simplifying, p value will be: -log (n X 10^-m) = m-log (n) Because n is between 1 and 10, log n is between 0 and 1 (closer n is to 1, closer log n will be to 0)(closer n is to 10, closer log is to 1) p value is about equal to m- 0.n where 0.n slides n one position to left (dividing n by 10) If Ka = 1.8 X 10^-5, pKa is -log (1.8) + -log (10^-5) = 5 - 0.18 = about 4.8

Lyman, Balmer, Paschen series

Lyman: A set of spectral lines that appear in the UV region when a hydrogen atom undergoes a transition from energy levels n (greater than or equal to) 2 to n=1. -Larger energy transmission = shorter photon wavelength in UV region Balmer: Corresponds to transitions from energy levels n (greater than or equal to) 3 to n = 2 (includes 4 wavelengths in visible region) Paschen: n (greater than or equal to) 4 to n =3 Energy associated with change in quantum number from higher initial value (n1) to lower final (nf) = energy of photon from Planck's quantum, theory Energy is inversely proportional to wavelength (E = hf = hc/wavelength) Takeaway: -Each element has certain set of energy levels. For e- to move from lower to high energy level, they need to absorb a unique amount of energy/light. When going from high to low energy level, they emit the same amount

Dilution

M1V1=M2V2 or MiVi = MfVf (Initial and final) Where M is molarity and V is volume A solution is diluted when solvent added to solution of higher concentration to produce a solution of lower concentration *Can also be expressed in parts per _____ (for example, ppm = parts per million, 10^-6, 1 mg/ L

Every sparingly soluble salt of general formula: MX will have Ksp = ? MX2 will have Ksp = ? MX3 will have Ksp = ?

MX will have Ksp = x^2 MX2 will have Ksp = 4x^3 MX3 will have Ksp = 27x^4 Where x is the molar solubility (assuming no common ion effect)

Constant-Pressure and Constant-Volume Calorimetry

Measuring heat transfer = calorimetry (include constant-pressure calorimetry (coffee cup) or constant volume (bomb calorimeter) Constant-Pressure calorimeter: Coffee cup calorimeter: insulated container covered with lid filled with liquid with reaction occurring *Atmopsheric pressure stays constant, temp can be measured (no gain or loss of heat to environment) Bomb calorimeter/ Decomposition vessel: Constant volume calorimetry: W = P Delta V, no work is done (Isovolumetric process where delta V = 0) so W = 0 -Calorimeter isolated -Heats of certain reaction like combustion can be measured indirectly by assessing temp change in water bath around vessel -System = sample plus O2a nd steel vessel, surroundings = water -No heat exchange to rest of universe so delta Q = 0 Delta U (change in internal energy of system) + delta U of surroundings = Delta U of calorimeter = Q of calorimeter - W of calorimeter = 0 so Delta U of system = - Delta U of surroundings q of system = - q of surroundings (q of cold = - q of hot) *Image shown is bomb calorimeter

Metals, non-metals, and metalloids

Metals: left side and middle (active, transition, lanthanide and actinide series) *actinide series = the series of chemical elements atomic numbered 89-103 and falling between s and d blocks -Lustrous/shiny solids (except for mercury) -Usually high melting points and densities (lithium) -Can be deformed without breaking (malleable (the ability to shape a material with a hammer) and ductile (can be pulled into wires)) -the most significant property that contributes to the ability of metals to conduct electricity is the fact that they have valence electrons that can move freely. -low effective nuclear charge/low electronegativity, large atomic radius (neutral element), small ionic radius (size of ion), low ionization energy, low electron affinity -easily lose electrons -Good conductors -Transition metals can have more than one oxidation state (charges when forming bonds) -Some transition metals can be relatively non reactive Non-Metals: Mostly upper right side -Generally brittle, solid, no luster, high ionization energy, electron affinity, and electronegative -small atomic radii and large ionic radii -Poor conductors Metalloids/semi-metals: Stair step group -Characteristics vary based on what they bond to -B, Si, As, Te, At, Po, Sb, Ge

The solubility of Fe(OH)3 in aqueous solution was determined to be 4.0 x 10^-10 mol/L. what is the value of the Ksp for Fe(OH)3 at same temp and pressure?

Molar solubility = 4.0 x 10^-10 M The dissociation reaction is: Fe(OH)3 (s) <-> Fe 3+ (aq) + 3OH- (aq) For every mole of Fe(OH)3 that dissociate, one mole of Fe 3+ and three moles of OH- are produced. Ksp = [Fe 3+] [OH-] ^3 Ksp = [4.0 x 10^-10 M] [3 * 4.0 x 10^-10 M] ^3 3^3 * (4.0 x 10^-10 M) ^4 = about 7.5 X 10^-37

Determine Normality (Looking at H+) 0.25 M H3PO4 95 g of PO4^3- in 100 mL solution

Molarity = Normality/ n N= 0.25 X 3 = 0.75 First convert grams to moles Molecular mass = 95 95 g/100mL = 950g/ 1L = 10 mol/ 1L = 10 M PO4^3- 10M PO4^3- X 3 equivalents of H+/ mol of PO4^3- = 30 N PO4^3-

Empirical Formula VS Molecular Formula What are the empirical and molecular formulas of a carbohydrate that contains 40.9% carbon, 4.58% hydrogen, and 54.52% oxygen and has a molar mass of 264 g/mol? (assume 100g sample)

Molecular = the actual number of atoms in a compound (C6H6) Empirical = basic ratio, smallest whole numbers of atoms in a compound (CH) *Empirical formula of CH2O is indicative of monosaccharide Moles C = 40.9 g/ (12 g/mol) = 3.4 mol Moles H = 4.58 g/ 1 g/mol = 4.6 mol Moles O = 54.52 g/ 16 g/mol = 3.4 Divide all by 3.4 (smallest number out of all) ( C1 H 1.33333 O1 ) X 3 = C3H4O3 To determine molecular formula, divide molar mass (264 g/mol) by formula weight of empirical formula (88g/mol) = 3 Empirical: C3H4O3 Molecular: C9H12O9 Method 2: When molar mass is given, easier to find molecular formula first -Multiply molar mass by % to find mass of each element present in 1 mole, then divide by atomic weights Moles C = (.409)(264g)/ (12g/mol) = about 9 mol Moles H = (0.0458)(264g)/ 1g/mol = about 13 mol (rounding error, should be 12) Moles O = )0.5452)(264)/ 16 g/mol = about 9 mol The molecular is same as empirical formula or a multiple of it. To calculate MF, you need to know mole ratio (to give you EF), and the molar mass (molar mass divided by empirical formula weight will give multiplier for empirical formula to molecular formula conversion)

Acid-base nomenclature

Mose acids are named from their parent anions ( the anion that combines with H+ to form the acid). Acids formed from anions with names in -ide have the prefix hydro- and the ending -ic. (F- = fluoride, HF = hydrofluoric acid) *Acids formed from oxyanions are called oxyacids. If anion ends in -ite (less oxygen), then the acid will end with -ous acid. If the anion ends in -ate (more oxygen), then the acid will end with -ic acid Acids ending in -ic are derivatives of anions ending in -ate, while acids ending in -ous are derivatives of anions ending in -ite. ClO3- is chlorate because it has more oxygen than the other commonly occurring ion, ClO2-, which is named chlorite. Therefore, HClO3 is chloric acid. HClO2 represents chlorous acid. HClO represents hypochlorous acid. *some exceptions, like MnO4- is called permanganate even though there are no MnO3- or MnO2- IO4^- has acid formula of HIO4 and acid name of periodic acid I- has acid form of HI and named Hydroiodic acid

Half-Reaction Method (Ion-Electron Method) Balance the following redox reaction using half-reaction method: Mg (s) + HNO3 (aq) -> Mg2+ (aq) + NO (g)

Most common method of balancing redox reactions Equation separated into 2 half reactions, oxidation part and reduction part (balanced separately) Mg (s) + HNO3 (aq) -> Mg2+ (aq) + NO (g) Mg -> Mg2+ + 2e- HNO3 + 3H+ + 3e- -> NO + 2H2O Balance electrons 3(Mg -> Mg2+ + 2e-) 2(HNO3 + 3H+ + 3e- -> NO + 2H2O) Cancel out 3Mg + 2HNO3 + 6H+ -> 3Mg2+ + 2NO + 4H2O

Aqueous Solutions (aq)

Most common type of solution Solvent is water Formation of hydronium ion (H3O+) can occur from transfer of proton H+ NEVER found alone in solution becuase free proton is difficult to isolate, so its bonded to electron pair donor (carrier) molecule like water (coordinate covalent bond) 7 general solubility rules for aqueous solutions: *1. All salts containing ammonium (NH₄⁺) and alkali metal cations (Group 1) are water-soluble.* *2. All salts containing nitrate (NO₃⁻) and acetone (CH₃COO⁻) are water-soluble.* 3. Halides (not F) are water soluble (except when bound to Ag⁺, Pb²⁺, Hg²⁺) 4. Salts with sulfate (SO₄²⁻) are soluble (not when bound to Ca²⁺, Sr²⁺, Ba²⁺, Pb²⁺). 5. Metal-oxides are insoluble (except *alkali, ammonium*, CrO, SrO, BaO). 6. Hydroxides are insoluble (except *alkali, ammonium*, Cr²⁺, Sr²⁺, Ba²⁺). 7. Carbonates (CO3 ^2-), phosphates (PO4 ^3-), sulfides (S^2-), and sulfites (SO3^2-) are insoluble (except alkali metals, ammonium). Most important: All salts of group 1 metals and all nitrate salts are soluble *Sodium ion concentration does not affect pH *The only time you need to worry about nitrate ion concentration is oxidation-reduction reaction, for which nitrate ion can weakly function as oxidizing agent

What are the pH and pOH of a solution containing 5 mL of 5M benzoic acid (Ka = 6.3x10^-5) and 100 mL of .005M benzoate solution?

N1V1 = N2V2 N2 = N1V1/V2 Normality same as molarity for Benzoic acid (monoprotic): N2 = 5mL * 5N / 105 mL For Benzoate solution: [A-] N2 = .005 * 100 / 105 pH = pKa + log [A- / HA] A- = conjugate base = Benzoate solution HA = weak acid = benzoic acid pH = pKa + log [0.5 / 25] pKa = 5 - 0.63 = about 4.37 pH = 4.37 + log (2X10^-2) 4.37 - 1.8 = About 2.57 = pH pOH = 14 - 2.57 = about 11.5

Ionic Bonding

One or more electrons from an atom with low ionization energy (typically metal) are transferred to atom with high electron affinity (typically nonmetal) -NaCl -Results from electrostatic attraction/force between opposite charges is what holds ions together -Creates crystalline lattice structures (as a solid) consisting of repeating rows of cations and anions, rather than individual molecular bonds -Tend to form between atoms that have significantly different electronegativities (atom that loses electrons (typically metals) becomes cation, atom that gains becomes anion) -Electrons NOT shared -For electron transfer to occur, difference in electronegativity must be greater than 1.7 on Pauling scale ("Stronger" atom gets electrons) -Strong electrostatic force = high melting and boiling points -Many dissolve readily in water/polar solvents, good conductors in liquid/gas state -Long bond distance = more weakly held together

Percent Composition of an element (by mass) What is the percent composition of chromium in K2Cr2O7?

Percent of specific compound that is made up of given element Percent Composition = Mass of element in formula / molar mass X 100% Molar mass of K2Cr2O7: (2X39.1 g/mol) + (2X52 g/mol) + (7X 16.0 g/mol) =2924.2 g/mol (estimated 292) Percent composition = (2X52)/ 294.2 is about (2X50)/300 .33 -> 33% Actual value = 35.4%

What is the pH of a solution with [HClO4] = 10 M

Perichloric acid is a strong acids o it fully dissociates. [H+] = 10M -log (10) = -1 This points out that the pH scale doesn't end at 0 and 14. There can be negative pH values and ones bigger than 14, implying a very high concentration of a very strong acid or base

Periodic Table

Periodic Law: Chemical and physical properties of the elements are dependent in a periodic way, upon their atomic numbers/ ARRANGED IN ORDER OF ATOMIC NUMBER Periods/Rows: principal quantum number (n, m, ms, ml)(7 periods for n=1 to n=7) Groups/columns/families: Valence electron configuration (all in column have same # of VE) Roman numeral above each group represents VE A and B beside Roman numerals (not used anymore really): A = representative elements (VE in either s or p subshells), and B = non-representative elements (transition elements with VE in s and d subshells) and La and Ac series (with VE in s and f subshells) Alkali Metals (1A): lower density, low Zeff, largest atomic radii, low ionization energy, electron affinity, and electronegativity, form univalent cations Oxidation state of +1 Alkaline Earth Metals (11A): slightly higher effective nuclear charge, slightly smaller atomic radii, form divalent cations 1A and 11A together are called active metals Oxidation state of +2 Chalcogens (V1A/ Group 16): Nonmetals and metalloids, 6 VE electrons, small atomic radii and large ionic radii, includes Oxygen, S, Se -Oxidation state of -2 or +6 in order to achieve noble gas Halogens (Group V11A/ 17): Highly reactive nonmetals with 7 VE, very highly electronegative, high electron affinities, usually found as halides (ions) or diatomic molecules (Cl2) -Typically oxidation state of -1 Noble Gases (VIIIA/18): Inert gases, high ionization energy, no electronegativity/ E affinity, gases at room temp, very low BP, not reactive Transition Metals (B) Groups IB to VIIIB or 3-12 low ionization energy, electron affinity, and electronegativity Usually hard with high BP and MP, malleable, can have different possible charged forms or oxidation states because they can lose different numbers of electrons Tend to associate with hydration complexes or with nonmetals -> colorful complexes

Galvanic (Voltaic) Cell

Positive EMF (ECell > 0), which means that rxn is spontaneous since Delta G = -nFE Ecell>0 and delta G<0 RED CAT: Reduction always occurs at the cathode AN OX: Oxidation occurs at anode Commonly used as batteries (non rechargeable batteries) Must be spontaneous (decreasing/negative free energy: Delta G< 0 as the cell release energy to environment) Because free energy is negative, electromotive force (Ecell) is positive (always have opposite signs) 2 electrodes of distinct chemical identity are placed in separate compartments (half-cells) and connected by conductive material (like copper wire) Surrounding each electrode is aqueous electrolyte solution composed of anions and cations Salt bridge (inert salt, like KCl or NH4NO3) connects 2 solutions: Exchanges anions and cations to balance or dissipate newly generated charges *Daniell cell (image shown): zinc electrode placed in aqueous ZnSO4 solution and copper electrode placed in aqueous CuSO4 solution *Anode is Zinc bar *Cathode is copper bar (site of reduction from Cu2+ to Cu) *anions from salt bridge diffuse into anion side (ZnSO4) to balance charge of new Zn2+ ions and the cations of the salt bridge (K+) flow into solution on cathode side (CuSO4) to balance charge of sulfate ions left when Cu2+ reduced to Cu *Precipitation process onto the cathode can be called plating or galvanization During the course of reaction electrons flow from zinc anode through wire to copper cathode *anions flow externally from salt bridge onto ZnSO4 and cations (like K+) flow externally from salt bridge onto CuSO4 -> this depletes salt bridge Depletion of salt bridge and finite amount of CuSO4 = short cell lifespan *ELECTRONS FLOW FROM ANODE TO CATHODE (think alphabetical order: from A to C)(current flows from cathode to anode) Rules for constructing cell diagram 1. Reactant and products listed from left to right in this form: anode | anode solution (concentration || cathode solution (concentration) | cathode 2. Single line means phase boundary 3. Double line means presence of salt bridge or type of barrier

Moles How many moles are in 9.53 of MgCl2

Quantity of any substance equal to the number of particles that are found in 12 grams of carbon-12 Avogasdro's number (NA) = 6.022X10^23 mol^-1 1 mol = mass (grams) equal to molecular/formula weight in amu Molecule is to amu as moles is to grams Mass of one mole of a compound is called molar mass (g/mol) Molecular weight is measured in amu/molecule Moles = Mass of sample (g) / Molar mass (g/mol) MgCl2 = 24.3 g/mol + (2X 35.5 g/mol) = 95.3 g/mol 9.53g/95.3 g/mol = 0.1 mol MgCl2

What is the change in vapor pressure when 180 grams of glyceraldehyde (C3H6O3) are added to 0.18 L of water at 100 deg C?

Raoult's Law: Vapor pressure of solvent with solutes = mole fraction X VP of solvent in pure state *vapor pressure of water at 100 deg C (boiling point) = 1 atm First, mole fraction 0.18 L = 180 mL = 180 grams of water / 18 g/mol = 10 moles 180 g / 90 grams per mole of glyceraldehyde = 2 moles Mole fraction = 10 moles water / 12 moles total = 0.83 Vapor pressure change: 0.83 X 1 atm = 0.83 1-0.83 = 0.17 atm

Density (p)

Ratio of mass per unit volume of a substance • For gases - g/L = m (mass)/ volume = pressure X molar mass (M) / (Rate constant X Temp)

Oxidation-Reduction Reaction

Reactions that involve transfer of electrons from one species to another (Law of conservation = charge neither created nor destroyed, so loss and gain of electrons must occur simultaneously) Oxidizing agent is reduced, reducing agent is oxidized Almost all oxidizing agents contain oxygen or strongly electronegative element (halogen), and reducing agents often have metal ions or hydrides Oxidation = loss of electrons, sometimes in form of hydrogen (dehydrogenation): Enzymes that catalyze oxidations are called dehydrogenases Other things besides enzymes can oxidize/reduce compounds (like hemoglobin) OIL-RIG: Oxidation is loss (of electrons), reduction is gain (of electrons) Common Oxidizing agents: O2, F2, Cl2, Br2, I2, H2SO4, HNO3, NaClO, KMnO4, CrO3, Na2Cr2O7, PCC, NAD+, FAD/FADH, H₂O₂ Common Reducing Agents: CO C B2H6 Sn2+ and other pure metals Hydrazine Zn(Hg) Lindlar's catalyst NaBH4 LiAlH4 NADH, FADH2

Amphoteric Species

Reacts like an acid in a basic environment and like a base in an acidic environment *In BL sense, amphoteric species can either gain or lose proton = amphiprotic (Species that may either gain or lose a proton) *Water is most common example: when it reacts with base it behaves like acid, but when it reacts with acid it behaves like a base *water, amino acids, and partially deprotonated polyphonic acids like bicarbonate and bisulfate are amphoteric and amphiprotic. Metal oxides and hydroxides (like Al, Zn, Pb, Cr) are amphoteric but not amphiprotic because they dont give off protons (amphiprotic are amphoteric species that can behave like Bronsted Lowry acid/base) *HSO4^- can either gain proton to form H2SO4 or lose to form SO4^2- *species that can be both oxidizing and reducing agent = amphoteric *Complex amphoteric molecules include amino acids that have zwidtterion intermediate with cationic and anionic character Examples: HCO3- + HBr -> H2CO3 + Br- The amphoteric reactant is HCO3- (the thing that is obviously not the acid) and it is also amphiprotic 3HCl + Al (OH)3 -> AlCl3 + 3 H2O The amphoteric reactant is Al)OH)3 but it is not amphoteric 2HBr + ZnO -> ZnBr2 + H2O The amphoteric reactant is ZnO but it is not amphoteric

Given that the standard reduction potential for Sm3+ and [RhCl6]^3- are -2.41 V and 0.44V, calculate emf of: Sm^3+ + Rh + 6Cl- -> [RhCl6]^3- + Sm

Rh is oxidized and Sm3+ is reduced (goes from 3+ to 0, gaining electrons) Sm3+ + 2e- -> Sm (Gaining electrons = reduction = cathode, and the fact that this is a more negative value means it is electrolytic) Rh + 6Cl- -> [RhCl6]^3- + 3e- (Losing electrons = oxidation = anode, and here, the anode is positive. That means that this is an electrolytic cell. If it were galvanic, the anode would be negative.) In either cell type, reduction is at cathode Cathode - anode: -2.41 - (0.44V) = -2.85 (this tells us its electrolytic because it is non-spontaneous. If it were galvanic cell the reaction would proceed spontaneously to left towards reactant (in which case Sm would be oxidized while [RhCl6]^3- would be reduced with emf of +2.85)

Free Energy Diagram

Shows relationship between activation energy, free energy of reaction, and freee energy of system +delta G = endergonic = energy absorbed -delta G = exergonic = energy given off Free energy change of the reaction (delta G rxn) is difference between free energy of products and reactants *negative free energy change = exergonic For exergonic reactions, the net energy change is negative, and the free energy of the final products is lower than the free energy of the initial reactants The difference in free energy between transition state and reactants is activation energy of forward reaction, difference in free energy between transition state and products is the activation energy of reverse reaction

Single Displacement and Double Displacement (Metathesis) Reactions

Single Displacement: Atom or ion replaced by atom/ion of another element -Often further classified into oxidation-reduction Cu (s) + AgNO3 (aq) -> Ag (s) + CuNO3 (aq) *IN this case, Ag is oxidized because it goes from +1 charge to 0, Oxygen stays -6 on both sides, N stays +5, Cu oxidized from 0 to +1 Double displacement/ Metathesis reactions: elements from two different compounds swap places to form two new compounds (switching of counterions) -One product removed from solution as precipitate or gas or when 2 of the original species combine to form weak electrolyte that remains undissociated in solution *not usually oxidation reduction reactions (all species retain the same oxidation number) Net ionic equation of AgNO3 (aq) + HCl (aq) -> HNO3 (aq) + AgCl (s) would be Ag+ + Cl- -> AgCl becuase AgCl is solid *In double displacement reactions where both reactants and products are aqueous, there is no net ionic reaction

Decomposition Reactions

Single reactant breaks down into 2 or more products, usually from heating, high-frequency, radiation, electrolysis (NH4)2Cr2O7 (aq) -> N2 (g) + Cr2O3 (s) + 4 H2O (g) On the right, N has -3 charge, H has +1, Cr has +6 and O = -2 On right, Cr = +3, O =. -2, H=+1 and O = -2 Half-Reactions: 2NH4^+ -> N2 + 8H+ Cr2O7 + 8H+ -> Cr2O3 + 4H2O Net Ionic: 2NH4^+ + Cr2O7^2- -> N2 + Cr2O3 + 4 H2O Nitrogen = reducing agent (oxidized from -3 to 0) and chromium is oxidizing agent (reduced from +6 to 3)

If 0.005 mol of NaOH is added to 0.5 L of the buffer solution (0.24 M NH3 and 0.20 M NH4Cl), what is resulting pH if pKa = 9.25 If 0.03 mol of HCl is added to 0.5 L of the buffer solution (0.24 M NH3 and 0.20 M NH4Cl), what is resulting pH

So if we add a base, we are reacting with the acid in our solution (NH4+) 0.005 mol / 0.5 L = 0.01 M [OH-] NH+ (.2 M) + OH- (0.1 M) -> H2O + NH3 (0.24) OH goes into ammonia, so subtract 0.01 from both, and add to NH3 because 0.01 added to product NH4+ = 0.19 M OH- = 0 NH3 = 0.25 (conjugate base) pH = 9.25 + log (025/0.19) = About 9.4 = pH Now what happens if we add acid: 0.03 mol/ 0.5 L = 0.06 M NH3 + H3O+ -> NH4+ + H2O 0.24 + 0.06 (all of this will get lost into the product) -> 0.20 0.18 + 0 -> 0.26 M pH = pKa + log [0.26/0.18] 9.25 - 0.16 = 9.09

Electrolytes

Solutes that enable solutions to carry currents *Contain equivalents of ions from molecules that dissociate in solution *Ionic compounds make good electrolytes because they dissolve most readily (non-polar covalent compounds are weakest) *pure water is a poor conductor Solid ionic compounds tend to be bad conductors, but in aqueous solution, lattice arrangement disrupted by ion-dipole interactions (ions free to move now) The tendency of an ionic solute to dissolve (solvate) in water can be high or low *Strength of electrolyte depends on degree of dissociation or salvation Strong electrolyte: dissociates completely into ions (NaCl and KI, highly polar covalent bonds (HCl)) Weak electrolyte: ionizes/hydrolyses incompletely (only some of the solute is dissolved)(Hg2I2 (very low Ksp), acetic acid (CH₃COOH)/weak acids, ammonia/ weak bases) *methylamine (CH3NH2) contains an alkyl group, which is electron-donating. This increases the electron density on the nitrogen in methylamine, making it a stronger (Lewis) base than ammonia. Nonelectrolytes: Dont ionize at all in water (non polar gases and organic compounds (O2, CO2, glucose))

Solvation/Solubiltiy VS Saturation

Solvation (reversible) refers to the breaking of intermolecular forces between solute particles and between solvent particles, with formation of intermolecular forces between solute and solent particles. In an aqueous solution, water is solvent Solubility is amount of solute contained in solvent Saturation refers to maximum solubility of compound at given temp, one cant dissolve anymore of the solute just by adding more at this temp Solubility = Maximum amount of substance/solute that can be dissolved in solvent at given temp Saturated = Max amount of solute has been added, dissolved solute in equilibrium with undissolved (add more and it won't dissolve Solvation tends toward equilibrium (lowest energy state of system under given conditions) -systems move spontaneously toward eq -At eq, neither dissolution nor precipitation is favored (free energy is zero) Saturation Point: Solute concentrations at its maximum value for given temp and pressures (rate of dissolution = rate of precipitation) After dissociation occurs, solute dissolved, then reverse process of precipitation of solute occurs When solution is dilute (unsaturated), dissolution is favored (initially, rate of dissolution greater than rate of precipitation ) As solution becomes more concentrated = approached saturation = rate of dissolution lessens and rate of precipitation increases Solubility of ion complexes determined by Ksp. -Depends on temp of solution, solvent, and in case of gas-phase solute, the pressure (also affected by adding other stuff)

Hydrogen Bonds

Specific, unusually strong form of dipole dipole -May be intra or intermolecular Not actual bonds: No sharing of lectrons When H bonded to either Nitrogen, oxygen or fluorine, it carries only small amount of electron density in covalent bond -> H acts as naked proton Super high BP Nucleotides stabilized by H bonds

Electromotive force (emf) (E*cell)

Standard reduction potentials used to calculate standard electromotive force, which is difference in potential voltage between 2 half-cells under standard conditions E*cell = E*(red, cathode) - E*(red, anode) *Number of moles doesn't matter because electrode potential doesn't depend on size, but rather on identity THE STANDARD REDUCTION POTENTIAL OF AN ELECTRODE WILL NOT CHANGE UNLESS THE CHEMICAL IDENTITY OF THAT ELECTRODE IS CHANGED *You need to multiply each half reaction by common denominator to cancel out electrons

States and State Functions

State of system described by certain macroscopic properties (state functions: Describe system in equilibrium state, don't describe process, only useful for comparing one equilibrium state to another ) Process function/path function: Pathway taken from one eq to another described by process functions (Work (W) and heat (Q))(if you climb straight up a cliff, you are doing less work than someone who backslid several times not he way up, so the path matters) State functions include pressure (P), density (p), temp (T), volume (V), enthalpy (H), internal energy (U), Gibbs free energy (G), and entropy (S) *Independent of path/process taken, but not necessarily independent of eachother Standard conditions for measuring enthalpy, entropy, and Gibbs FE (used for kinetics, eq, and thermodynamics): 25 deg C (298 K), 1 atm, 1M concentration *Don't confuse with STP, which has temp at 0 deg C (273 K) and 1 atm: Used for ideal gas Under standard conditions, most stable form = standard state (changes that occur in this state are standard enthalpy (ΔH°), standard entropy (ΔS°), and standard free energy changes (ΔG°)

Thermodynamics

Study of energy changes that accompany chemical and physical processes Zeroth Law of thermodynamics: Objects are in thermal eq only when temps are equal 1st law: Energy neither created nor destroyed (but changed from one form to another) Delta U = Q - W where U is change in internal energy of system, Q is heat added, and W is work done 2nd law: Energy spontaneously disperses from being localized to becoming spread out if its not hindered from doing so 4 Processes: Isothermal (constant temp), adiabatic (no heat exchange), isobaric (constant pressure, isovolumetric (isochoric)(constant volume) 3rd law: There is a finite limit to temp below which nothing can exist (determined Kelvin scale: No temp below 0 K because system unable to lose anymore heat energy)

Heat vs. Temperature

Temp is a scaled measure of average kinetic energy of a substance Heat is the transfer of energy that results from differences of temp between two substances Heat is the total energy of molecular motion in a substance while temperature is a measure of the average energy of molecular motion in a substance. Heat energy depends on the speed of the particles, the number of particles (the size or mass), and the type of particles in an object. Temperature does not depend on the size or type of object. For example, the temperature of a small cup of water might be the same as the temperature of a large tub of water, but the tub of water has more heat because it has more water and thus more total thermal energy. It is heat that will increase or decrease the temperature. If we add heat, the temperature will become higher. If we remove heat the temperature will become lower. Higher temperatures mean that the molecules are moving, vibrating and rotating with more energy. Heat is a specific form of energy that can enter or leave a system. (process function) Temperature is an indirect measure of the average kinetic energy of the particles in a system. (scale of how hot or cold something is)(related to thermal energy (enthalpy) ) Heat (Q) is transfer of thermal energy from one substance to another as a result of differences in temp *Zeroth Law of thermodynamics: Objects are in thermal eq only when temps are equal, so heat is process function (define path between eq functions), not state function (describes equilibrium state): We can quantify how much thermal energy is transferred between two or more objects as a result of their difference in temps by measuring heat transfer Heat and work are measured independently Endothermic: System absorbs heat (Delta Q >0): Exothermic: System releases heat (Delta Q < 0) Unit of heat/energy = Joule (J) or calorie (1 cal = 4.184 J) Enthalpy (Delta H)(state function) = heat (Q) under constant pressure *change in enthalpy is equal to heat transferred into/out of system at constant pressure *Delta H (enthalpy can't be measured directly, only change in enthalpy, only for certain fast and spontaneous reactions) = H products - H reactants (Positive Delta H = endothermic process, negative = exothermic) Energy moves from warmer substance hot cooler substance Measuring heat transfer = calorimetry (include constant-pressure calorimetry (coffee cup) or constant volume (bomb calorimeter) Heat (q) absorbed or released = mc (delta T) where m is mass, c is specific heat Specific heat = amount of energy required to raise temp of one gram of substance by one degree (Specific heat of H2O = 1 cal/ g*K) *MC (mass time specific heat) = heat capacity Note: Specific heat (c) is energy required to raise temp of 1 gram by 1 degree C. Heat capacity (mc) is the product of mass and c and is energy required to raise any given amount of a substance by 1 degree Evaporation of sweat = endothermic (energy absorbed from body for liquid articles to gain enough energy to become gas)

Concentrations Percent Composition by Mass, Mole Fraction, Molarity, and Normality

The amount of solute dissolved in solvent Can be expressed in volume % (solute/solution), units of degrees Brix (AKA mass percent: mass of glucose / mass of solution X 100), percent composition by mass, mole fraction, molarity, and normality Percent composition by mass: mass of solute/ mass of solution X 100 Used for aqueous solution and metal alloys and other solid-in-solid solutions Mole Fraction (X): XA = moles of A / Total moles of all species -Sum of mole fractions in system will always = 1 -Used to calculate vapor pressure depression of solution or partial pressure of gases Molarity (M): moles of solute/ liters of solution (solution volume, not the volume of solvent used to prepare solution) * If it is in brackets, like [Na+], its molarity *For dilute solutions, volume of solution usually about = to volume of solvent *Super common used for pH, law of mass action, osmotic pressure.... Molality (m): moles of solute/kilograms of solvent *For dilute aqueous solutions at 25 deg C, mloarityk = molality becuase water density = 1 Kg/liter -Used in boiling point and freezing point depression *For water, molarity and molality are nearly = at room temp because 1 L of solution = 1 Kg of solvent because density of water = 1 g/mL Normality (N): number of equivalents of interest per liter of solution *an equivalent is measure of reactive capacity of molecule *Molarity of species of interest *Equal to a mole of the species of interest (protons, ions, electrons...) *Reaction dependent The easiest way to find normality is from molarity. All you need to know are how many mole of ions dissociate. For example, a 1 M sulfuric acid (H2SO4) is 2 N for acid-base reactions because each mole of sulfuric acid provides 2 moles of H+ ions. 1 M sulfuric acid is 1 N for sulfate precipitation since 1 mole of sulfuric acid provides 1 mole of sulfate ions. *In acid base reactions, look at concentration of hydrogen ions *In oxidation-reduction reactions, look for concentration of electrons

Given the two half reactions below, what would be the spontaneous oxidation-reduction reaction between these two species? Fe3+ + 3e --> Fe; -0.036V (Reduced because it gains electrons, happens at cathode) I3- + 2e --> 3I- ; +.534V (oxidized because it loses electrons, happens at anode) Note: Spontaneous tells me that it is galvanic, which tells me that the anode is negative

The reduction potential of triiodide is higher than ion (III), so triiodide will be reduced and iron will be oxidized 2 Fe + 3I3^- -> 2Fe^3+ + 9I- Cathode - anode = 0.534 - (-0.036 V) E cell = +0.57 V

Henry's Law

The solubility of a gas will increase with increasing partial pressure Characteristic of gas's vapor pressure: pressure exerted by evaporated particles above the surface of liquid Evaporation = dynamic process that requires molecules at surface to gain enough energy to escape into gas phase. Vapor pressure from evaporated molecules forces some gas back to liquid, and equilibrium reached between evaporation and condensation [A] = kH X PA [A]1/ P1 = [A]2/P2 = kH where [A] is concentration of A in solution, kH is Henry's constant (depends on identity of gas), and PA is partial pressure of A

Common Ion Effect

The solubility of a salt is considered reduced when it is dissolved in a solution that already contains one of its constituent ions as compared to its solubility in a pure solvent This reduction in molar solubility of a compound is called the common ion effect Molar solubility of a compound is its concentration (moles per liter) at equilibrium at constant temp If X mols of AmBn (s) can be dissolved in one liter of solution to reach saturation, then the molar solubility of AmBn (s) is X molar Common ion presence results in reduction in molar solubility of salt (no effect on value of solubility product constant ) *For example, if a salt like CaF2 is dissolved into water already containing Ca2+ ions (perhaps from CaCl2), the solution will dissolve less CaF2 than would an equal amount in pure water Really just Le Chateleirs principle in action System shifts towards left side (opposite of complex ion, which shifts to right), reforming solid salt (molar solubility reduced, less solid dissolves, but Ksp remains constant) *Decreases dissociation Can be used to separate out specific compounds in solution mixture *in solution of silver salts, you could add NaCl or KCl to preferentially precipitate silver chloride. By adding an appropriate counterion in excess, the dissociation reaction shifts to left, forming solid salt Ksp of AgI = 8.5 X 10^-17. If 1X10^-5 M solution of AgNO3 is saturated with AgI , final concentration of iodide ion is...... concentration of Ag+ in original AgNO3 solution = 1X10^-5 M Some small amount of AgI will dissociate (x) Net silver concentration = 1X10^-5 M + x Concentration of iodide = x (because none was present in solution until AgI) Ksp = [Ag+][I-] 8.5 X 10^-17 = [1X10^-5 M + x ][x] since ksp is so tiny, only tiny amount of AgI dissociated, so when added to 1X10^-5 M, its negligible 8.5 X 10^-17 = [1X10^-5 M][x] x = 8.5 X 10^-12 M (concentration of iodide)

To form a cation, a metal must ____ electrons. Therefore, metals like to get _________ (Oxidized or reduced) and act as good _________ agents. Nonmetals like to get ______ (Oxidized or reduced) (_______ electrons) and act as good __________ agents

To form a cation, a metal must lose electrons. Therefore, metals like to get Oxidized and act as good reducing agents. Nonmetals like to get reduced) (gain electrons) and act as good oxidizing agents

Combination Reaction

Two or more reactants forming one product H2 (g) + F2 (g) -> 2 HF (aq) On left, both H and F have 0 charge, on right H = +1 and F = -1 Relevant half reactions: H2 -> 2H+ + 2e- F2 + 2e- -> 2F- Net ionic equation: H2 + F2 -> 2H+ + 2F- Hydrogen = reducing agent (it is oxidized, charge gets bigger because it loses electrons) and F is oxidizing agent No spectator ion

VSEPR Theory: 5 most common electron configurations of molecules NH3, CH4, and H2O all have tetrahedral electronic structure, but differ in their molecule shape: NH3 is _____, CH4 is ______ and H2O is ______ Shape of MgF2, PCl5, and SF6:

Uses Lewis dot structures to predict molecular geometry of covalently bonded molecules -Determined by repulsion between bonding and non-bonding E- pairs in valence shell of central atom -NH3: Trigonal pyramidal because lone pair on N repels 3 bonding E- pairs Electron geometry: spatial arrangement of all pairs of electrons around central atom (bonding and lone pairs) (Important to determine ideal bond angel because non-bonding pairs can exert more repulsion than bonding pairs) Molecular geometry: Only describes spatial arrangement of bonding pairs (look at coordination number = number of atoms that are bound to a central atom) NH3, CH4, and H2O all have tetrahedral electronic structure, but differ in their molecule shape: NH3 is trigonal pyramidal, CH4 is tetrahedral, and H2O is bent MgF2 = linear PCl5 = Trigonal bipyrimidal (90 degrees, 120, 180) SF6 = Octahedral (90, 180)

Gases

Very weak intermolecular forces exist between gas particles Large intermolecular distances Can fill any volume, compressible (differs from liquid) State of gaseous sample defined by four variables: Pressure (P), volume (V), temperature (T), and moles (n) Gas pressure in units of atmospheres (atm) or millimeters of mercury (mmHg) = torr SI unit of pressure = pascal (Pa) 1 atm = 760 mmHg = 760 torr = 101.325 kPa BP measured in mmHg by sphygmomanometer (high BP = systolic 140< and low = >90) You can continuously monitor BP using barometer Mercury in barometer rises becuase atmospheric pressure creates downward force while mercury in column exerts opposing force (weight) based on its density When external air creates higher force than weight of mercury in column, column rises Volume of gas express in liters (L) or mL Temp usually in Kelvins

Dalton's Law of Partial Pressures

When two or more gases that don't interact are in one vessel, each will behave independently (acts like its the only gas in the container) Pressure exerted by each gas = pressure that gas would exert if it were the only one container Pressure exerted by individual gas = partial pressure Dalton's Law of Partial Pressures: total pressure of gaseous mixture is equal to sum of partial pressures of the individual components PT = PA + PB + PC +....... where PT is total pressure Partial pressure of each gas is related to its mole fraction PA = XA * PT where XA = moles of gas A/ total moles of gas

Oxidation-Reduction Titration

Whereas acid-base follows movement of protons, oxidation reduction follows charge transfer *can use color indicators responding to certain voltage value *starch indicator with iodine complex *Potentiometric titration: No indicator used, instead electrical potential difference (voltage) is measured using voltmeter Voltmeter measures emf, potentiometer is kind of voltmeter that draws no current and gives more accurate reading of difference in potential between 2 electrodes

Electrolytic Cells

Whereas galvanic cells house spontaneous oxidation-reduction reactions that generate electrical energy, electrolytic cells house non-spontaneous reactions that require the input of energy to proceed (drive reaction in direction that is thermodynamically unfavorable) Positive change in free energy : Driven by external voltage source (electrolysis) in which chemical compounds are decomposed Because it is non-spontaneous, electrode (anode or cathode) can consist of any material so long as it can resist high temp and corrosion *Molten NaCl: Na+ ion migrate to cathode (reduced to liquid Na) and Cl- ions migrate to anode (oxidized to Cl2 gas) *According to the equation ΔG° = -RT ln Keq, Keq < 1 would result in ln Keq < 0, which means ΔG° > 0. Faraday: First to define certain quantitative principles governing behavior of electrolytic cells: said that liberation of gas and deposition of elements on electrodes is directly proportional to # of electrons transferred *amount chemical charge induced is proportional to number of moles of electrons exchanged: M^n+ + ne- -> M (s) n = number of electrons transferred per atom M One mole of metal M (s) will be produced if n moles of electrons are supplied to one mole of M^n+ *Number of moles of electrons needed to produce amount of M (s) can be related to electrical property of charge (1 electron carries 1.6X10^-19 Coulombs (C) charge) *Faraday's Constant: Amount of charge contained in one mole of electrons 1.6X10^19 C per electron, and 6.02X10^23 electrons per mol, so F = 96485 C/ mol e- 1F = 96485 C (usually rounded to 10^5) Electrodeposition equation: tells number of moles of element deposited on plate: mol M = It / nF where M = amount of metal ion deposited, I = current, t = time (sec) and n is number of electron equivalents for specific metal ion, and F= Faraday's constant *Equation can also be used to determine amount of gas liberated

Important Electrochemistry Equations: Standard change in free energy from equilibrium constant: ΔGº = -RTln(Keq) = -nFE°cell Free energy change for Non-standard conditions: ΔG = ΔG° + RT*lnQ There are only two equations involving standard change in free energy in electrochemical cells: ΔG° = -nFE°cell and ΔG° = -RT ln Keq Rearranged: E°cell = RT/nF * lnKeq Rearranged: ΔG° = nF(E°red, anode - E° red, cathode)

Whether it is log or ln, remember that log will be positive when equilibrium constants are greater than 1, and negative when equilibrium constants are less than 1, and 0 the eq constants are = 1 If E°cell is positive, than lnKeq is positive (Keq >1) and equilibrium lies to the right (products favored) (galvanic cells) Redox reactions with eq constants less than 1 (favors reactants), E°cell will be negative (natural log of decimal is negative) -This is characteristic of electrolytic cells (non-spontaneous) If eq constant = 1, E°cell will be 0 (no net ionic equation because both half-cells contain the same ions) If Q/Keq greater than 1, (Q> Keq) then ln will be positive and free energy will be positive So, change in Gibbs free energy of electrochemical cell with varying concentrations can be derived from: ΔG = ΔG° + RT*lnQ where ΔG = free energy change under non-standard conditions ΔG° = free energy change under standard conditions

A stock solution for making typical IV saline bags contains 90.0 g of NaCl per 10 liters (density = 1g/mL). What is mole fraction and percent composition by mass of NaCl in saline solution?

X of NaCl = moles of NaCl / total moles = (90g / 58.5g/mol) / ((90g / 58.5g/mol)+(10^4g / 18 g/mol)) = 1.5 / 500 = about 3X10^-3 Percent composition = mass of solute/ mass of solution X 100% = 90g / 90g + 10^4 g X 100% = 90/10^4 X 100% = 0.9% *10 liter * 1 gram /.001 mL = 10000L

Reaction Orders

Zero Order: rate of formation of product C is independent of changes in A or B (reactants) Rate = k[A]^0[B]^0 = k *K has units of Molarity/s *K can be changed by temperature or by adding catalyst = lower Ea = higher K *Slope is opposite of rate constant (Linear line down) First Order: Rate directly proportional to one reactant -Doubling one reactant double product -Rate = k[A]^1 *K has units of s^-1 -Radioactive decay -Reaction begins when molecule undergoes chemical change by itself (no interaction) *Image shown is kinetics of first order reaction, on the right rate constant is opposite of slope of graph of ln [A] vs time (k= - slope) Second Order reactions -Rate is proportional to either concentration of two reactants or to square of concentration of single reactant *A second-order reaction can be second-order with respect to one reactant, or first-order with respect to two different reactants. Rate = k[A]^1*[B]^1= k [B]^2 where k as units of M^-1*s^-1 -Suggests physical collision *First graph looks about the same as first order, but for 2nd order, rate constant is equal to slope of graph of 1/[A] vs time (k = slope, upward line) Higher order reactions: Rare that three particles collide simultaneously NOTE: Slope =...... 0 order: slope of concentrations time = -k 1st order: slope of ln[A] vs time = -k 2nd order: slope of 1/[A] vs time = k

According to the following standards REDUCTION potentials: Zn^(2+)(aq)+2e- → Zn(s) E = -0.76 and Ag+(aq)+e- → Ag(s) E = +0.80 What is the cell potential of the rxn Zn(s) + 2 Ag + (aq) -> Zn ^ 2+ (aq) + 2 Ag (s)

Zinc solid is oxidized, and Ag+ is reduced. Since we are given reduction potential values, the value of zinc needs to be revered to = +0.76 Reduction: To add electrons/hydrogen or to remove oxygen (Rxn goes from Ag+ to Ag so it gains electron) Oxidation: To increase the valence (the positive charge) of an element by removing electrons. Add standard reduction potential + standard oxidation potential E = 0.76 + 0.8 E = 1.56 V

Net Ionic Equation

Zn (s) + CuSO4 (aq) -> Cu (s) + ZnSO4 (aq) Split into various species gives us complete ionic equation: Zn (s) + Cu2+ (aq) SO4^2- (aq) -> Cu (s) + Zn2+ (aq) + SO4^2- (aq) Sulfate ion doesn't change (Spectator ion) -> we can take it out *Look for compounds like polyatomic anions that retain charge before and after reactions, these are usually spectator ions and won't be found in net ionic equation Zn (s) + Cu2+ (aq) -> Cu (s) + Zn^2+ (aq) All aqueous compounds should be split but solid salts should stay together

Decompisition reaction

a reaction in which a single compound breaks down to form two or more simpler substances

Neutralization Reaction

a reaction in which an acid and a base react in an aqueous solution to produce a salt and water

Ion Product (IP)

analogous to the reaction quotient (Q) for other chemical reactions Determines where system is with respect to eq IP = [A^(n+)]^m * [B^(m-)]^n (where the concentrations are the of the ionic constituents at a given moment in time, not necessarily at equilibrium) Each salt has its own distinct Ksp at given temp

Strong Acids and Bases

completely dissociate into their component ions in aqueous solutions Generally indicated by single headed arrow, generally indicate strong acids or bases (complete dissociation, no reversibility) NaOH (s) -> Na+ (aq) + OH - (aq) 1 M of NaOH yields 1 M Na+ and 1 M of OH- pH = 14 - pOH = 14 - (-log [OH-]) = 14 + log (1) = 14 We assume concentration of OH- from autoionization of water is negligible due to addition of strong base *OH- and H+ contribution from water is negligible is concentration of acid or base is way bigger then 10^-7 M Said to go to completion pH of 1 X 10^-8 M solution of HCl pH = 14 - (-log [OH-]) Kw = [H3O+][OH-] = (x+ 1 X 10^-8 M)(x) = 10^-14 x = 9.5 X 10^-8 M, so H+ concentration = 1.05 X 10^-7 M log (1.05 X 10^-7 M) = really close to 7, so the pH is really close to 7, just a little below, which makes sense for a really diluted acidic solution Strong Acids: HCl (hydrochloric acid), Hbr (hydrobromic acid), HI (Hydroidic acid), H2SO4 (sulfuric acid), HNO3 (nitric acid), and HClO4 (perchloric acid) Strong bases: NaOH (sodium hydroxide), KOH (potassium hydroxide), other soluble hydroxides from group IA metals. Calculation of pH and pOH of strong acids and bases assumes complete dissociation of the acid or base in solution

Electrochemical Cells

contained systems in which oxidation-reduction reactions occur Three types: Galvanic cells (voltaic cells), electrolytic cells, and concentration cells *Also specific commercial cells (like Ni-Cd batteries) Galvanic and concentration cells are spontaneous, electrolytic cells are non spontaneous *Spontaneity indicated by delta G (Gibbs free energy change) All three contain electrodes where oxidation reduction reactions take place Electrodes in electrochemical cell: AN OX and a RED CAT the anode is the site of oxidation and the cathode is the site of reduction Electromotive force (emf): corresponds to voltage or electrical potential difference of cell. *The temperature of the solutions in the half-cells can alter the emf (E°cell is dependent upon the change in free energy of the system through the equation RT ln Keq = nFE°cell. The temperature, T, appears in this equation; thus, a change in temperature will impact the E°cell.) If EMF is positive, cell can release energy (Delta G<0, reaction is spontaneous) If EMF is negative, cell must absorb energy (Delta G> 0) which means it is non-spontaneous Movement of electrons in all three is from anode to cathode, and the current runs from cathode to anode (electrons more opposite to the flow of current)

Graham's Law of Diffusion and Effusion If neon gas travels at 400 m/s at given temp, calculate average speed of krypton at same temp?

diffusion rate1/diffusion rate2 = sqrt(molar mass2/molar mass1) r1/r2 = sqr rt of (M2/M1) *molar mass of oxygen gas = 32 g/mol! *Molar mass of Hydrogen gas = 2 g/mol Gas with molar mass 4X another will travel half as fast The movement of molecules from high concentration to low concentration through a medium (like air or water) is called diffusion Heavier gases diffuse more slowly than lighter ones due to averaging speeds More massive the gas particle = slower the average speed Effusion: flow of gas particles under pressure from one compartment to another through small opening (think of pleural effusion = fluid inters intrapleural space) *For 2 gases at same temp, rates of effusion are proportional to average speed Diffusion: When gases mix with one another Effusion: When a gas moves through small hole under pressure Both will be slower for larger molecules, both conditions use same equations If neon gas travels at 400 m/s at given temp, calculate average speed of krypton at same temp? 400 / r2 = sqr rt (84/20)

Ionic Radii

generalizations: 1. metal lose electrons and become positive 2. nonmetals gain electrons and become negative metalloids can go either way nonmetals closer to the metalloid line indicates that they require more electrons than other nonmetals (like Nobel gases in group VIIIA = 18) to achieve the noble gas electron configuration - these nonmetals gain electrons while the nucleus remains the same (becoming more negative) THUS HAVING A GREATER IONIC RADIUS of the anions, compared to those closer to noble gases metals closer to the metallonic line have to lose more electrons (while nucleus charge stays the same) to get the stable electron configuration -> so the ionic radius for metals decreases close to metalloids Losing electrons = smaller Gaining electrons = bigger cations - loose more electrons (more positive) the smaller the ionic radii (Li+ smaller than Li) anions - gain more electrons (more negative) the bigger the ionic radii (O2- larger than O) *purpose of ions in stoichiometry is to identify a states THE IONIC RADII OF ANIONS ARE LARGER THAN THE ASSOCIATED ATOMIC RADII, WHILE THE IONIC RADII OF CATIONS ARE SMALLER F- is larger than F K is larger than K+

Colligative Properties

physical properties of solutions that depend on the concentration of dissolved particles but not on their chemical identity (depends only on number of solute particles) *Vapor pressure depression, boiling point elevation, freezing point depression, and osmotic pressure Usually associated with dilute solutions VPD and BPE go hand in hand Lowering VP means higher temp is required to match atmospheric pressure, raising the boiling point *Low vapor pressure does not evaporate easily *Melting point depresses upon solute addition, Solute particles interfere with lattice formation, the highly organized state in which solid molecules align themselves. Colder-than-normal conditions are necessary to create the solid structure.

Le Chatelier's Principle 3 Types of Stresses: Changes in concentration, pressure/volume, and temperature

if a stress is applied to a system at equilibrium, the system shifts in the direction that relieves the stress (system temporarily moved out of equilibrium because partial pressures no longer in eq or because eq ratio has changed as a result of change in temp) Reaction responds by trying to reestablish equilibrium *Bicarbonate buffer system: In tissues, high concentration of CO2 = shift to right *In lungs, CO2 lost = shift to left (blowing off CO2/hyperventilation gets rid of excess H+) If reactants are added or products removed, Qc < Keq (forward direction, increasing Qc until Qc=Keq)= shift to right If reactants removed or products added, Qc>Keq (reverse direction, decreasing Qc) = shift to left *Always react in direction away from added species toward removed species Changes in pressure and volume: -Only chemical reactions that involve at least 1 gaseous species will be affected by changes in volume or pressure -Compression = volume decrease = pressure increase = increase in partial pressure of each gas in system = Not in equilibrium (Qp does not equal Keq) -System moves towards side with fewer moles of gas (increasing pressure decreases number of gas moles to decrease pressure) -Expanding volume = pressure decrease = move toward side with more moles -N2 (g) + 3H2 (g) = 2NH3 (g): left side has four moles and right side has 2, so pressure increase would cause shift to right, and pressure decrease means reverse reaction will occur Changes in temperature: -Change in Keq, not Qc or Qp because change is not immediate (Q doesn't change, Keq does) -New direction determined by enthalpy: Endothermic (delta H >0) = heat functions as reactant -Exothermic (Delta H<0), heat functions as product-Increasing temp of endothermic or decreasing temp of exothermic will shift to right -Decreasing temp of endothermic or increasing temp of exothermic shifts to left

Heisenberg uncertainty principle

it is impossible to simultaneously determine, with perfect accuracy, the momentum and the position of an electron (you can learn one or the other, but you cant determine position and momentum at exactly the same time) Error in one variable is increased by attempts to measure the other The limitations placed by the Heisenberg uncertainty principle are caused by limitations inherent in the measuring process: if a particle is moving, it has momentum, but trying to measure that momentum necessarily creates uncertainty in the position. Even if we had an exact definition of the meter, or perfect measuring devices, we still wouldn't be able to measure position and momentum simultaneously and exactly.

What mass of copper will be deposited in a Daniell cell if a current of 2A flows through the cell for 3 hours?

mol M = It/nF where F = Faraday's Constant: 96485 C/mol e- (about 10^5) I = 2A t = 3 hours = 10800 seconds n = A Daniell cell uses a copper electrode in copper sulfate (CuSO4), and because oxidation state off copper is +2 (because SO4 has -2 charge), n =2 22000/2X10^5 = 2.2X10^4 / 2 X 10^5 = 0.1 mol Cu Now determine mass 0.1 X 63.5 g/mol = about 6.35 g

Atomic Mass Unit (amu)/ Atomic mass

one twelfth the mass of a carbon-12 atom (1.66X10^-24 grams) Atomic mass is nearly equal to mass number (assume same thing) -Measured in AMU -Sum of protons and neutrons

Henderson-Hasselbalch equation

pH = pKa + log [A-]/[HA] Weak acid buffer solution Used to estimate the pH or pOH of buffer solution A- is concentration of conjugate base and HA is concentration of weak acid *When conjugate base = weak acid, pH = pKa because log (1) = 0 (this occurs at half equivalence points in titration and buffering capacity is optimal here For weak base buffer solution: pOH = pKb + log [B+]/[BOH] where B+ is concentration of conjugate acid and BOH is concentration of weak base Buffering capacity optimal at pOH when pOH = pKb when conjugate acid = weak base Really just rearranges dissociation constant equation of Ka = [H3o+][A-]/ HA by taking -log of both sides because -log (H3O+) = pH -log (Ka) = pKa

A cup with 100g of water at 300 K is mixed with 200.0 g of water at 450 K. How would you calculate the final temperature of the mixture assuming no heat is lost to the surroundings/ What is equilibrium temperature of the system (assume no boiling)

q of cold = - q of hot q=m⋅c⋅ΔT , where q - heat lost or gained m - the mass of the sample c - the specific heat of the substance ΔT - the change in temperature, defined as final temperature minus initial temperature m(hot)⋅c⋅-ΔT(hot) = m(cold)⋅c⋅ΔT(cold) We ca n use any value of c as long as they are equal, in this case c= 1 cal/g*K but we can ignore it Let's say that the final temperature is Tf. This means that you have 100g⋅(Tf−300)K=200g⋅(450-Tf)K 100 Tf - 30000 cal = 90000 cal - 200 Tf 300 Tf = 120000 Tf = 400 K

Ionization Energy (IE)/ Ionization Potential

the energy required to remove an electron from an atom or ion in its gaseous state -Increases from left to right and from bottom to top -Endothermic process (always requires input of heat) -Greater Zeff = closer VE to nucleus = more tightly bound = more difficult to remove electrons = higher ionization energy -Removal of first electron (first ionization energy) smaller than second (divalent cation) or third ionization energy -Groups 1 and 2 (active metals) have super low ionization energies (rarely neutral in nature) Mg to Mg+ = 738 kJ/mol (1st ionization energy Mg+ to Mg2+ = 1450 kJ/mol (results in noble like configuration, so next one will be WAY higher) Mg2+ to Mg3+ = 7730 kJ/mol Highest ionization energies = Nobel gases

Gibbs Free Energy in Electrochemical Cell

ΔG°cell = −nFE°cell (similar to W = qΔV in physics) Gibbs free energy determines spontaneity of reaction (change in amount of energy available in chemical system to do work)(if ΔG°cell is positive reaction is nonspontaneous, if ΔG°cell is negative reaction is spontaneous) *Work done is dependent on number of coulombs of charge transferred and energy available ΔG° = Standard change in free energy (in Joules) n = number of moles of electrons exchanged F = Faraday constant (in J/V) E°cell = standard emf Negative sign in equation because E°cell and ΔG°cell will always have opposite signs *Galvanic cells have negative ΔG°cell and positive Ecell *Electrolytic cells have positive ΔG°cell and negative Ecell values

Free Energy, Keq, and Q

ΔGº = -RTln(Keq) where R is ideal gas constant Greater Keq = More positive ln = more negative standard free energy change = more spontaneous free energy of a reaction at nonstandard conditions: ΔG = ΔG° + RT-ln Q. ΔG of reaction = ΔGº of reaction + RTln(Q) = RTln(Q/Keq) when not at equilibrium/ reaction is in progress. The use of Q indicates system not at equilibrium If Q/Keq greater than 1, (Q> Keq) then ln will be positive and free energy will be positive *Image: Catalysts alter kinetics but not equilibrium of free energy change

Freezing point depression

ΔTf=iKfm where Tf is increase in freezing point, I is Van't Hoff factor, Kf is proportionality constant, and m is molarity of solution *This equation gives you the number that you should subtract from the normal freezing point *Normal freeing point of water = 273 K Presence of solute particles in solution interferes with formation of lattice arrangement of solvent molecules associated with solid state = greater amount of energy must be removed = lower temp *adding solute lowers freezing point *Pure water freezes at 0 deg C, but for every mole of solute dissolved in 1 Kg of water, the freezing point is lowered by 1.86 deg C

*No calculator* -The valence electron in a lithium atom jumps from energy level n=2 to n=4. What is energy of transition in joules? eV? RH = 2.18 X 10^-18 J J/electron = 13.6 eV/electron If an electron emits 3eV of energy, what is the corresponding wavelength of the emitted photon 1eV = 1.60 X10^-19 J = 1.60 X10^-19 J/eV h = 6.626 X 10^-34 J*s Calculate the energy of a photon of wavelength 662 nm h=6.626 X 10^-34 J*s

-2 X 10^-18 [1/2^2 - 1/4^2] = -2X 10^-18 [4/16-1/16] (-2X 10^-18) X about 0.2 = -4 X 10^-19 J -13.6 [1/2^2 - 1/4^2] = -14 [3/16] = -(21/8) = -2.5 Both values are negative which makes since because energy is absorbed as e- goes from low to high (e- need energy to go up) 3eV(1.60X10^-19 J/eV) = 4.8 X10^-19 Wavelength = hc/E (6.626X10^-34 J*s)(3.00X10^8 m/s)/ 4.8 X10^-19 J (6.6X3)(10^-34 X 10^8)/4.8 X10^-19 = 6.6/1.6 X 10^ (-26+19) 4X10^-7 m = 400 nm E=hc/wavelength E = 6.626 X 10^-34 J*s X (3X10^8)/ (662X10^-9 m) (6.6X3)(10^-34 X 10^8) / (6.6 X 10^-7 m) 3.00X10^-19 J

SN1 Reaction

-2 steps formation of carbocation -rate depends on only 1 reactant

STP (standard temperature and pressure)

0 degrees celcius (273K) and 101.3 kPa (1 atm) Used for measuring characteristics of an ideal gas Not the same as standard state! At standard state, temp is 25 deg C (298K) STP is generally used for gas all calculations, standard state conditions (298K, 1 atm, 1M concentrations) are used when measuring standard enthalpy, entropy, free energy changes, and electrochemical cell voltage

Oxidation State Rules Example: What are oxidation numbers for Na2CO3?

1. Oxidation number of free electron is 0 (N2, P4, S8, He....) 2. Oxidation number for monatomic ion = charge of ion (Na+ = +1, Fe3+ = +3, Cl- = -1) 3. Oxidation number of group A (Column 1 (Alkali metals)) in compound = +1 and oxidation number of group IIA (alkali earth metals) in compound = +2 4. Oxidation number of group VIIA element in compound = -1 UNLESS combined with more electronegative element (like oxygen) *In HCl Cl = -1 but in HOCl, Cl = +1 5. Hydrogen has 1+ state when bonded to atom more electronegative and has 1- when bonded to atom less electronegative (Group 1A and IIA) *H = +1 in HCl and -1 in NaH 6. Oxidation number for oxygen usually -2, unless peroxides or when combined with more electronegative element ( OF2 means oxygen has +2 charge) *In peroxides ( O2^2-, like H2O2 and Na2O2) oxygen is in a 1- oxidation state instead of -2 7. Sum of oxidation numbers of all atoms present in neutral compound is 0. 8. If charge gets bigger on element in compound across reaction (going from +2 charge to +4 charge), it has been oxidized because it has lost electrons (making it a reducing agent) 8. The sum of oxidation numbers for all atoms in polyatomic ion = charge of ion (In SO4^2- the sum = -2) *oxidizing agents oxidize other stuff, and thus reduce themselves Nonmetals tend to form anions: Group 17 forms monoatomic anions with charge of -1 Carbon = Can vary from -4 to +4 Note: Oxyanions of transition metals, like MnO4- and CrO4^2- ions have really high/positve oxidation number on the metal, tend to gain electrons in order to reduce oxidation number (make good oxidizing agents) -In oxyanions of halogens (ClO- and ClO2^-), the halogen is assigned positive oxidation state Atoms like manganese/transition metals don't matter, usually positive Trends in oxidation state can be determined by color (Ionicity) -> changes in oxidation state in transition metals usually have color change Na2CO3 +1(2) + -2(3) + C = 0 So, C = +4 *Almost all groups on periodic table have negative oxidation states, especially groups 14-17 (nonmetals) *Put cation first and anion second, so HCl implies H+ and NaH implies H-

Atomic Radius Trend

Atomic Radius: 1/2 the distance between the centers of two atoms connected (equals diameter) Atomic radius decreases from left to right across a period (increasing charge attraction pulls things closer, making them smaller) Atomic radius increases from top to bottom (farther away from nucleus = less attraction) -Helium = smallest Effective nuclear charge increases from left to right across period since number of protons in nucleus increases. Increasing number of positive charge = electron cloud retracts so atomic radius decreases Essentially the opposite of all other trends: Most increase up and right, atomic radii increases going down and left

Avogadro's Principle A 2.0 L samle at 100 deg C and 20 atm contais 5 moles of gas. If 25 moles added at same temp and pressure, what is final volume?

Avogadros principle: All gases at a constant temp and pressure occupy volumes that are directly proportional to the number of moles of gas. Equal amounts of gas at same temp and pressure will occupy equal volumes n/V = k where k is a constant n1/V1 = n2/V2 where n1 and n2 are the number of moles of 2 gases *as the number of moles of gas increases, the volume of gas increases Pressure and temp held constant, so use n1/v1=n2/v2 5 mol/ 2.0 L = 5+ 25 / V2 = 12 L

What is the order of increasing boiling points in the following compounds CH3CH2COOH CH3CH2CHO CH3CH2CH2OH

CH3CH2CHO (Aldehyde, dipole-dipole IMF) CH3CH2CH2OH (Primary alcohol, can form one H bond) CH3CH2COOH (Carboxylic acid, can form two H bonds with carbonyl O grabbing H of another COOH)(More H bonding = higher boiling point)

A chemical reaction has a negative enthalpy and a negative entropy. Which of the following terms necessarily describes this reaction? A. Exothermic B. Endothermic C. Exergonic D. Endergonic

Correct Answer: A Explanation: A reaction with a negative enthalpy is, by definition, exothermic. Because both enthalpy and entropy are negative, this is a temperature-dependent process, and the reaction will be both endergonic and exergonic—but only at particular temperatures, eliminating choices (C) and (D).

The salt KCl is dissolved in a beaker. To an observer holding the beaker, the solution begins to feel colder as the KCl dissolves. From this observation, one could conclude that: A. ΔS°soln is large enough to overcome the unfavorable ΔH°soln. B. KCl is mostly insoluble in water. C. ΔS°soln must be negative when KCl dissolves. D. boiling point depression will occur in this solution.

Correct Answer: A Explanation: Dissolution is governed by enthalpy and entropy, which are related by the equation ΔG°soln = ΔH°soln - TΔS°soln. The cooling of the solution indicates that heat is used up in this bond-breaking reaction. In other words, dissolution is endothermic, and ΔH is positive. The reaction is occurring spontaneously, so ΔG must be negative. The only way that a positive ΔH can result in a negative ΔG is if entropy, ΔS, is a large, positive value

A reaction coordinate for a chemical reaction is displayed in the graph below Image on definition Which of the following terms describes the energy of this reaction? A. Endothermic B. Exothermic C. Endergonic D. Exergonic

Correct Answer: A Explanation: Eliminate choices (C) and (D), describe the free energy of reaction and cannot be determined from this graph. While most reaction coordinate graphs we've explored in this book use free energy for the y-axis, this one uses potential energy (enthalpy). If the heat of formation of the products is greater than that of the reactants, the reaction is endothermic. We can determine this information from their relative positions on the graph: because the products are higher than the reactants, this is an endothermic reaction (Positive delta H). For endothermic and exothermic, the y-axis is potential energy (delta H) and the X axis is reaction coordinate EnderGonic and ExerGonic (Delta G) EndotHermic and exotHermic (delta H)

Explosions are necessarily characterized by: A. ΔG < 0. B. ΔH > 0. C. ΔS < 0. D. T < 0.

Correct Answer: A Explanation: In an explosion, a significant amount of heat energy is released, meaning that the reaction is exothermic (ΔH < 0), eliminating choice (B). The entropy change associated with an explosion is positive because energy is dispersed over a much larger area, eliminating choice (C). If this is true, the expression ΔH - TΔS must be negative, indicating that this is an exergonic process (ΔG < 0). Absolute temperature can never be negative, eliminating choice (D).

Lead is a toxic element that can cause many symptoms, including mental retardation in children. If a body of water is polluted with lead ions at 30 ppb (parts per billion), what is the concentration of lead expressed as molarity? (Note: The density of water is 1 g/mol and ppb = grams per 10^9 grams of solution) A. 1.4 × 10-10 M Pb2+ B. 1.4 × 10-7 M Pb2+ C. 6.2 × 10-7 M Pb2+ D. 6.2 × 10-6 M Pb2+

Correct Answer: B Explanation: 30 ppb of Pb2+ is equivalent to 30 grams of Pb2+ in 109 grams of solution; given the extremely low concentration of lead, we can assume the mass of the water is around 109 grams, as well. From here, this is simply a dimensional analysis question. The units we want at the end are moles per liter (molarity), so we must covert from grams of lead to moles of lead and grams of water to liters of water: *There are 1000 grams in 1 liter of water (30 g lead / 10^9 g H2O) * (1000 g H2O/1 L H2O) * (1 mol of lead / about 200 g/mol) 30/10^6 * 1/200 = 3/20 * 1/10^6 .15 / 10^6 = 1.5 X 10^-7 M

The atomic weight of hydrogen is 1.008 amu. What is the percent composition of hydrogen by isotope, assuming that hydrogen's only isotopes are 1H and 2D? A. 92% H, 8% D B. 99.2% H, 0.8% D C. 99.92% H, 0.08% D D. 99.992% H, 0.008% D

Correct Answer: B Explanation: The easiest way to approach this problem is to set up a system of two algebraic equations, where H and D are the percentages of H (mass = 1 amu) and D (mass = 2 amu), respectively. Your setup should look like the following system: H + D = 1 (percent H + percent D = 100%) 1 H + 2 D = 1.008 (atomic weight calculation) Rearranging the first equation and substituting into the second yields (1 - D) + 2D = 1.008, or D = 0.008. 0.008 is 0.8%, so there is 0.8% D.

Which of the following will cause the greatest increase in the boiling point of water when it is dissolved in 1.00 kg H2O? A. 0.46 mol calcium sulfate B. 0.54 mol iron(III) nitrate C. 1.09 mol acetic acid D. 1.11 mol sucrose

Correct Answer: B Explanation: The equation to determine the change in boiling point of a solution is as follows: ΔTb = iKbm. m is the molality of the solution, and Kb is the boiling point elevation constant. In this case, the solvent is always water, so Kbwill be the same for each solution. What we do need to know is how many particles dissociate from each of the original species. This is referred to as the van 't Hoff factor (i) and is multiplied by our molality to demonstrate a normality (the concentration of the species of interest—in this case, all particles). We'll use normality values to determine which will cause the greatest change in boiling point. i × m (Normality) CaSO4 0.46 * 2 dissolved particles = 0.92 Fe(NO3)3 0.54 * 4 = 2.16 CH3COOH 1.09 * (usually) 1 = about 1.10 (acetic acid is a weak acid and a low percentage of the molecules will dissociate into 2 particles) C12H22O11 1.11*1 = 1.11

Which of the following species is represented by the electron configuration 1s22s22p63s23p64s13d5? I. Cr II. Mn+ III. Fe2+ A. I only B. I and II only C. II and III only D. I, II, and III

Correct Answer: B Explanation: When dealing with ions, you cannot directly approach electronic configurations based on the number of electrons they currently hold. First examine the neutral atom's configuration, and then determine which electrons are removed. Neutral Atom's Configuration Cr0: [Ar] 4s13d5 Mn0: [Ar] 4s23d5 Fe0: [Ar] 4s23d6 Ion's Configuration ----------------- Mn+: [Ar] 4s13d5 Fe2+: [Ar] 4s03d6 Due to the stability of half-filled d-orbitals, neutral chromium assumes the electron configuration of [Ar] 4s13d5. Mn must lose one electron from its initial configuration to become the Mn+ cation. That electron would come from the 4s subshell, according to the rule that the first electron removed comes from the highest-energy shell. Fe must lose two electrons to become Fe2+. They'll both be lost from the same orbital; the only way Fe2+ could hold the configuration in the question stem would be if one d-electron and one s-electron were lost together.

The properties of atoms can be predicted, to some extent, by their location within the Periodic Table. Which property or properties increase going up and to the right? I. Electronegativity II. Atomic radius III. First ionization energy A. I only B. I and II only C. I and III only D. II and III only

Correct Answer: C Explanation: Electronegativity describes how strong an attraction an element will have for electrons in a bond. A nucleus with a larger effective nuclear charge will have a higher electro-negativity; Zeff increases toward the right side of a period. A stronger nuclear pull will also lead to increased first ionization energy, as the forces make it more difficult to remove an electron. The vertical arrow can be explained by the size of the atoms. As size decreases, the positive charge becomes more effective at attracting electrons in a chemical bond (higher electronegativity), and the energy required to remove an electron (ionization energy) increases.

Which of the following statements is true of process that is spontaneous in the forward direction? A. ΔG > 0 and Keq > Q B. ΔG > 0 and Keq < Q C. ΔG < 0 and Keq > Q D. ΔG < 0 and Keq < Q

Correct Answer: C Explanation: For a process to progress forward spontaneously, Q must be less than Keq and will therefore have a tendency to move in the direction toward equilibrium. A spontaneous reaction's free energy is negative by convention.

A saturated solution of cobalt(III) hydroxide (Ksp = 1.6 × 10-44) is added to a saturated solution of thallium(III) hydroxide (Ksp = 6.3 × 10-46). What is likely to occur? A. Both cobalt(III) hydroxide and thallium(III) hydroxide remain stable in solution. B. Some cobalt(III) hydroxide precipitates and thallium(III) hydroxide remains stable in solution. C. Some thallium(III) hydroxide precipitates and cobalt(III) hydroxide remains stable in solution. D. Some of both cobalt(III) hydroxide and thallium(III) hydroxide precipitate.

Correct Answer: C Explanation: Thallium(III) hydroxide has a lower Ksp value than cobalt(III) hydroxide. It is important to note that one can assume the molar solubility of thallium(III) hydroxide is lower than cobalt(III) hydroxide only because both salts have a formula MX3 (one of one particle, three of another). When the solutions are mixed, [OH-] is at saturation levels in the cobalt solution—which is higher than saturation levels in the thallium solution. Therefore, the ion product for thallium(III) hydroxide is higher than its solubility product constant, and the system will shift left to form solid thallium(III) hydroxide, which precipitates. So, if you combine two saturated solutions, the TlOH3 is more likely to precipitate out given the relatively large increase in the concentration of OH compared to what is was before for the saturated TlOH3 solution on its own. In comparison, the amount of OH in solution has not changed much for the CoOH3 solution, so it will not be pushed over the edge and begin to precipitate. Precipitates will form once your Qsp exceeds your Ksp. Since both solutions are saturated your Qsp= Ksp. However upon mixing, the hydroxide shifts the Qsp of thallium hydroxide over its Ksp( because it's 100x less than the other compound). As a result, thallium hydroxide precipitates.

An electron returns from an excited state to its ground state, emitting a photon at λ = 500 nm. What would be the magnitude of the energy change if one mole of these photons were emitted? (Note: h = 6.626 × 10-34 J·s) A. 3.98 × 10-21 J B. 3.98 × 10-19 J C. 2.39 × 103 J D. 2.39 × 105 J

Correct Answer: D Explanation: E=hc/wavelength where h = 6.626 × 10-34 J·s (Planck's constant), 3.00X10^8 is the speed of light, and λ is the wavelength of the light. This question asks for the energy of one mole of photons, so we must multiply by Avogadro's number, NA = 6.02 × 1023 mol-1. (6.6X3)(10^-34 X 10^8)/(500X10^-9) X (6X10^23) (100/500 X 10^-6) J = 2X 10^5

How many valence electrons are present in elements in the third period? A. 2 B. 3 C. The number decreases as the atomic number increases. D. The number increases as the atomic number increases.

Correct Answer: D Explanation: This question is simple if one recalls that periods refer to the rows in the Periodic Table, while groups or families refer to the columns. Within the same period, an additional valence electron is added with each step toward the right side of the table.

The Haber-Bosch process creates ammonia through several reactions, the final step of which is N2+3H2 -> 2NH3 (delta Hrxn = -93kJ/mol, delSrxn=-198 J/mol*k). Determine what Gibb's free energy of this reaction is at standard conditions and at 500K: At what temperature would the reactions described in previous questions be at equilibrium if you were to suddenly flood the reaction vessel with significant amounts of ammonia, what would occur

Delta G = Delta H - T Delta S = -93000 J/mol - 300(-200 J/mol*K) = about -33 kJ/mol At 500K: -93000 - (500X200) = about 7 kJ/mol At equilibrium = Delta G = 0 = -93000 J/mol - T(-200 J/mol*K) 93000 = 200T T = about 470 K the value of Q would increase significantly (more products/reactants), causing the system to shift left, forming more reactants until the system again reached equilibrium Q not Keq because Q has same form but can be calculated at any concentrations of reactants and products

The Bohr Model; Niels Bohr; 1913

Developed model of the electronic structure of the hydrogen atom Predicted that the possible values for the angular momentum of an electron orbiting H nucleus could be given by L = nh / 2 pi where n= principal quantum number, h is Planck's constant Bohr then related the permitted angular momentum values to the energy of the electron to obtain: E = =RH/ n^2 where RH = Rydberg unit of energy = 2.18X 10^-18 J/electron *Energy increase as principle quantum number (n) increases *Energy is less negative (greater) as it moves further out from nucleus (increasing n) Defined orbit with smallest, lowest energy radius as ground state (n=1) (all electrons in lowest possible orbital) -Larger radius = higher energy = excited state (at least one electron has moved to a sub shell of higher than normal energy) *From Bohr we also learned that electrons are not restricted to specific pathways, but tend to be localized in certain regions of space *Useful for explaining atomic emissions nd absorption spectra -Didnt take into account repulsion from multiple electrons -Bohr thought electrons followed defined, circle path (they actually move rapidly and are localized in orbitals)

Max Planck (1900)

Developed the first quantum theory, proposing that energy emitted as electromagnetic radiation from matter comes in discrete bundles called quanta. The energy of a quantum is given by Planck's relation: E = hf where h is proportionality constant known as Planck's constant (6.626 X 10^-34 J*s) and f (Sometimes greek letter nu, v) is the frequency of radiation

Hund's Rule

Electrons will fill degenerate orbitals or orbitals of identical energy with parallel spins (+1/2 and +1/2) first before filling higher energy orbitals. Electrons remain unpaired if possible in order to minimize electron-electron repulsion Orbitals filled so there is max amount of half-filled/parallel spins Applies when subshells contain more than 1 orbital (like p contains 3) Half filled and fully filled have lower energy (more stable) 2 exceptions to hunds rule: Elements in chromium group (column) and elements in copper group (column) -Chromium should be [Ar] 4s^2 3d^4, but its actually 4s^1 3d^5 so more subshells are half filled -Copper = [Ar] 4s^2 3d^9 but its actually 4s^1 3d^10 -Nothing ever shifts from p sub shell

Hess' Law Standard heat/enthalpy of formation (ΔHf^o) Standard Heat/enthalpy of Reaction

Enthalpy is a state function and property of eq state (State functions are always path independent) Hess's Law states that enthalpy changes of reactions are additive (can be used to calculate total enthalpy change for series of reactions) *Total change on potential energy of a system is equal to the changes of potential energies of individual steps *Applies to ANY state function, including entropy and Gibbs free energy Standard enthalpy change (Delta H) = sum of heat of formation of products - sum of the heat of reactants = heat of reaction Heat of formation for O2 gas not included -Can be expressed in terms of bond enthalpies/ bond dissociation energies (average energy required to break bond between atoms in gas phase)(Endothermic because it takes energy to pull apart) (Given in units of kJ/mols of bonds broken) Bond formation is the opposite, same magnitude but negative (exothermic, energy released) (usually makes atoms more stable Endothermic: Increase in heat content of system from surroundings (Delta H >0) Exothermic: release of heat contents from system (Delta H < 0) Standard enthalpy of formation (ΔHf^o) = enthalpy required to produce 1 mole of compound from elements in standard states (most stable physical state of element or compound at 298 K and 1 atm) ΔHfo of element in stand state = 0 Standard enthalpy of reaction: ΔHrxn^o: Enthalpy change accompanying reaction carried our under standard conditions Calculated from difference between sum of standard heats of formation for products and reactants ΔHrxn^o=∑ΔHf^o{products}−∑ΔHf^o{reactants} Enthalpy: Measure of potential energy of system found in intermolecular attractions and chemical bonds -Can be calculated using heats of formation, heats of combustion, of bond dissociation energies

Electronegativity Trend

Measure of attractive force atom will exert on electron oil chemical bond Greater electronegativity = more it attracts electrons within a bond -Related to ionization energy (Lower IE = lower electronegativity) (Except for first three noble gases, which have high IE but no electronegativity because they dont form bonds) Usually measured with Pauling electronegativity scale (ranges form 0.7 (Cs) to 4.0 = Most electronegative = Fluorine) -Increases from left to right (across period) -Increases from bottom to top

The following equilibrium exists when AgBr (Ksp = 7.7 × 10-13) is in solution: AgBr (s) ←→ Ag+(aq) + Br-(aq) What is the solubility of AgBr in a solution of 0.0010 M NaBr?

Explanation: The solubility of AgBr can be determined using the Ksp value given in the equation Ksp = [Ag+][Br-]. Some amount of AgBr will dissolve. If we call this amount x, then there will be x amount of silver(I) formed and x amount of bromide—which is added to the 0.0010 M already present from NaBr. 7.7 × 10^-13 = [x][0.0010 + x] Remember that x is always very small. Even though 0.0010 M is also very small, it will still be much larger than the value of x on Test Day. Thus, the math can be simplified to: 7.7 × 10-13 = (x)(0.001)M. x = 7.7 X 10^-10 Multiply by molar mass of AgBr = about 188 g/mol = About 1.5 X 10^-7 g/L

Determine the standard change in free energy of a cell with the following net reaction: 2Fe3+ (aq) + 2Cl- (aq) -> 2Fe2+ (aq) + Cl2 (g) Standard reduction potential of: Iron (III) = + 0.77 V Chlorine = +1.36 V

First, split into half reactions 2Fe3+ + 2e- -> 2Fe2+ (reduction) 2Cl- -> Cl2 + 2e- (oxidation) Reduction (gaining electrons) always happen at cathode Electrons always go from anode to cathode, current always goes from cathode to anode Determine emf: Cathode - anode 0.77 - 1.36 = -0.59 V Now use free energy change ΔG°cell = −nFE°cell n = 2 because 2 electrons were transferred F = about 10^5 C/mol e- E°cell= -0.59 V = about 1.2X10^5 J = non-spontaneous

Ernest Rutherford (1910)

Gold foil experiment: Experimental evidence that atom has a dense, positively charged nucleus that accounts for only a small portion of the atom's volume

The kinetics of any reaction will __________ when the temperature increases

The kinetics of any reaction will increase when the temperature increases Increase temp = increase kinetic energy = increase probability that transition state increases = rate of reaction will increase = greater formation

Arrhenius Equation

K=Ae^(-Ea/RT) where k is the rate constant / rate of reaction A is frequency factor/Attempt frequency (measure of how often molecules in a certain reaction collide (units = s^-1)(Can be increased by increasing number of molecules (more molecules/higher concentration = more collisions) Ea is activation energy R is ideal gas constant T is temperature *Low activation energy and high temp make the negative exponent smaller and and thus increase the rate constant (k) Used for analysis of collision theory Rate dependent on temperature, activation energy, gas constant, and frequency factor * Enthalpy does not affect rate of reaction *rate of reaction determined by Ea (activation energy), not Delta G

What is the concentration of Ag+ in a saturated solution of AgCl at 25 degC? Ksp of AgCl = 1.8 X 10^-10

Ksp = [Ag^+][Cl^-] Assuming molar solubility of AgCl is X, Ksp = X^2 X = Sqr rt (1.8 X 10^-10) X = (Sqr rt (1.8)) X 10^-5 Taking square root of negative exponents divides exponent by two!

Periodic Trends: Left to Right (across a period): Atomic Radius Ionization Electron Affinity Electronegativity Top to bottom: Atomic Radius Ionization Electron Affinity Electronegativity

Left to Right: Atomic Radius: Decreases Ionization: Increases Electron Affinity: Increases Electronegativity: Increases Top to bottom: Atomic Radius: Increases Ionization: Decreases Electron Affinity: Decreases Electronegativity: Decreases *Atomic radius is opposite, ionic radius is variable Cs = largest, least electronegative, lowest ionization energy, least exothermic (lowest) electron affinity F = smallest, most electronegative, highest ionization energy, most exothermic (highest) electron affinity

Atomic mass / Mass Number VS Atomic Number VS Atomic weight

Mass number (A) = Protons + neutrons (on top) -Atomic mass (slightly less but pretty much) = mass number: varies from one isotope to another Mass number - Atomic number (Z) (number of protons) = # of neutrons Atoms of the same element all have the same atomic number (Z)(below) but may may different masses because of different amounts of neutrons Atomic weight is constant (reported on periodic table) (Average of isotope weights) -Represents both the mass of the "average" atom of that element in amu and the mass of one mole of the element in grams -3 isotopes of H are protium, deuterium, and tritium

A scientist wants to prepare a 50 mM solution of NaOH. If he starts out will 60 mL of 0.3 M NaOH and adds pure water until reaching the desired concentration, how much water will he have added?

Molarity X Volume = constant M1V1 = M2V2 60 mL * 0.3 M = 0.05 M V2 V2 = 360 mL

Bronsted-Lowry acid and base

More inclusive than Arrhenius Acid donates hydrogens (H+) and base accepts hydrogen's Not limited to aqueous solutions Bases: OH- (also Arrhenius base), NH3, F- (all have ability to accept H) *Both Arrhenius and BL say acid must produce H ions, but Arrhenius require aqueous medium and the acidity of water: Water is not considered Arrhenius acid because it does not produce excess H+, but it is considered BL acid because it can donate proton (Usually classified like this) *BL acids and bases always occur in pairs (conjugate acid-base pair) BRONSTED LOWRY ACID HAS TO HAVE HYDROGENS

Which will be the most soluble? CO2 gas in 348 K water N2O gas in 348 K water N2O gas in 298 K water

N2O gas in 298 K water Solvent (water) is polar, so it dissolves polar better than non-polar N2O is more soluble Gas dissolving liquid is an exothermic process, so it will be more soluble at lower temp (Gas -> liquid -> solid is exothermic, solid -> liquid -> gas is endothermic)

Entropy

Not just measure of disorder, entropy is the measure of spontaneous dispersal of energy at a specific temperature: how much energy is spread out, how widely it is spread out -Ratio of heat transferred per mole per unit kelvin -State function: Doesn't matter how you went from final to initial (whether it happened fast or slow or whatever, only depends on where it started and where it arrived) (pathway independent) The chemical energy of all molecules and atoms is released into the environment during the process of death or decay Second law of thermodynamics: Energy spontaneously disperses from being localized to becoming spread out if its not hindered from doing so (entropy will increase unless hindered)(However, energy can be concentrated, but this rarely happens spontaneously) Entropy Equation: S = Q rev / T where delta S is change in entropy, Q rev is heat that is gained or lost in a reversible process, and T is temp in K Measured in units J/mol * K Energy distributed out of system = entropy decrease ENTROPY IS MAXIMIZED AT EQUILIBRIUM Freezing is accompanied by decrease in entropy (disorder liquid becomes well ordered solid) and boiling is accompanied by increase in entropy (Liquid becomes more disordered gas). Sublimation has greatest increase in entropy N2 (g) + 3H2 (g) -> 2 NH3 (g) is decrease in entropy because there are fewer moles of gas Ice pack placed on wound = increase in entropy because heat is transferred Gases have highest entropy

Which of the following would result in the shortest bond in an atom? Two pi bonds Two Sigma bonds One sigma bond and one pi bond only

One sigma bond and one pi bond only The first bond between any two molecules is sigma bond. After two molecules are bound with sigma bond, pi bonds form to strengthen the bond. Without the sigma bond, no pi bond can exist, but only one sigma bond can form

By what % does real pressure of 1 mole of ammonia in 1 L flask at 227 deg C deviate from its ideal pressure (R = 0.821 L*atm/mol*K, for NH3, a = 4.2 and b = 0.037)

P= nRT/V =(1mol X 0.0821 X 500K)/ 1L = (0.0821 X 1000)/ 2 According to van der Waals equation of state, P = (nRT/ V-nb) - (n^2 * a/ V^2) = ( 1 mol * 0.0821 * 500/ 1L-(1mol * 0.037)) - ( 1 mol ^2 * 4.2 / 1 L^2) = (41.5 / 0.963 ) - 4.2 = about 39 atm Thus, the pressure is about 2.7 atm less than would be predicted from ideal gas law Error of 2.7 atm / 41.5 atm X 100% = about 7.5%

Paramagnetic vs diamagnetic

Paramagnetic: unpaired electrons in orbital diagram -Unpaired electrons orient spins with/are weakly attracted to magnetic fields -PARAmagnetic = PARAllel spins in unpaired electrons = attraction Diamagnetic: all electrons have a pair -Slightly repelled by magnetic field (because they are attracted to each other) Note: Allotrope = configuration

Phase Change Diagram

Phase changes are reversible (rates of forward = reverse processes)(can reach equilibrium: relative amounts of ice and water remain constant for example) Phase changes are temperature dependent processes Entropy (measure of disorder in J/k*mol) increases when a substance changes from a solid to a liquid, a liquid to a gas, or a solid to gas Gas-Solid Equilibrium: (line C) Sublimation = change in phase from solid to gas = greatest increase in entropy (can be done using cold finger device) -Gas to solid = deposition Gas-Liquid Equilibrium: (line B) Generally, gases have greatest amount of entropy and solids have smallest amount of entropy Liquid phase, molecules are relatively free to move around, some have enough kinetic energy to leave liquid phase into gaseous phase = evaporation/vaporization *each time liquid loses high-energy particle, temp decreases *Evaporation is an endothermic process for which heat source is liquid water *Evaporation happens in all liquids at all temps, boiling can only occur above boiling point and involves vaporization through the entire volume of liquid Condensation: Closed container, gas forced back to liquid (facilitated by lower temp or higher pressure) *Vapor pressure: Pressure gas exerts over liquid at eq: increases as temp increases *Boiling point: Temp where vapor pressure of liquid = ambient/external pressure Liquid-Solid Equilibrium: (line A) Solids can undergo vibrational motions even though atoms are confined (increase when heat applied)(called micro states) -Solid to liquid = Fusion/melting (melting is endothermic process, heat absorbed ) Liquid to solid = Solidification, crystallization, freezing Solids (except crystals) tend to have more broad melting points On the graph: Gas generally found at high temp and low pressure, solid at low temp and high pressure-All three phase boundaries meet at triple point: temp and pressure where 3 phases exist in equilibrium *Phase boundary between solid and liquid extends forever, but phase boundary between liquid and gas stops at the critical point (supercritical fluid, can't tell between gas and solid, heat of vaporization above critical point = 0)

Primary Alcohol with PCC Secondary Alcohol with PCC

Primary Alcohol with PCC: Aldehyde Secondary Alcohol with PCC: Ketone

Acid Anhydride

RCOOCOR Less reactive than Acyl halides but more reactive than esters and amides

Electron Configuration

The pattern by which subshells are filled and # of electrons Spectroscopic notation: 1st # = principle, letter designates sub shell, subscript gives number of electrons in that sub shell -Electrons fill from lower to higher energy subshells according to Aufbau principle/ building up principle n+l rule : rank by increasing energy (larger sum of n+l = larger energy) -If values are same, larger n value has more energy (6s more energy than 5d) Take atomic number (which gives you number of electrons) Add up exponents, this will equal atomic number

Real Gases

Real gases have particles that occupy non negligible volumes and interact in measurable ways Deviation especially when gas forced into close proximity under high pressure (at low volume) or at low temp At high temperature and low pressure (high volume), derivation from ideality are usually small, good approximations can still be made from ideal gas law Deviations due to pressure: Pressure of gas increases = particles pushed closer = condensation pressure for given temp approached = intermolecular attraction forces becomes more and more significant until gas condenses to liquid At moderate high pressure (few hundred atm), volume less than predicted by ideal gas laws due to intermolecular attraction *at moderate high pressure, low volume or low temp, real gases will occupy more volume than predicated Very High pressure = particle size becomes relatively large compared to distance = gas takes of larger volume that predicted *extremely high pressure, lower volumes, or lower temp, real gases will occupy more volume than predicted While the ideal gas law assumes that a gas can be compared to 0 volume, this isn't possible Deviation due to temp: Temp of gas decreased = speed decrease = attractive intermolecular forces become more significant (condensation temp -> gas eventually becomes liquid) As temp reduced to condensation point/boiling point, intermolecular attraction causes gas to have smaller volume than predicted by IGL Closer gas is to boiling point, less ideally it acts Extremely cold = gas will occupy more space than predicted because particles cant be c oppressed to zero volume

Calculate the enthalpy change for the following reaction: C(s) + 2H2(g) -> CH4(g) deltaH= ? Bond dissociation of H-H is 436kJ/mol and C-H is 415kJ/mol and deltaHf of C(g)= 715kJ/mol

Remember that bond breaking is endothermic (+) and bond making is exothermic (-). Energy must be added to break a bond and energy is given off when a bond is formed. ΔHrx = ΣΔH bonds broken - ΣΔH bonds formed = energy absorbed = energy released ΔHrx = 2(H−H) + (-4(C−H) + ΔHvap C ΔHrx = 2(436 kJ) + 4(-415kJ) + 715kJ ΔHrx = -73 kJ

Reversible Reactions /Equilibrium

Reversible reactions eventually reach state in which energy is minimized and entropy is maximized -can proceed in either forward or reverse directions •Forward: toward the products or to the right -Reverse: toward the reactants or to the left •Often don't proceed to completion b/c products can react together to reform reactants Chemical equilibria are dynamic Dynamic equilibrium: When rate of forward reaction = rate of reverse reaction (reactions still occurring, unlike static equilibrium, just occurring at same rate (no net change in concentrations of products or reactants -At equilibrium, concentrations of A and B are constant, but not necessarily equal, and A -> B and B-> A occur at same rate Equilibrium: Balance between forward and reverse reactions *Entropy: measure of the distribution of energy through out system or between system and environment -Reverse reaction reaches equilibrium when entropy (energy distribution) is at maximum and Gibbs freee energy is at minimum

Hydrogen sulfide (H2S), methyl salicylate (C8H8O), benzaldehyde (C7H6O) all have distinct smells. Which would you smell first?

SAME TEMP AND KINETIC ENERGY, SO the lightest travels first. H2S first, benzaldehyde, then methyl salicylate

Bond angle associated with: SP3 with four single bonds (most associated with SP3)(tetrahedral) SP3 with one lone pair (pyramidal, like ammonia/NH3) SP3 with two lone pairs (Bent) SP2 with planar geometry: SP hybridized with triple bonds (linear geometry)

SP3 with four single bonds: 109.5 degrees SP3 with one lone pair (pyramidal, like ammonia/NH3): 107 SP3 with two lone pairs: 104.5 SP2 with planar geometry: 120 SP hybridized with triple bonds: 180

Quantum Numbers Pauli Exclusion Principle

Set of numbers used to completely describe an electron (size, shape, orientation): n, l, ml, and ms (getting more specific -> n limits value of l, which limits value of ml....) Pauli Exclusion Principle : no two electrons in atom can posses same set of 4 #'s *Quantum = energy difference between levels Position and energy as decreased by 4 #'s = energy state Principal Quantum Number (n) -Larger n = larger radius of e- shell and higher energy level -Within each shell, how many electrons it can hold is given by 2n^2 -Difference in energy between 2 shells decreases as distance from nucleus increases ( 1/ni^2 - 1/nf^2) -As one moves down group, increases by 1 each time (VE increasingly separated from nucleus by more inner shells/energy levels = less attraction between VE and nucleus) = VE held less tightly to nucleus moving down group Azimuthal Quantum Number: Angular momentum (l) -Refers to shape and number of subshells within principal energy level (shell) -range of values for l is 0 to (n-1) -> n = number of possible subshells) -If n = 2, l = 0 and 1 -l can be designated as letter (l=0 called s, 1=p, 2=d, 3=f) -Within each subshell, maximum amount of electrons that can be held = 4l +2 -energy increases with increase in l, but 4s has less energy than 3d Spectroscopic Notation: Shorthand representation of n and l Magnetic Quantum Number (ml)(number of orbitals) *2l+1 possible values for ml = n^2 *Values range from -l to l *Specifies particular orbital (can hold 2 e-) within sub shell *possible values = -1 to +l (f-subshell has 7 orbitals because it goes from -3 to +3, including 0) -s-subshell has 1 spherical orbital, 3 p orbitals are dumbbell shaped -2p contains 3 orbitals = up to 6 electrons (atomic number increase -> max number of electrons increase) *p block contains 6 elements, s contains 2, d contains 10, f contains 14 Spin Quantum Number (ms) -Two spin orientations designated +1/2 or -1/2 -Within same orbital have to have opposite spins

Calculate the enthalpy for this reaction: 2C(s) + H2(g) ---> C2H2(g) ΔH° = ??? kJ Given the following thermochemical equations: C2H2(g) + 5⁄2O2(g) ---> 2CO2(g) + H2O(ℓ) ΔH° = −1299.5 kJ C(s) + O2(g) ---> CO2(g) ΔH° = −393.5 kJ H2(g) + 1⁄2O2(g) ---> H2O(ℓ) ΔH° = −285.8 kJ

Solution: 1) Determine what we must do to the three given equations to get our target equation: a) first eq: flip it so as to put C2H2 on the product side b) second eq: multiply it by two to get 2C c) third eq: do nothing. We need one H2 on the reactant side and that's what we have. 2) Rewrite all three equations with changes applied: 2CO2(g) + H2O(ℓ) ---> C2H2(g) + 5⁄2O2(g) ΔH° = +1299.5 kJ 2C(s) + 2O2(g) ---> 2CO2(g) ΔH° = −787 kJ H2(g) + 1⁄2O2(g) ---> H2O(ℓ) ΔH° = −285.8 kJ Notice that the ΔH values changed as well. 3) Examine what cancels: 2CO2 ⇒ first & second equation H2O ⇒ first & third equation 5⁄2O2 ⇒ first & sum of second and third equation 4) Add up ΔH values for our answer: +1299.5 kJ + (−787 kJ) + (−285.8 kJ) = +226.7 kJ

What is the electron configuration of osmium (76)? What is the electron configuration of Fe3+? Which of the following is the correct electron configuration for Zn2+? A. 1s22s22p63s23p64s03d10 B. 1s22s22p63s23p64s23d8 C. 1s22s22p63s23p64s23d10 D. 1s22s22p63s23p64s03d8

Start with Xenon (noble gas before) Xe = 54 Start from H at top, count down to 6s at Cs, passes through 4f series, count down from boron to 5d, and osmium is the sixth element in the sub shell. So, [Xe] 6s^2 4f^14 5d^6 [Ar] 3d^5 Electron configuration of Fe is [Ar] 4s^2 3d^6, and we subtract from 4s first because it has higher principal quantum number Correct Answer: A Explanation: Remember that when electrons are removed from an element, forming a cation, they will be removed from the subshell with the highest n value first. Zn0 has 30 electrons, so it would have an electron configuration of 1s22s22p63s23p64s23d10. The 4s subshell has the highest principal quantum number, so it is emptied first, forming 1s22s22p63s23p64s03d10. Choice (B) implies that electrons are pulled out of the d orbital, choice (C) presents the configuration of the uncharged zinc atom, and choice (D) shows the configuration that would exist if four electrons were removed.

Strecker Synthesis of Amino Acids (2 step process)

Step one: Aminonitrile formation Step two: Protonation of aminonitrile and attack by water to turn the nitrile into a carboxylic acid

Enthalpy (H)

The heat content of a system at constant pressure; the change in enthalpy (delta H) in the course of a reaction is the difference between the enthalpies of the products and the reactants.

Half-lives Equation for Radioactive Decay

The time it takes for half the amount of a radioactive substance to disappear as a result of radioactive decay Corresponds with stability Helps us determine relative proportions of different isotopes. [A] = A₀(e^-kt)

Molecular geometry

Trigonal Planar: 3 bonds, 0 lone pairs SP2 hybridized (making it planar) No lone pairs (trigonal) BF3

Valence Electrons

Valence electrons: more likely to bond because they experience least amount of electrostatic pull from nucleus -Farthest from nucleus, greatest amount of Potential energy -Largely determine chemical reactivity and properties of elements Outermost energy shell/ Electrons in highest n (so if you have 2 in 4s and 2 in 4d you would have 4 VE) Valence electrons = for transition elements, VE found in s and either d or f orbitals Roman numeral above each group represents VE

Which electrons are the valence electrons of elemental vanadium, selenium, and sulfur atom in sulfate ion?

Vanadium = 5 VE (2 in 4s and 3 in 3d) VE. : [Ar] 3d3 4s2 (5 from the left, 5 VE. So Ti, which is 4 to the left, has 4, cobalt has 9, nickel has 10 (doesn't work for column with iron or last two transition elements.... when in doubt, say its 2) Selenium = 6 (2 in 4s and 4 in 4p): [Ar] 3d10 4s2 4p4 REMEMBER WE ARE LOOKING AT HIGHEST N (ENERGY LEVEL) WHICH IN THIS CASE IS 4 SO WE INCLUDE ELECTRONS FROM 4S AND 4P Sulfur: [Ne] 3s² 3p⁴ 12 VE (6 from sulfur + 6 from oxygen that its bonded to)(3s and 3p can only hold 8, so 4 are in 3d which is normally empty )

Carboxylic Acids What can not be oxidized to form a carboxylic acid? What can be oxidized to form a carboxylic acid?

What can not be oxidized to form a carboxylic acid? Ketones Secondary and tertiary alcohols What can be oxidized to form a carboxylic acid? -Primary alcohols -Alkyl Benzene (KMnO4) -Aldehyde

Gibbs free energy equation A reaction will be spontaneous at all temperatures under which set of conditions?

ΔG = ΔH - TΔS (Delta H and Delta S also tells us if process is temp dependent) Measure of change in enthalpy and entropy Indicates whether reaction is spontaneous or non spontaneous TΔS represents total amount of energy absolved by system when entropy increases reversibly Change in free energy is max amount of energy released by process at constant temp and pressure Spontaneous Process: a process that can occur without an input of energy (may have high activation energy, some may proceed very slowly, can be sped up by enzymes (biological catalysts) Systems move in direction the results in reduction of free energy *Spontaneous reactions may not go to completion and settle in equilibrium. Common method for supplying energy for nonspontanous reactions is by coupling them to spontaneous ones (combustion of glucose is exergonic, formation of peptide bind is endergonic, energy from combustion of glucose stored in bonds in GTP, which are lysed to provide energy for forming peptide bonds) The change in Gibbs free energy determines whether or not a reaction is spontaneous +delta G = endergonic/nonspontaneous = energy absorbed *If the activation energy of the forward reaction is greater than the activation energy of the reverse reaction, then the products must have a higher free energy than the reactants. The overall energy of the system is higher at the end than it was in the beginning. The net free energy change is positive, indicating an endergonic (nonspontaneous) reaction. - Movement towards equilibrium = decrease in Gibbs FE = -delta G = exergonic/spontaneous = energy given off - Movement away from equilibrium = increase in Gibbs FE = Delta G > 0 = nonsponteneous (Graph has products higher than reactants) (Reaction proceeds in reverse) Spontaneous at all temps/ Exergonic: (remember, spontaneous doesn't mean fast) Delta G < 0 (reaction proceeds forward) Delta G is negative S>0 and H<0 (S is positive and G is negative) Delta G is temp dependent when delta H and delta S have the same sign! Spontaneous at low temps: Delta S and Delta H < 0 (small T Delta S) Spontaneous at high temps: Delta S and Delta H > 0 (Large T delta S) Not spontaneous/endergonic: Delta G > 0 Delat H > 0 and Delta S < 0 If Delta G = 0, the system is in state of equilibrium where delta H = T delta S Changes in Gibbs free energy must = 0 so Delta G = G (gas) - G (solid) = 0


Related study sets

Chapter 4 Loops and Files (reading)

View Set

Tversky and Kahneman- Availability heuristic

View Set

Pituitary Disorder NCLEX Questions

View Set

CHAPTER 2 (Business Pressures, Organizational Responses, and IT Support)

View Set

Wireless Networking Quiz N10-007

View Set

Economics Today The Macro View Ch. 13 Fiscal Policy

View Set